Download as pdf or txt
Download as pdf or txt
You are on page 1of 184

Surgery

1.A mother brings her 2-week-old male neonate with noisy breathing which becomes
worse when the baby is laid on his back. The child is otherwise healthy.
Which one of the following is the most likely cause of this presentation?
A Acute asthma.
B Laryngomalacia.
C Croup.
D Acute bronchiolitis.
E Acute epiglotitis.
Option B is correct The clinical history is suggestive of laryngomalacia. Laryngomalacia is
congenital softening of the tissues of the larynx above the vocal cords. This is the most
common cause of noisy breathing in infancy. The laryngeal structure is malformed and floppy,
causing the tissues to fall over the airway opening and partially block it. Laryngomalacia
symptoms are usually present at birth, and can become more obvious within the first few weeks
of life. Most children outgrow laryngomalacia by 18 to 20 months of age. Symptoms of
laryngomalacia include: Noisy breathing - an audible wheeze when a baby inhales. It is often
worse when the baby is agitated, feeding, crying or sleeping on his back High pitched sound
Difficulty feeding (in severe cases) Poor weight gain (in severe cases) Choking while feeding
(in severe cases) (Option A) Acute asthma makes breathing difficult, and wheeze is audible in
any body position. Asthma is an extremely rare possibility in a 2-week-old neonate. (Option C)
Croup (laryngotracheobronchitis) is most commonly caused by para-influenza virus type 1
infection. It is characterized by a seal barking cough and inspiratory stridor. Croup affects
mainly children aged 6 month to 3 years. This child is in no respiratory distress, is otherwise
healthy, and does not have any couhgh. (Option D) Acute bronchiolitis is an acute viral
infection of the lower respiratory tract, typically affecting infants (≤24 months) and is
characterized by respiratory distress, wheezing, and inspiratory fine crackles. It is primarily
caused by respiratory syncytial virus (RSV). Treatment is supportive with oxygen and
hydration. Prognosis is generally excellent; however, some chidlren may develop apnea or
respiratory failure. (Option E) Epiglottitis is a rapidly progressive bacterial infection of the
epiglottis and surrounding tissues that may lead to sudden respiratory obstruction and death.
Symptoms include severe sore throat, dysphagia, high fever, drooling, and inspiratory stridor.
Treatment includes airway protection and antibiotics. This child does not have any of these
symptoms.

2. Which one of the following statements is correct about lipomas?


A Lipoma is a premalignant lesion.
B Lipomas are usually lobulated.
C Lipomas cannot be tethered to the skin.
D Lipomas often occur in the scrotum.
E Lipomas are invariably subcutaneous.
Option B is correct Lipomas are common benign tumors of mature fat cells that can be seen in
any site of the body containing fatty tissue. They are not premalignant. Although they
frequently occur in the subcutaneous fat tissue, it is not uncommon to see lipomas in the deeper
layers such as beneath the fascia or within muscles. As there is very little fat in the scrotum,
lipomas almost never occur there, but fat deposition within the spermatic cord may be seen.
Unlike lipomas, epidermoid cysts of the scrotum are common. Lipomas are usually freely
mobile and not attached to the overlying skin; however in areas such as the back of the neck or
the trunk, where the skin has less mobility, they could be less mobile or even immobile. The
most characteristic physical feature of a lipoma is the lobulated contour, which can reliably
differentiate it from an epidermoid cyst and its smooth contour.

3 A 68-year-old man presents with a history of headache for the past several months. The
headache is worse on waking up, coughing, sneezing and bending forward. He also
mentions that he has been suffering from cough for the past 3 months. His history is
significant for smoking of 45 cigarettes a day for the past 45 years. Significant finding on
examination is left hemiparesis. A chest X-ray reveals an opacity of 3x2 cm in size in the
left lung field.

A contrast CT scan of the head is obtained which is shown in the following picture. Which
one of the following could be the most likely diagnosis?
A Primary brain tumor.
B Metastatic brain tumor.
C Brain abscess.
D Cerebral hemorrhage.
E Ischemic stroke.
Option B is correct The clinical picture of headache worse in the morning, and by sneezing,
coughing and bending forward is suggestive of a space-occupying lesion in the brain that can
be either a tumor or abscess. Considering the age of the patient, history of heavy smoking and
the cough, this patient has lung cancer with high certainty. Lung cancer is one of the three
primary cancers that tend to metastasize to the brain (21%). The other two are melanomas
(40%) and breast cancer (9%). With this history and chest X-ray findings, the lesion is most
likely to be a metastatic brain lesion from a primary lung cancer. Metastatic brain tumors are
the most common form of brain tumors. Of all brain tumors, more than a half are metastatic. In
patients with systemic malignancies, brain metastases occur in 10- 30% of adults and 6-10% of
children. (Option A) With the high probability of an underlying lung cancer in this patient, this
tumor is more likely to be metastatic than primary. (Option C) Brain abscess has similar
appearance on contrast CT scan (ring enhancement); however, with absence of fever and
considering the history, a brain abscess is not a likely diagnosis. (Options D and E) The
appearance of the lesion as well as the insidious onset and progression of the symptoms make
cerebral hemorrhage and ischemic stroke unlikely diagnoses.

4 The lesion shown in the accompanying photograph has been present on the finger of a
56-year-old woman for the past 8 months. It has slowly enlarged since then and is causing
mild discomfort. Which one of the following is the most likely diagnosis?

A Chronic paronychia.
B Heberden’s node of osteoarthritis.
C Pyogenic granuloma.
D Mucous (synovial) cyst of the finger.
E Rheumatoid nodule.
Option D is correct Mucous (synovial) cysts of the finger are subcutaneous cystic lesions
found on the dorsal aspect of the distal phalanx, distal to the distal interphalangeal joint and
overlying the germinal nail bed. They may cause distortion of the nail growth. They are located
in the midline or laterally. They grow slowly and may undergo spontaneous resolution.
Recurrence is likely if they are inadequately excised. (Option A) Chronic paronychia is
inflammation of the nail fold usually caused by staphylococcus aureus or fungi. The
characteristic feature is a painful swelling and erythema of the whole nail fold (crescent-
shaped). (Option B) Heberden’s node of the osteoarthritis is a bony lesion over the distal
interphalangeal joints. (Option C) Pyogenic granuloma is a fleshy red vascular structure
usually followed by a minor trauma to the fingers. It is painless.

(Option E) Rheumatoid nodules are periarticular


nodules seen in patients with rheumatoid arthritis.

5 Title A lump in the mouth of a 37-year-old man Last revision.On a routine health
examination, the lesion shown in the following photograph is found in the inner side of the
lower lip of a 37-year-old man. The lesion is painless. Which one of the following is the

most likely diagnosis?


A Sebaceous cyst.
B Peutz - Jegher's syndrome.
C Squamous cell carcinoma of the lip.
D Mucoid cyst.
E Basal cell carcinoma.
Option D is correct The lesion shown has a bluish glistening color and is dome-shaped. These
are characteristics of a benign mucoid cyst resulted from an obstructed mucoid salivary gland.
If persistent or bothersome, an incision will be needed for evacuation of the cyst. (Option A)
Sebaceous cysts are caused by obstruction of a sebaceous gland in hair-bearing skin. (Option B)
Peutz - Jegher's syndrome is associated with melanocytic spots on buccal mucosa as well as in
the gastrointestinal tract. (Options C and E) Both squamous cell carcinoma and basal cell
carcinoma of the lip occur at the mucocutaneous junction of the lip. The mucus membrane of
the lip is unlikely to be the site a SCC or BCC arises. On the other hand, patients are typically
older.

6.Which one of the following breast diseases has been shown to be associated with vitamin
D deficiency?
A Fibroadenoma.
B Periductal mastitis.
C Ductal carcinoma in situ.
D Intraductal papilloma.
E Ductal ectasia.
Option C is correct Studies suggest that vitamin D deficiency has been associated with
increased risk of cancer, especially colon cancer, prostate cancer and breast cancer. Vitamin D
may play a role in controlling normal breast cell growth and may be able to stop breast cancer
cells from growing.

7 Which one the following test is a mass population screening test which has been
demonstrated to reduce cancer mortality significantly?
A Annual fecal occult blood testing in the over 50 age group.
B Annual colposcopy in sexually active women.
C Annual plasma CA125 in post menopausal women.
D Annual colonoscopy in siblings of patients with colon cancer.
E 2-yearly mammography in women aged 35-45 years.
Option A is correct There have been at least 3 randomized control trials conducted which
show that 1- to 2-yealy fecal occult blood testing (FOBT) in the population over the age of 50
years reduces the mortality from colorectal cancer by approximately 20%. (Option B)
Colposcopy is a diagnostic, not a screening test. The appropriate screening test for cancer of the
cervix in sexually active women is the Papanicolaou (Pap) smear. (Option C) Plasma CA125
levels may be raised in asymptomatic women with ovarian cancer but there is not, as yet, any
evidence to support it as a mass screening test. (Option D) Colonoscopy for siblings of patients
with colorectal cancer may reduce their mortality, but this is 'selective' screening, by targeting a
high-risk group. (Option E) While mammography in the 50-70 age group has been shown to
reduce mortality from breast cancer by up to 30%, extracted from different studies, the benefit
for women between 40-50 years is small. This benefit is even smaller in younger women.
NOTE – Regular pap smear has been shown to reduce the incidence and mortality of cervical
cancer by up to 80% and would be the correct answer if it was an option.

8. A 76-year-old lady suffers a femoral neck fracture and is brought to the emergency
department by her daughter who she lives with. She is unconscious and the “Do Not
Resuscitate (DNR)” status is unknown. Based on evaluations, she is in need for surgery on
the fracture. Which one of the following is the most appropriate action in this situation?
A Ask if there is a DNR order in place first.
B Proceed to the surgery without consent.
C Ask for consent from her daughter.
D Ask for consent from guardianship tribunal.
E Refer her for palliative care.
Option C is correct A "Do Not Resuscitate (DNR) order means if the patient dies i.e.
cardiopulmonary arrest, the doctor does not perform chest compressions, attempt electrical
cardioversion, or use acute antiarrhythmic medications. In other words DNR is defined as not
attempting any treatment once the patient is DEAD, indicated by loss of pulse, blood pressure
and breathing. DNR orders cannot be considered valid, and should not be followed unless the
patient losses all three components (pulse, blood pressure, breathing). The most common
misunderstanding about DNR is that being DNR must mean the patient is pre-terminal and is
just about to die. Another common misunderstanding is that a person who is DNR should not
get other aspects of an appropriate medical treatment and care such as biopsies, surgeries,
dialysis, etc. DNR is does not mean that the patient should let be dead just because a DNR
order is in place. DNR just means that the treating physician should consider death as the end
point of life, and attempts no further actions to reverse it. So DNR is not valid until the patient
is clinically dead evident by loss of pulse, blood pressure and the ability to breathe
spontaneously. In this case scenario the patient is unconscious, not dead; therefore, asking if
there is a DNR order in place will be inappropriate because even so it does not apply in this
situation. The patient, however, needs surgery and consent is required for that matter. Since the
patient is unconscious, her next of kin (daughter) can give consent to the surgery on her behalf.
(Option B) Proceeding to the surgery without consent could be the correct answer in
emergency cases, where no one is present to consent on the patien's behalf, and to his/her best
interes, and if failing to take prompt action would put the patient’s health at serious risks. In
such circumstances it is presumed that the carer would consent to the patient’s best interest if
he or she was present. (Option D) Referal of the case to guardianship tribunal is appropriate
only if there is conflict between family members or carers or when the family wish is in
conflict with the patient’s best interest. (Option E) Palliative care would be the correct answer
if the patient or his/her caregiver/next of kin based on sound grounds has refused active
treatment.

9 A man is admitted with acute appendicitis. He is elderly and has had a DNR order in
place after he had a previous admission for another reason last year. He needs urgent
surgery. He still wishes to be DNR. What should you do about the surgery?
A No surgery can be done on a patient who is DNR.
B Reverse the DNR order for the surgery.
C DNR is acceptable only if the surgery does not require intubation.
D Surgery is acceptable while DNR if an additional consent is signed.
E DNR does not preclude surgery; proceed with the appendectomy.
Option D is correct
DNR is not meant to be a generalized limitation on all forms of therapy. You can still intubate
patients even if they are DNR. DNR just means you are taking death as the endpoint of giving
treatment. In the event of cardiopulmonary arrest, do not give the additional therapy of
cardiopulmonary resuscitation, defibrillation, and chest compressions. DNR is not a general
equivalent for withholding any other forms of therapy. This patient needs surgery, and a DNR
does not preclude this; therefore, proceeding with the appendectomy is the most appropriate
management option, after consent is obtained from the patient for the surgery if the patient is
alert and competent to understand the treatment options, their risks and complications and the
consequences that might arise if treatment is withheld. However, if the patient insists that DNR
order be in place despite being informed of the consequences, no CPR should be attempted in
the event of cardiopulmonary arrest the surgery or in the perioperative period.

10. A 65-year-old is found to have an abdominal aortic aneurysm. The aneurysm was
repaired by a vascular surgeon, and the patient was transferred to the Intensive Care
Unit (ICU) where he stayed for 24 hours. The recovery period was uneventful. The
patient is now ready to be discharged, and keen to know whether he can drive his private
car. Which one of the following is the most appropriate advice regarding driving?
A He is unfit to drive for 6 months post-repair.
B He is unfit to drive for 6 months as he has been in intensive care unit.
C He can drive after he spends a week at home without any symptoms.
D Refer him to his general practitioner to look into the matter.
E He is unfit to drive for 4 weeks.
Option E is correct
After repair of an aortic aneurysm or cardiac valvular repair, one is unfit to drive their private
motor vehicles for at least 4 weeks. This extends to 3 months for commercial vehicles. They
can then have their unconditional driving license again. ICU admission for 24 hours is a normal
routine after some surgeries and does not pose any restriction on driving. Referral to general
practitioner is not the correct answer as the patient should be informed of the restrictions as
soon as possible. The patient may plan to visit his general practitioner in a week and without
knowing the limitations he might drive during this period.

11. A 43-year-old man presents to your clinic concerned about his risk of developing
prostate cancer. The issue arose when his father was diagnosed with prostate cancer at
the age of 88 years a few months ago. His worry was worse when he realized that his
neighbour has prostate cancer as well. The neighbor has told him that there is screening
test for early detection of prostate cancer. He asks to undergo screening. On digital rectal
exam (DRE), prostate is smooth and of normal size and shape. The medial sulcus is felt.
Which one of the following is the next best step in management? A Reassure him.
B Order a transrectal ultrasonography.
C Annual DRE.
D Perform prostate-specific antigen (PSA) testing in 2 weeks.
E Biannual DRE.
Option D is correct Studies suggest that routine screening for prostate cancer with DRE, PSA,
or abdominal ultrasound is not associated with a no or marginal reduction in mortality from
prostate cancer; therefore, it is not recommended. GPS do not require to raise the issue of
prostate cancer screening in asymptomatic men, but if they are asked about it they should fully
inform the patient about the potential risks, benefits and of uncertainties of prostate cancer
screening. If a patient chooses screening, both PSA and DRE should be performed. DRE can
cause temporary elevation of PSA and may lead to false positive results. It is recommended
that if DRE is performed first, 2 weeks be waited before PSA is measured. This interval allows
PSA to return to its baseline levels. Some authors, on the other hand, believe that the rise in
PSA is not so significant to lead to misinterpretation of the results.

12 You are a medical officer in a tertiary hospital. A patient is about to undergo


abdominal surgery. When the patient is being transferred to the operating theatre, the
nurse informs you that the consent for the surgery has not yet been obtained. Which one
of the following is the most appropriate step regarding consent?
A Send the patient to the operating room and ask the anesthesiologist to obtain the consent.
B Obtain the consent yourself and send the patient for the surgery.
C Download information from the internet and discuss it with the patient and obtain the consent.
D Call the treating surgeon and ask him to obtain the consent.
E Ask the attending nurse to obtain the consent from the patient on his way to the operating
theatre.
Option D is correct The ultimate responsibility of obtaining consent is with the clinician
directly in charge of the treatment; therefore the operating surgeon should be called for
obtaining consent from the patient. However, it is possible for that clinician to delegate
authority to other healthcare professionals, such as more junior members of staff. The proviso
for such delegation is that the person obtaining consent must be fully equipped to deal with the
consent process.

13. A 52-year-old man presents at to your clinic, seeking advice on screening for colon
cancer. Although he has no gastrointestinal symptoms, he is concerned because his
younger brother has been recently diagnosed with colon cancer at the age of 50 years.
There is no other family history of colorectal cancer. Which one of the following is the
most appropriate advice regarding current and future management?
A He should have a fecal occult blood test (FOBT) now.
B No screening procedure is required now.
C Colonoscopy should be performed now, and if normal, no further testing is required.
D Colonoscopy should be performed now and yearly thereafter.
E Colonoscopy should be performed now and 5-yearly thereafter.
Option E is correct Colorectal cancer (CRC) screening methods and intervals depend on the
individual’s risk category. Based on the risk of developing CRC, the population is classified in
4 categories: check table on page 1484 This patient is asymptomatic, but has a brother (first-
degree) relative diagnosed with CRC at the age of 50 years (<55 years). Therefore, he should
be placed in category 2. People in category 2 should be offered screening with colonoscopy at
5-years intervals starting at the age of 50 or 10 years earlier than the age the family member
was diagnosed with CRC. Ideally, this man should have started CRC screening at the age of 41.
He should be offered colonoscopy now and 5-yealy thereafter.

14.Which one of the following could the most likely finding in a patient with mid-shaft
humerus fractures?
A Numbness of the little finger.
B Inability to extend the thumb.
C Numbness of the ventral aspect of the thumb.
D Decreased grip strength.
E Decreased wrist flexion.
Option B is correct Radial nerve injury and palsy associated with fracture of the shaft of the
humerus is the most common nerve injury complicating fractures of long bones. Rarely,
median or ulnar nerves can be affected. Injury to the axillary nerve is seen in the fractures of
humeral neck. The primary motor function of the radial nerve is to innervate the muscles
responsible for wrist and metacarpophalangeal (MCP) joints extensors, and abduction and
extension of the thumb; therefore, proximal radial nerve injuries results in wrist drop or the
decreased or absent thumb extension and abduction. (Option A) The sensation of the little
finger and half of the ring fingers are supplied by terminal braches of the ulnar nerve which is
very less like to be damaged in humeral mid-shaft fracture compared to the radial nerve.
(Option C) The sensation of the ventral aspect of the thumb is provided by the terminal
branches of the median nerve. Compared to the radial nerve, the median nerve is less likely to
be affected in a mid-shaft humeral fracture. (Options D and E) The motor innervation of hand
grip and wrist flexion is by the median nerve.

15. Which one of the following is the most likely sensory disturbance associated with the
following xray?
A Paresthesia over the deltoid.
B Decreased sensation of the ulnar aspect of the forearm and hand.
C Decreased strength of hand grip.
D Decreased sensation over dorsum of the hand.
E Decreased sensation over the 2/3 radial side of the palm.
Option D is correct The X-ray shows a fracture in the mid-shaft of the left humerus. Radial
nerve injury and palsy associated with fracture of the shaft of the humerus is the most common
nerve injury complicating fractures of long bones. Rarely, median or ulnar nerves can be
affected. Injury to the axillary nerve is seen in the fractures of humeral neck. The primary
motor function of the radial nerve is to innervate the muscles responsible for wrist and
metacarpophalangeal (MCP) joints extensors, and abduction and extension of the thumb;
therefore, proximal radial nerve injuries results in wrist drop. Radial nerve also provides the
sensation of dorsum of the hand and fingers at the 2/3 radial portion (thumb, index, middle and
half of the ring fingers), the web space between the thumb and index fingers, most parts of the
posterior aspect of the forearm. Depending on the anatomical site of the injury, sensory
disturbances over these areas can be noted. Of the options, decreased sensation over dorsum of
the hand can be caused by radial nerve injury. (Option A) Paresthesia over the deltoid is a sign
of damage to the axillary. Axillary nerve damage can be an associated injury in fractures of the
humeral neck. (Option B) Decreased sensation over the ulnar aspect of the forearm and hand is
associated with ulnar nerve injuries that, compared to that of the radial nerve, are rare in
humeral shaft fractures. (Option C) The strength of grip are the action of flexor muscles of the
wrist and fingers. These muscles are innervated by median (mostly) and ulnar

nerves. (Option E) Decreased sensation over the 2/3 radial side of the palm can be caused by
injuries of the median nerve.

16. A 60-year-old man presents to your clinic with complaints of fever and a painful
swollen right thigh. He has diabetes well controlled on metformin 500mg 8-hourly and
hypertension for which he is taking valsartan 80mg. He is on atorvastatin 20 mg daily for
hypercholesterolemia as well. His recent medical history is remarkable for vein
thrombosis (DVT) of his right leg for which he is currently on warfarin. One week ago, he
was admitted to the hospital after one episode of syncope and started on amiodarone after
he was diagnosed with ventricular tachycardia (VT). Physical examination is remarkable
for a temperature of 38.3°C and a warm swollen tender right thigh that is erythematous.
Right thigh circumference is 3 cm greater than that of the left thigh. Which one of the
following could be the most likely cause of this presentation?
A Deep vein thrombosis (DVT).
B Cellulitis.
C Drug interaction.
D Hematoma.
E Rhabdomyolysis.
Option B is correct At first glance, there are a number of differential diagnoses
for a swollen painful thigh including DVT, cellulitis and hematoma. This patient
is on warfarin for treatment of a previous DVT, which is a risk factor for another
venous thromboembolic event (VTE) such as another DVT or pulmonary
embolism (PE). However, it is less likely, yet not impossible, for this patient to
develop another VTE while on treatment with warfarin. Therefore, DVT of the
right leg (option A) is a weak possibility. Interaction between warfarin and
amiodarone is well-known. Amiodarone results in decreased metabolization of
warfarin through hepatic pathways and leads to increased bleeding tendency.
Considering that the patient has been recently started on amiodarone while on
warfarin, a hematoma (option D) can also be possibility. However, the presence
of fever, warmness and erythema of the swelling makes this diagnosis less likely.
Pressure from hematoma on surrounding tissues such as skin or deeper layers (e.g.
muscles can compromise circulation and results in complications such as necrosis
or a superimposed infection that can justify, to some extent, the clinical picture;
however, the expected sequence would be hematoma formation first and then
development of signs such as tenderness, warmth and erythema later. It is also
worth reminding the fact that hematomas present as bruise or bluish not red
discoloration. Hematomas are not warm and are often non-tender. Swelling is a
feature though. Drug interaction (option C) between warfarin and amiodarone
can result in increased bleeding tendency and development of a hematoma and
interaction between atorvastatin and amiodarone can lead to an increased potential
for development of statin-related muscle problems. None of these events are
associated with the given presentation and are very unlikely to be the cause of this
clinical picture. Rhabdomyolysis (option E) is a serious condition followed by
muscle fibers breakdown and release of muscle cell contents such as myoglobin
and potassium. Myoglobinuria results in deposition of myoglobin in kidneys and
renal failure. Release of excess potassium from damaged muscle cells leads to
hyperkalemia and serious complications such as cardiac arrhythmias. Statins such
as atorvastatin and simvastatin in particular are metabolized by cytochrome P450-
3A4 (CYP3A4), and amiodarone is a potent inhibitor of this cytochrome.
Concomitant use of amiodarone and statins can result in increased activity of
statins and rises the likelihood of statin-related adverse effects such as muscular
pain, myopathy and rarely rhabdomyolysis. However, in the event of statin-
induced rhabdomyolysis, a systemic presentation is expected. Rhabdomyolysis
never causes focal signs mentioned in the scenario. Of the given optins, this
clinical picture is most consistent with cellulitis of the left thigh as the most likely
diagnosis. Cellulitis presents with systemic signs such as fever and focal findings
of induration, warmth, redness and tenderness of the affected area. Diabetes can
be a risk factor.

17. Which one of the following is the most common complication of Colles fracture?
A Malunion.
B Nonunion.
C Ischemic Volkmann contracture.
D Osteoarthritis of the wrist.
E Wrist joint stiffness.
Option A is correct Colles fracture is defined as fracture of the distal 3cm of the radius. Colles
fracture can be associated with complications including: Rupture of extensor pollicis longus
tendon Malunion and/or nonunion Stiffness of the elbow, metacarpophalangeal joints (MCPs),
interphalangeal joints (IPs) Discomfort at distal radiolunalr joint Regional pain syndrome
Volkmann contracture Post-reduction redisplacement is the most common complication of
Colles fracture. This can result in malunion as the second most common. Nonunion, wrist joint
stiffness, osteoarthritis of the wrist, regional pain syndrome, median nerve injury and inferior
radioulnar joint laxity are other less common complications associated with Colles fracture.

18. Which one of the following could be the earliest complication of the fracture shown in

the following photograph?


A Malunion.
B Nonunion.
C Ischemic Volkmann contracture.
D Wrist drop.
E Rupture of the extensor pollicis longus tendon.
Option C is correct The photograph shows fracture of the distal 3cm of the radius, referred to
as Colles fracture. Potentially, Colles fracture can be associated with complications including:
Rupture of extensor pollicis longus tendon Malunion and/or nonunion Stiffness of the elbow,
metacarpophalangeal joints (MCPs), interphalangeal joints (IPs) Discomfort at distal
radiolunalr joint Regional pain syndrome Volkmann contracture Of these, Volkmann
contracture develops earliest. Volkmann ischemic contracture is the permanent shortening of
forearm muscles, usually resulting from compartment syndrome in the forearm. It causes a
claw-like deformity of the wrist, hand and fingers. The clinical presentation of Volkmann
contracture includes five Ps: pain, pallor, pulselessness, paresthesia, and paralysis. Of these,
pain is the earliest sign. On physical examination, pain accentuated by passive stretching seems
to be the most reliable finding. Firmness of the tissues often is noted on palpation.
Pulselessness and paralysis are late findings. Induration of the forearm is another useful
diagnostic finding. NOTE - Any fracture of the humerus, elbow, forearm and wrist can result
in compartment syndrome and Volkmann contracture. Malunion, non union and rupture of the
extensor pollicis longus tendon does not develop as early as Volkmann contracture. Rupture of
extensor pollicis longus tendon often occurs 4-8 weeks after the fracture.
19. A 72-year-old woman develops pain and deformity of the left wrist after she falls on
her Out stretched left hand. The x-ray is as shown in the following photograph. Which
one of the following is the characteristics of the cast that should be used for treatment?

A Should include the wrist but not the thumb.


B Should include the wrist and thumb.
C Should include the wrist, thumb and elbow.
D Should include the wrist and elbow.
E Should include the thumb, wrist, elbow and the upper arm.
The X-ray shows fracture of the distal 3cm of the radius, namely Colles fracture. Casting after
meticulous reduction of any displacement remains the most commonly used treatment option
for Colles fracture. The wrist is flexed at 10° and deviated to the ulnar at 10°. The cast
includes the wrist and the forearm but not the thumb and the elbow. However, if there is
severe displacement and casting is considered for treatment, the cast may be extended to
include up to above the elbow is selected patients.

20. A 76-year-old lady presents to the emergency department after she sustained a fall on
her outstretched hand. The X-ray obtained is shown in the following photograph. Which
one of the following should be the position of the the wrist in cast?

A Full flexion of the wrist with radial deviation.


B Partial flexion of the wrist with radial deviation.
C Full flexion of the wrist with ulnar deviation.
D Partial extension of the wrist with radial deviation.
E Partial flexion of the wrist with ulnar deviation.
Option E is correct The X-ray shows fracture of the distal radius, also known as Colles
fracture. Falling on an outstretched (dorsiflexed) hand is the most common mechanism of this
fracture. Colles fracture is very common in elderly women, and can occur with minimal trauma
if there is osteoporosis. There can be posterior dislocation of the head of the radius, giving rise
to the 'fork deformity'. Lateral displacement may also occur. Casts are often used for treatment
of Colles fractures. Severely displaced fractures may be treated with external or even internal
fixation. For casting, the wrist should be in 10° flexion and 10° ulnar deviation, and the
forearm must be in slight pronation. Often a below-elbow cast for 4-6 week is sufficient;
however, the cast may be extend to include above the elbow if there is significant displacement.
Every significant displacement should be reduced under anesthesia before casting.

21. A 34-year-old woman trips over and falls on her outstretched right hand. Immediately
after the fall she fells severe pain in her right wrist. On examination in the emergency
department, there is tenderness over the lateral aspect of the head of the radius and the
anatomical snuffbox. Application of axial pressure to the thumb of the affected hand
causes pain. An Xray is ordered which is completely normal. Which one of the following
is the next best step in management?
A Reassurance.
B Place the wrist in thumb spica cast and repeat the X-ray in 7-10 days.
C Repeat the X-ray in 6 weeks.
D Analgesics.
E Place the wrist in a sugar tongue splint.
Option B is correct The mechanism of injury as well as the exam findings is suggestive of
scaphoid fracture. Scaphoid fracture often results from falling on an outstretched hand.
Tenderness of anatomical snuff box and pain elicited by applying axial pressure to the thumb of
the affected hand are typical clinical findings. Once scaphoid fracture is suspected, plain X-ray
films are the most appropriate next step in management. If fracture is evident on X-ray, definite
treatment with casting or surgery to fix a displaced fracture follows. Some patients, however,
will have normal X-rays, despite high index of clinical suspicion. In such cases, further
management depends on the availability of advanced imaging studies such as MRI or CT scan.
MRI is the preferred modality, as it is radiation-free and can assess both the scaphoid and the
surrounding soft tissue. CT scan can detect fractures missed on plain radiographs but it cannot
visualize possible soft tissue injuries. If advanced imaging is not available, the wrist should be
immobilized by a thumb spica cast for 7-10 days (up to 14 days) and X-ray repeated after this
period. By 7-10 days, the fracture line should be visualized on plain films. A negative control
X-ray film in the presence of symptoms warrants referral for MRI. (Option A) Scaphoid
fracture may not be evident on initial X-rays, but seen on follow-up films. A negative Xray in a
symptomatic patient does not exclude fracture; so reassurance is not appropriate. (Options C
and D) When symptoms suggest scaphoid fracture but X-rays are clear, the wrist should be
immobilized and X-ray repeated in 7-10 days. Analgesic may be prescribed for pain control,
but giving only analgesics without wrist immobilization is incorrect. (Option E) Sugar tongue
splints are insufficient for immobilization of the wrist after scaphoid fracture. A thumb spica
cast stabilizes the wrist in a neuter position with an slight ulnar deviation.

22. After a fall on an outstretched hand, a 32-year-old woman develops pain and
tenderness of her wrist. An X-ray is obtained that is shown in the following photograph.
Which one of the following is the most common complication of this fracture?

A Joint stiffness.
B Nonunion.
C Misalignment.
D Avascular necrosis.
E Pain.
Option D is correct The X-ray shows a scaphoid fracture of the left wrist. Scaphoid fracture is
common and often results from fall on an outstretched hand. Tenderness of anatomical snuff
box and pain elicited by applying axial pressure to the thumb of the affected hand are typical
clinical findings. Avascular necrosis is the most common complication of scaphoid fracture
seen in 13-40% of patients. Nonunion is the second most common complication occurring in 5-
12%. Joint stiffness and pain are other possible complications but are not as common.
Misalignment is not a complication of scaphoid fracture because the fracture is either non-
displaced or displaced. Displaced fractures are treated with open reduction and internal fixation.
During the procedure the displacement is corrected and fixed.

23.Which one of the following are the most appropriate medications to give prior to
reduction of an anterior dislocation of the shoulder?
A IV diazepam and morphine.
B IV midazolam.
C Intranasal fentanyl and IV midazolam(. Reduction can be achieved under general anaesthesia
(easier and more comfortable) or with intravenous pethidine ± diazepam.)JM
D IV midazolam and morphine.
E IM diazepam and nasal fentanyl.
Option C is correct Reduction of anterior shoulder dislocation is performed in the emergency
department after adequate muscle relaxation and sedation is provided. Intravenous midazolam
plus intranasal fentanyl (an opiate) is the most commonly used drug combination. Ketamine,
etomidate and propofol are other appropriate alternatives. Use of parenteral morphine is not
justified while intranasal fentanyl can provide the same pain relief. Fentanyl is easier to
administer. Diazepam is a long-acting benzodiazepine. While a short duration of muscle
relaxation is required for joint reduction, use of midazolam in preference of diazepam is more
appropriate. Furthermore, IM diazepam has an delayed and unpredictable effect.

24. A patient with anterior dislocation of the shoulder is unable to perform which one of
the following actions?
A Turning the door knob.
B Combing hair.
C Taking a knife from the table.
D Shaking hand.
E Wrist extension.
Option B is correct Shoulder dislocation often result in injury of the axillary nerve. Axillary
nerve runs inferiorly to the humeral head and wraps around the surgical neck of the humerus.
Axillary nerve innervates the deltoid and terese minor muscles and the skin of the lateral
shoulder (shoulder badge region) Axillary nerve dysfunction manifests as loss of sensation in a
'shoulder badge' distribution and deltoid muscle weakness. Even in the absence of such injury,
a patient with anterior shoulder dislocation cannot raise the arm; hence not able to comb hair.
Patients with anterior shoulder dislocation can use their elbow, wrists and fingers, so they can
grasp and turn the door knob, pick and hold objects with their hand, shake hand or extend the
wrist.

25. A young man presents to the emergency department after injuring his shoulder in a
fall. His Xray is shown in the following photograph. Testing the motor function by
resisted isometric contraction of which one of the following muscles would be most likely
to confirm the presence of an associated nerve injury?

A Latissimus dorsi.
B Pectoralis major.
C Trapezius.
D Serratus anterior.
E Deltoid.
Option E is correct Significant findings on this X-ray of the shoudler are a humeral head out
of the glenoid fossa and displaced inferiorly. These findings are typical for anterior dislocation
of the glenohumeral joint. Shoulder dislocations often result in injury of the axillary nerve.
Axillary nerve runs inferiorly to the humeral head and wraps around the surgical neck of the
humerus. Axillary nerve innervates the deltoid and terese minor muscles and the skin of the
lateral shoulder (shoulder badge region). Axillary nerve dysfunction manifests as loss of
sensation in a 'shoulder badge' distribution and deltoid muscle weakness. It is impractical to ask
the patient to move the affected arm due to pain; however, in the presence of axillary nerve
injury, the patient is often unable to isometrically contracts the deltoid muscle. No other muscle
mentioned in the options are innervated by the axillary nerve; hence, unaffected by axially
nerve injury.

26. 17-year-old boy falls on his outstretched right arm, with subsequent shoulder pain
and limited movement of his right shoulder, especially when attempting to elevate his arm
over the head. His appearance is illustrated in the following photograph. Which one of the

following is the most likely diagnosis?


A Fracture of the surgical neck of humerus.
B Rupture of the rotator cuff tendon.
C Dislocated glenohumeral joint.
D Rupture of the long head of biceps tendon.
E Dislocated acromioclavicular joint.
Option C is correct The flattened shoulder, as well as the way the patient is holding the
affected arm to limit shoulder for preventing pain is typical for anterior glenohumeral
dislocation. Shoulder dislocations account for 50% of all major joint dislocations. In over 90%
of cases, such dislocation occurs anteriorly. Posterior, inferior, or superior dislocations account
for the remaining 10%. An anterior shoulder dislocation is usually caused by a blow to the
abducted, externally rotated, and extended arm (e.g. blocking a basketball shot). Less
commonly, a blow to the posterior humerus or a fall on an outstretched arm may cause an
anterior dislocation. An anteriorly dislocated shoulder causes the arm to be slightly abducted
and externally rotated. The patient resists all movement. There is loss of the normal round
appearance of the shoulder resulting in a flat shoulder as seen in the photograph. (Option A)
Fracture of the surgical neck of the humerus does not cause a flat shoulder. Pain and tenderness
are almost always present. Bruising, swelling, or deformity may occur. (Option B) Rotator cuff
ruptures does not cause the shoulder to become flat. Pain and varying inability to abduct the
arm are characteristic features. (Option D) Rupture of the long head of the biceps tendon does
not cause a flattened shoulder. Resisted flexion can elicit pain in the antecubital fossa. On
forearm flexion, the bulk of the contracted biceps is lower than the unaffected arm. (Option E)
Acromioclavicular (AC) joint dislocation causes pain, tenderness and the classic step deformity
in the AC joint. Arm raise is limited due to exquisite pain when attempted.
27.An 80-year-old woman sustains a fall in the bathroom and is brought to the emergency
department by her daughter. She has severe pain and decreased range of motion in all
directions in the right hip. The right leg appears shorter than the left and is externally
rotated. A bedside portable X-ray of the pelvis is taken which is shown in the following
photograph. Which one of the following is the most appropriate management option for

this patient?
A Intramedullay nail placement.
B External fixation.
C Hip arthroplasty.
D Analgesics and bed rest.
E Traction.
Option C is correct The X-ray shows an intracapsular (Garden) fracture of the right femoral
neck. The fracture can be subcapital, transcervical, or basicervical. Only such fractures can be
considered genuine femoral neck fractures. Garden classification of intracapsular (subcapital)
fractures of the femoral neck is as follows: Garden type I: incomplete fracture with valgus
impaction Garden type II: Complete fracture without displacement Garden type III: Complete
fracture with partial displacement of the fracture fragments Garden type IV: Complete fracture
with total displacement allowing the femoral head to rotate back to an anatomic position
In reality the distinction
between classes is difficult; however, complete fracture and displacement of the femoral head
makes Garden type III (or IV) femoral neck fracture the most likely diagnosis. Garden types I
and II femoral neck fractures are surgically stabilized with closed reduction and internal
fixation. Garden types III and IV are controversial in the type of implant used for treatment. In
younger patients, closed or open reduction is recommended. In less active older patients,
however, prosthetic replacement is recommended. None of other options are superior to hip
arthroplasty.

28. The parents of a 6-week-old baby has brought her to your clinic after they noticed a
swelling on the right side of her neck. She is otherwise healthy, is being breastfed, and is
gaining weight appropriately. Which one of the following should not be considered in the
differential diagnosis of the lateral cervical swelling in this child?
A Acute cervical adenitis.
B Branchial cleft cyst.
C Cystic hygroma.
D Sternocleidomastoid muscle hematoma.
E Thyroglossal duct cyst.
Option E is correct Of the given options, thyroglossal duct cyst is invariably in the midline
and not seen on the lateral aspect of the neck. (Option A) Cervical adenitis (lymphadenitis) is
defined as enlarged, inflamed, and tender lymph node(s) of the neck. Cervical lymph nodes are
distributed along the course of the sternocleidomastoid muscle, in the posterior neck,
supraclavicular, or submandibular region. (Option B) A branchial cleft cyst is a congenital
epithelia cyst that arises on the lateral part of the neck due to failure of obliteration of the
second branchial cleft (or failure of fusion of the second and third branchial arches) during
embryonic development. A branchial cyst can also be among differential diagnoses of a lateral
neck swelling in children. (Option C) Cystic hygroma, also known as cystic lymphangioma or
macrocystic lymphatic malformation, is an congenital multiloculated lymphatic lesion that can
arise anywhere, but is classically found in the left posterior triangle of the neck (especially left
side) and axillary regions. It can cause a swelling observed in the lateral side of the neck.
(Option D) Hematomas of the sternocleidomastoid present with lateral swellings along the
course

29. A 40-year-old man with long history of alcohol consumption and smoking presents
with a 2- cm painless lump in his left tonsil. Which one of the following is the most likely
diagnosis?
A Squamous cell carcinoma (SCC).
B Nasopharyngeal cancer.
C Carotid artery aneurysm.
D Metastasis.
E Non-Hodgkin lymphoma (NHL).
Option A is correct Presence of alcohol and smoking in the history is highly suggestive of
SCC. Smoking and alcohol are the strongest contributing factors to SCCs of the head and neck.
(Option B) Nasopharyngeal carcinomas are different from SCCs of the head and neck.
Epstein-Barr virus (EBV) infection is the most common risk factor. Other risk factors include
Human Papilloma Virus (HPV) and genetic predisposition. (Option C) Carotid artery
aneurysm does not present with a lump in the tonsil. (Option D) Although metastasis can be
among differential diagnoses, given the history of alcohol and smoking, SCC is more likely as
the underlying cause of this presentation. (Option E) Although lymphoma is the second most
common cause of a tonsillar lump in adults, a single lump in tonsil is a very unusual
presentation for NHL.

30. While playing basketball, a 23-old-man landed on his left foot awkwardly and injured
his ankle. He is in the emergency department now. On examination, lateral dislocation of
the ankle is evident. Dorsal pedis pulse is absent and the foot looks pale and is cold to
touch. Which one of the following would be the next best step in management?
A Reduce the dislocation in the emergency department.
B Send the patient to the operating room for reduction under general anesthesia.
C Immobilize the joint with plaster.
D Send the patient to the operating room for open reduction and internal fixation.
E Angiography.
Option D is correct Early reduction/alignment of a dislocation/fracture is always the most
important step in management. This is even more crucial when a dislocation or fracture has
caused neurovascular compromise. In the field (prehospital), if there is evidence of
neuromuscular compromise such as a cold, discolored, and pulseless or insensate foot,
reduction of the dislocation or alignment of the fracture should be tried after the patient is given
analgesia with intravenous opiates (morphine). If morphine is not available, intravenous
benzodiazepines can be used instead. Reduction and alignment is achieved by gentle in-line
traction. Alignment should be maintained en route to the hospital. In the emergency department,
if the reduction/alignment has not been performed in the field, reduction should be attempted as
soon as possible. By definition, dislocation of the ankle is always considered unstable due to
accompanying disruption of the lateral or medial ligaments or the tibiofibular syndesmosis.
Ankle dislocation requires immediate orthopedic or podiatric consultation for surgical
intervention such as internal or external fixation of any accompanying fracture and repair of
capsular or ligamentous tears. None of the other options take precedence over early reduction
of the joint.

31. Which one of the following is the most likely presentation of a rectal cancer?
A Melena, constipation and colicky lower abdominal pain.
B Diarrhea, bloody stool and abdominal pain.
C Constipation.
D External hemorrhoids.
E Fatigue.
Option B is correct Bloody stool (rectal bleeding), bowel habit changes mostly in form of
diarrhea or changed stool caliber and abdominal pain are the most common presenting
symptoms in order of incidence in rectal cancer (B is correct). Rectal cancers present with a
variety of sign and symptoms including the following: Rectal bleeding – bleeding is the most
common presenting symptom of rectal cancer, occurring in 60% of patients. It should be noted,
however, that profuse bleeding and anemia are rare. Occult bleeding detected through a fecal
occult blood test (FOBT) is present in 26% of all cases. Changes in bowel habits – this is the
second most common presenting symptom, occurring in 43% of patients. Bowel habit changes
is often in form of diarrhea rather than constipation. Some patients report a decrease in stool
caliber. Large tumors can cause obstruction and obstructive symptoms. Tumors located low in
the rectum can cause a feeling of incomplete evacuation and tenesmus. Back pain is usually a
late sign caused by a tumor invading or compressing nerve trunks. Urinary symptoms may also
occur if the tumor is invading or compressing the bladder or prostate. Malaise is a nonspecific
symptom and present in 9% of rectal cancer cases. Bowel obstruction due to a high-grade rectal
lesion is rare, occurring in 9% of all cases. Pelvic pain is a late symptom, usually indicating
nerve trunk involvement, and is present in 5% of all cases. Other manifestations include
emergencies such as peritonitis from perforation (3%) or jaundice, which may occur with liver
metastases (< 1%). (Option A) Melena is the black stool caused by oxidation of blood in GI.
Melena is a feature in upper gastrointestinal rather than lower gastrointestinal bleeding, and not
a presenting symptom in rectal cancers. Although a change in bowel habit is the second most
presenting symptom of rectal cancers, this change is more often in form of diarrhea rather than
constipation. Lower abdominal pain due to partial or complete obstruction can be feature.
(Option C) As mentioned earlier, diarrhea is more common than constipation in rectal cancers.
(Option D) External hemorrhoids (a thrombosed anal vein) is not a common feature in patients
with rectal cancer. (Option E) Fatigue is feature more commonly seen in right-sided colon
cancers. As the blood mixes with the stool, bloody stool is a less frequently encountered
presentation. On the other hand, bleeding from right-sided lesions result in diarrhea (blood is
cathartic) and anemia. The latter is the main cause of fatigue in patients with right-sided
malignant lesions. Fatigue or malaise is seen in only 9% of patients with rectal cancer.

32. A 76-year-old man is brought to the emergency department with colicky abdominal
pain and abdominal distention for the past 24 hours. He has not passed any stool or gas
since. He gives the history of constipation for the past month. His past medical history is
significant for appendectomy at the age of 42 years. On examination, he has stable vital
signs. Abdominal exam reveals a distended abdomen, tympanic to percussion. No
tenderness, rebound tenderness or guarding is elicited. Rectum is empty with no mass.
Which one of the following can be the most likely cause to this presentation?
A Sigmoid volvulus.
B Sigmoid cancer.
C Adhesions from previous surgery.
D An obstructed hernia.
E Fecal impaction.
Option A is correct The history of prolonged constipation, as well as the exam findings of a
tympanic distended abdomen, colicky abdominal pain and an empty rectum is more consistent
with sigmoid volvulus as the most likely diagnosis (A is correct). A colonic volvulus occurs
when a part of the colon twists on its mesentery, causing colonic obstruction. Such obstruction
can be acute, subacute or chronic. Although volvulus can occur in any portion of the large
bowel, the sigmoid colon is the most frequently affected part, followed by volvulus of the right
colon and terminal ileum, namely cecal or cecocolic volvulus. In very rare cases, volvulus may
develop in the transverse colon or the splenic flexure. Patients with colonic volvulus are
commonly elderly, debilitated, and bedridden. A history of dementia or neuropsychiatric
impairment is often present. The symptoms are acute in more than 60-70% of patients while the
rest present with subacute or chronic symptoms. A history of chronic constipation is common.
The patient may describe previous episodes of abdominal pain, distention, and obstipation,
which suggest repeated subclinical episodes of volvulus. Regardless of its anatomic site,
colonic volvulus presents the same way with cramping abdominal pain, distention, constipation
and/or obstipation. Abdominal distention often increases progressively. The distention is
characteristically tympanic over the gas-filled, thin-walled colon loop. Tenderness or rebound
tenderness indicates that peritonitis has occurred or is just imminent. With progressive
obstruction, nausea and vomiting occur. The development of constant abdominal pain is an
ominous sign indicating the development of a closed-loop obstruction with significant
intraluminal pressure. This, in turn, portends the development of ischemic gangrene and bowel
wall perforation. Depending on the extent of bowel ischemia or fecal peritonitis, signs of
systemic toxicity may be apparent. Because of the massive abdominal distention, the patient
may have respiratory and cardiovascular compromise. Plain abdominal films are the initial
imaging choice. Massive dilation of the sigmoid colon loop arising from the pelvis and
extending to the diaphragm is a typical finding of sigmoid volvulus. The walls of the loop are
evident as three bright lines converging in the pelvis to create a beaklike appearance. (Option
B) A sigmoid tumor can also be the underlying cause of this clinical picture. Left-sided colon
cancer can present with bloody stool, changes in bowel habits and abdominal pain specially if
the tumor has caused partial or complete obstruction. However, a sigmoid tumor large enough
to cause complete obstruction of the colon to result in obstipation is expected to have more
pronounced exam findings. (Option C) Obstruction caused by the entrapment of a loop of the
small bowel in an adhesion band formed from previous surgery can cause small bowel
obstruction. In fact, adhesions from previous abdominal surgeries is the most common cause of
small bowel obstruction. In this patient, however, the clinical picture more favors large bowel
rather than small bowel obstruction because in small bowel obstruction nausea and vomiting is
a prominent feature that occurs earlier in the course of the process. This patient has not vomited
after 24 hours of symptoms onset. NOTE – the clinical presentation of small bowel obstruction
includes Nausea/vomiting (60-80%): The vomitus can often be bilious in nature
Constipation/absence of flatus (80-90%): Typically, a later finding of SBO Abdominal
distention (60%) Fever and tachycardia: Late findings and may be associated with strangulation
(Option D) An obstructed hernia is expected to cause constant abdominal pain and tenderness.
However, in early stages and before the strangulation occurs, these findings might be absent.
This patient has no clinical findings suggestive of a hernia as the likely cause of this
presentation. Furthermore, with obstructed hernias, presentation will favor that of the small
bowel obstruction with early onset nausea and vomiting in the course of the disease. (Option E)
As the name implies, this kind of obstruction is caused by impaction of fecal matter often
in the rectum. An empty rectum excludes this diagnosis.

33. A 72-year-old man is brought to the emergency department with abdominal pain. He
reports constipation in the past two months and absolute constipation and not even gas
pass since yesterday. On examination, the abdomen is mildly distended, tympanic to
percussion, and there is mild tenderness in the lower abdomen in particular. On digital
exam, the rectum is empty and otherwise normal. An abdominal X-ray shows sudden cut
–off the sigmoid gas at its lower end and proximal dilation of the colon. Which one of the
following is the most accurate tool to reach a diagnosis with high certainty?
A CT scan of the abdomen.
B Barium enema.
C Colonoscopy.
D Abdominal ultrasound scan.
E Barium meal and follow through.
Option A is correct This patient is very likely to have bowel obstruction, and with the 2-month
history of constipation and radiological finding of a sudden cut-off of the sigmoid and dilated
proximal colon, sigmoid volvulus is the most likely diagnosis. Once colonic volvulus is
suspected based on the history and physical findings and the patient is stable, confirmatory
imaging should follow for confirmation of the diagnosis and prompt action should follow to
reduce the risk of complications (if not already developed). The initial imaging to consider is a
plain abdominal x-ray. Abdominal x-rays reveal a distended loop of colon that may resemble a
coffee bean or a dilated U-shaped colon with a cut-off point at the site of obstruction. Plain
abdominal x-rays may also show distention of the small bowel with air-fluid levels and
decompressed colon distal to the point of volvulus. Further diagnostic modalities to consider
for more accurate diagnosis and determination of the exact anatomical site of obstruction are
contrastenhanced CT scan and water-soluble contrast enema. Currently, contrast-enhanced
abdominal CT scan is the preferred confirmatory diagnostic tool for both cecal and
sigmoid volvulus because it is noninvasive, almost readily available, accurate for both
cecal and sigmoid volvulus, and has the advantage of identification of incidental
pathology that may be missed with plain radiographs or contrast studies. In addition,
abdominal CT may facilitate the diagnosis of colonic ischemia (A is correct). (Option B)
Barium, as a contrast matter, is contraindicated if bowel perforation is suspected. Although this
patient does not have signs of bowel perforation or even ischemia, barium should be avoided.
Should enema be considered as a diagnostic tool, a water-soluble contrast is the option. NOTE
– Avoid barium if viscus perforation is suspected or if the goal is to assess the post-operative
leak. (Option C) Rigid or flexible sigmoidoscopy is the most appropriate initial treatment
option for patients in whom bowel perforation or ischemia has been excluded with high
certainty. After endoscopic detorsion, a rectal tube is left in place for 1 to 3 days to maintain
reduction. Colonoscopy is only used in unusual cases in which the obstruction point is beyond
the reach f shorter scopes. Definitive treatment is with surgery. (Option D) Ultrasound scan is
not an appropriate diagnostic tool for suspected colonic volvulus. It has no benefit over plain
radiograph for this purpose. (Option E) Barium meal and follow through is an investigation to
consider for suspected small bowel obstruction.

34. 54 years of age, has presented to your practice with complaints of intermittent
painless rectal bleeding. You arrange for colonoscopy. After a good colonic preparation,
the entire colon is visualized showing one pedunculated polyp in transverse colon, one in
descending colon and one in rectum. All the three polyps are resected endoscopically and
sent for histopathology exam. The polyp resected from the transverse colon shows high-
grade dysplasia. Which one of the following would be the most appropriate investigation
to consider for Robert?
A CT colonography.
B CT scan of the abdomen and pelvis.
C Rectal ultrasound.
D Surveillance colonoscopy in 3 years.
E Serum alpha-feto protein (AFP) and CEA tumor markers assay.
Option D is correct Although adenomatous polyps (adenomas) of the colon and rectum are
benign growths arising from the colonic and rectal mucosa, there is the risk of progression to
carcinoma in-situ and from there to invasive colorectal cancer, therefore, surveillance is
required. The nature and interval of such surveillance depends on the size, number and
histopathological examination of the polyps (adenomas). As per current National Guidelines by
the National Health and Medical Research Council (NHMRC) any colorectal polyps observed
during colonoscopy should be resected and sent for histopathologic exam, and surveillance
program considered. Current Australian guidelines recommend the following surveillance
program for colorectal polyps: Group A (low risk) – any patient with any of the following falls
into this group: One to 2 adenomas AND all <10mm AND no villous features AND no high-
grade dysplasia For group A surveillance includes colonoscopy at 5-year intervals or 1 to 2-
yearly Fecal Occult Blood Test. Group B (high risk) – any patient with any of the following
falls into this group: three to 4 adenomas OR any adenoma ≥10mm OR villous features OR
high-grade dysplasia For group B surveillance includes colonoscopy at 3-year intervals. Group
C (multiple) – this group includes patients with ≥5 adenomas. For group C, surveillance
includes colonoscopy at 1-year intervals if the number of adenomas is 5-9 and sooner if ≥10.
Group D (possible incomplete or piecemeal excision of large or sessile adenomas) Surveillance
in this group consists of colonoscopy at 3- to 6-months intervals until no residual adenoma is
detected after that time surveillance continues at 1-year intervals. Either the number of
adenomas (three) or the high-grade dysplasia histopathology of the adenoma resected from the
transverse colon puts Roberts in B group (high-risk); therefore, colonoscopic surveillance at 3-
year intervals will the most appropriate next investigation for him. (Option A) CT
colonography is a rather newly developed safe and well tolerated tool for the diagnosis of
colonic polyps and masses, and when performed by experienced hands is as accurate as
traditional colonoscopy. Cost and training issues however, will need to be addressed before it
will be widely accepted beyond the current indications of incomplete colonoscopy or patients
unfit to undergo optic colonoscopy. (Options B and C) CT scan of the abdomen and pelvis des
not play a role in surveillance of colorectal adenomas. It is used, however, in case of colorectal
cancer to assess the extent of the disease and staging. Rectal ultrasound is not an assessment
tool for surveillance of colorectal adenomas. It is neither sensitive nor specific. (Option E)
Serum tumor markers such as AFP and CEA are used if the diagnosis is invasive carcinoma.
Such markers are uses for assessment of the tumor burden as well as monitoring for any
recurrence post-treatment.

35. A 35-year-old woman presents to the emergency department with sudden onset
shortness of breath and right-sided pleuretic chest pain. On examination, she has a blood
pressure of 130/89mmHg, pulse rate of 102bpm, and respiratory rate of 25 breaths per
minute. Neck forehead veins are flat and the trachea is slightly deviated to the right side.
Her right-lung field has reduced breath sounds and is hyper-resonance to percussion. A
chest X-ray shows a 25% right-sided pneumothorax. There are no other abnormalities on
chest X-ray. Which one of the following is the most appropriate next step in management?
A Chest tube drainage.
B Needle aspiration.
C Intubation and mechanical ventilation.
D Observation for 24 hours.
E Thoracotomy.
Option B is correct This woman has developed pneumothorax in the absence of a history of
trauma or an invasive procedure. Such pneumothoraces are termed 'spontaneous pneumothorax'.
She has no chest X-ray abnormalities apart from those related to the pneumothorax. She also
has no history of lung disease, making ‘primary spontaneous pneumothorax (PSP)’ the most
likely diagnosis. The management option of choice for patients with symptomatic PSPs or
asymptomatic PSPs that are large (>15%) is needle aspiration. (Option A) Chest tubs insertion
is the management of choice in patients with traumatic pneumothorax, patients with a large
(>15%) secondary spontaneous pneumothorax, or when a patient with pneumothorax is about
to undergo mechanical ventilation or being transferred by air (see topic review). (Option C)
Mechanical ventilation or positive pressure ventilation of any kind makes pneumothorax worse.
(Option D) Observation is an acceptable management option for patients with primary
spontaneous pneumothoraces that are asymptomatic and small (≤15%). (Option E)
Thoracotomy has no role in management of pneumothorax but may be indicated for the
underlying airway disease as the cause of the pneumothorax. TOPIC REVIEW Pneumothorax
is defined as presence of air in the pleural space. Based on the etiology, pneumothorax can be
defined as traumatic or spontaneous. Traumatic pneumothorax is caused by blunt or penetrating
chest traumas or invasive procedures (iatrogenic) such as central vein catheterization, pleural
biopsy, etc. Spontaneous pneumothorax is the term used when the condition occurs in the
absence of an apparent trauma or procedure. Spontaneous pneumothorax is classified as
primary or secondary based on absence or presence of an underlying lung pathology or chest
X-ray abnormalities. Primary spontaneous pneumothorax (PSP) – spontaneous pneumothorax
is primary if all of following criteria are met: There is no respiratory finding on exam except
those related to the pneumothorax There is no history of lung disease There is no chest X-ray
finding other than those related to the pneumothorax The patient is young (≤50 years) There is
no significant history of smoking Secondary spontaneous pneumothorax (SSP) – spontaneous
pneumothorax is considered secondary if either of the following is present: Respiratory
findings other than those related to pneumothorax History of lung disease such as COPD (the
most common cause of SSP), asthma, cystic fibrosis sarcoidosis, TB, lung cancer, chronic
bronchitis, bronchiectasis, extra- or intrathoracic restrictive lung disease, etc Any chest X-ray
abnormality other than those related to pneumothorax Age >50 years Significant history of
smoking NOTE – the typical patient with PSP is a thin tall man of 20- to 40-years old. PSP is
rarely seen after 40 year and almost never after the age of 50; therefore, spontaneous
pneumothorax in those older than 50 is considered secondary and treated accordingly until
proven otherwise. Size of pneumothorax - Accurate estimation of the size of a pneumothorax is
difficult. There different methods for estimation: Average interpleural distance (AID) method -
approximates the size of a pneumothorax from a PA CXR in standing position by taking the
sum of the distances (measured in millimeters) between the ribs and the visceral pleura at the
apical level (A), midthoracic (B), and basal level (C), then dividing the sum by 3. Light Index –
An upright PA chest X-ray is obtained. The width of the lung and the hemithorax are measured
(in centimeters). The percentage of pneumothorax is calculated from the following formula:
Pneumothorax percentage = (1 – (width of the lung) /(width of the hemithorax) )x100 For
example if the width of the lung and the hemithorax are 5 cm and 10 cm respectively, the
percentage of pneumothorax will be: 1 – (5) / (10) = 1 – 0.125 = 0.875 or 87.5 % (87.5% of the
affected hemithorax is occupied with the pneumothorax) These methods are difficult to apply
in practice and often underestimate the size of the pneumothorax. As a result, some clinicians
tend to describe a pneumothorax as large or small, rather than using the percentage. Chest wall
– pleural line distance at the hilum level - British Thoracic Society guidelines define a
pneumothorax as small if the distance from chest wall to the visceral pleural line (at the level of
the hilum) is less than 2 cm or large if the distance from the chest wall to the visceral pleural
line is 2 cm or greater. Some clinicians prefer 3 cm laterally and 4 cm apically as the threshold
to distinguish small and large pneumothoraces. Symptoms: Small pneumothoraces are often
asymptomatic. Larger pneumothoraces can present with: Pleuretic chest pain (pain may be
referred to shoulder tip) Shortness of breath 3 3 3 3 Clinical findings: Clinical findings
associated with pneumothoraces include: Decreased breath sounds over the affected area due to
decreased air entry Hyperresonance over the affected area Decreased tactile fremitus Tracheal
deviation to the affected side Management: Primary spontaneous hemothorax (PSP) –
management of PSP depends on the presence of the symptoms and/or the size of the
pneumothorax: Consider discharging the patient and review in 24 hours, and every 1-2 weeks
until the pneumothorax is resolved if: The patient is asymptomatic (or minimal symptoms)
AND The size of the pneumothorax is less than 15% of the affected lung, or the distance
between the chest wall and the visceral plural line is ≤2cm Consider needle aspiration and
REVIEW if: The patient is symptomatic (pleuretic chest pain or dyspnoea), OR The size of the
pneumothorax is ≥15%of the affected lung, or the distance between the chest wall and the
visceral plural line >2cm After needle aspiration, admission and catheter drainage (chest tube)
would be indicated as the next best step in management if: The aspirated air is ≥3 litres The
distance between the chest wall and visceral pleural line is still >2cm on a chest x-ray taken 4
hours after needle aspiration There is significant shortness of breath NOTE – the rationale
behind less invasive initial management of PSP is based upon the fact that the patients with
PSP are young and otherwise healthy, and there is no underlying pathology to perpetuate the
pneumothorax or prevent it from spontaneous healing Seconadary spontaneous pneumothorax
(SSP) – in SSP, the underlying lung problem prevents the pneumothorax from spontaneous
healing. All patients with SPS need to be admitted to the hospital and undergo: Needle
aspiration and REVIEW if there are no significant symptoms AND the size of the
pneumothorax is less than 15% of the affected lung (or the distance between the chest wall and
the visceral plural line is ≤2cm) Catheter drainage (chest tube) if the patient is symptomatic
(pleuretic chest pain or dyspnoea) OR the size of the pneumothorax is ≥15%of the affected
lung (or the distance between the chest wall and the visceral plural line >2cm) Traumatic
pneumothorax Unless the patient is asymptomatic and the size of pneumothorax is <15% (or
the distance between the chest wall and the visceral plural line is ≤2cm), he patient should
undergo catheter drainage (chest tubes). Asymptomatic patients whose pneumothorax is 15%
(or the distance between the chest wall and the visceral plural line is ≤2cm) can be closely
observed for spontaneous resolution of the pneumothorax. NOTE – Pneumothorax of any kind
and size should be treated with chest catheter (chest tube) insertions if: The patient is
undergoing general anesthesia for any reason The patient is planned to be intubated and
mechanically ventilated The patient is planned to be transported by air (air transport)

36. 48 Sarah, 48 years of age, underwent cholecystectomy 6 weeks ago for frequent
episodes of biliary colic caused by gallstones. An intra-operative cholangiogram was clear
with no left stones evident at that time. Today,0 she has presented because after being
symptom-free for one week after the operation, she has started to have bloating and
belching again. Which one of the following best explains her symptoms? A Pancreatitis.
B Cystic duct stump syndrome.
C Stone left in the common bile duct.
D Symptoms are not related to gallstones.
E Post-cholecystectomy syndrome.
Option E is correct The dyspepsia manifested by bloating and belching after cholecystectomy
is suggestive of postcholecystectomy syndrome (PCS) as the most likely diagnosis. PCS
affects between 10-15% of patients with cholecystectomy and is characterized by a
heterogenous group of symptoms including: Upper abdominal pain Nausea and vomiting
Diarrhea Jaundice Bloating Excessive gas Dyspepsia. These symptoms can be the continuation
of symptoms thought to be caused by gallbladder pathology, the development of new
symptoms normally attributed to the gallbladder, or symptoms caused by removal of the
gallbladder. In 90% of the time an etiology can be found. The most common etiologies are:
Choledocholithiasis – stones remained or formed in the common bile duct or cystic duct
remnant Biliary dyskinesia Continuously increased bile flow to the GI tract Dilation of cystic
duct remnant NOTE – choledocholithiasis is the most common cause of PCS. It can involve
the common bile duct of the cystic duct remnant. Choledocholithiasis is classified as retained,
if found within 2 years of cholecystectomy or recurrent, if the stone id found 2 years after the
surgery. Recurrent stones formed as result of the biliary stasis are often caused by strictures,
papillary stenosis, and biliary dyskinesia. NOTE – ultrasonography is the initial imaging study
of choice for patients with suspected PCS. PCS is a provisional diagnosis and the presentation
should be renamed after a specific cause as an explanation for the symptoms is clinched.
(Option A) The presentation is quite different from acute pancreatitis due to chronicity of
symptoms and lack of the typical presentation. Chronic pancreatitis can cause dyspepsia but
other features suggestive of the diagnosis are absent. Chronic pancreatitis presents with
abdominal pain that can radiate to the back, chronic diarrhea due fat malabsorption and other
signs and symptoms caused by pancreatic insufficiency. (Option B) Cystic duct stump
syndrome or cystic duct remnant syndrome refers to gastrointestinal symptoms of bloating,
dyspepsia, nausea, etc caused by dilation of the remnant cystic duct. It is among the etiologies
for PCS. The patient is likely to have PCS either due to this condition or other causes. Unless
further studies establish the exact etiology, it cannot be said that cystic duct stump syndrome is
the most likely cause to this presentation. (Option C) Retained stones can also be an etiology
for PCS and the presentation; however an intraoperative cholangiogram has been clear, making
this condition less likely, yet not impossible. (Option D) A clear cholangiogram makes this
option correct but again this is not an etiology.

37. Which one of the following is the most common finding on ultrasound examination in
patient with right upper quadrant pain?
A Gallbladder wall thickness and extravasation.
B Gallstones.
C Pericholecystic edema.
D Bile duct dilation.
E Normal findings.
Option B is correct Most RUQ pains in adults are usually due to biliary or hepatic diseases.
Biliary causes of the RUQ pain include: Gallstones Acute cholecystitis Acute cholangitis
Dysfunction of the Sphincter of Oddi Hepatic etiologies include: Hepatitis Perihepatitis Liver
abscess Budd-Chiari syndrome Portal vein thrombosis. Gallstones are very common among
different populations. They could be asymptomatic and incidentally found during evaluation
for another reason, or cause problems such as biliary colic, cholecystitis, choledocholithiasis,
and cholangitis. Of the options, gallstones are the most common ultrasonographic findings in
patients with RUQ pain. Thickened gallbladder wall and extravasation is the diagnostic finding
seen in patients with RUQ with cholecystitis. Gallstones, either as the cause, or as an incidental
finding may be noted. Pericholecystic edema can be noticed as well. Bile duct dilation can be
seen in patients with RUQ caused by bile duct obstruction.

38. A 55-year-old man presents to your clinic with complaint of recurrent abdominal pain
radiating to back. Each episode of pain lasts for 3 to 4 hours. No specific trigger is
mentioned and the pain does not seem to be related to eating or hunger. An ECG is
obtained which is normal. Abdominal ultrasound exam is inconclusive. Laboratory tests,
including serum amylase, lipase and troponin, are all within normal parameters. Which
one of the following is the most likely diagnosis?
A Duodenal ulcer.
B Esophagitis.
C Acute coronary syndrome.
D Pancreatitis.
E Cholecystitis.
Option D is correct Epigastric pain can have a myriad of etiologies. Cardiac ischemia, peptic
ulcer disease, hepatobiliary diseases, pancreatitis, and lower lobe pneumonia can all cause
epigastric pain. Of the options, however, chronic pancreatitis is the most likely diagnosis..
Chronic pancreatitis presents with epigastric pain that typically radiate to back. The pain may
or may not be triggered by eating and lasts for several hours. Serum amylase and lipase are
often normal, or just slightly elevated. Characteristic finding on imaging is calcification of the
pancreas. Ultrasound shows calcifications in only 60 to 70% of patients. 30 to 40% of patients
may have normal ultrasound scan results. (Option A) Duodenal ulcer presents with epigastric
pain. It can radiate to back. Laboratory findings and ultrasound scan are inconclusive; however,
it is worse with hunger and relieved by eating. (Option B) Esophagitis presents with dysphagia
and/or odynophagia (aggravated by swallowing). Laboratory tests and ultrasound exam
findings are unremarkable, but it is uncommon for the pain to radiate to back. Retrosternal
chest pain and heart burn are other features. (Option C) A normal ECG makes acute coronary
syndrome (ACS) less likely. On the other hand, chronicity of the pain and the episodic nature
excludes ACS with high certainty. (Option E) Cholecystitis can presents with pain often
aggravated byr eating, especially fatty foods. The pain can be felt in the right upper quadrant or
epigastrium. The pain can radiate to the tip of right shoulder. Furthermore, fever is a common
feature. Alkaline phosphates and gamma glutamyl transpeptidase (GGT) are often elevated.
Lipase and amylase are normal or only slightly elevated. It is unlikely that sonography misses
the diagnosis.

39. A 65-year-old female patient undergoes percutaneous coronary intervention and stent
placement through femoral artery, and is started on aspirin andclopidogrel. After 24
hours, she develops a pulsatile painful mass in the groin though which the catheter was
sent in. Which one of the following is the definitive treatment of this mass?
A Massage and application of compression.
B Surgical repair.
C Vitamin K.
D Angiography.
E Injection of thrombin into the mass.
Option E is correct The scenario is a typical description of a pseudoaneurysm as a
complication of femoral artery catheterization. A pseudo aneurysm is a hematoma that forms as
the result of a leaking hole in an artery. The hematoma forms outside the arterial wall and is
contained by the surrounding fibromuscular tissue. The hematoma must continue to
communicate with the artery to be considered a pseudoaneurysm. Pseudoaneurysm occurs in up
to 7.5% of femoral artery catheterizations and can cause distal embolization, extrinsic
compression on the neurovascular structures, rupture, and hemorrhage. A pseudoaneurysm
presents with a painful pulsatile groin mass. A bruit over the mass may or may not be heard.
Duplex Doppler ultrasound may provide evidence of extra-arterial flow or there may be classic
‘toand- fro’ Doppler waveform in the neck of the pseudoaneurysm. Ultrasound-guided
thrombin injection for the pseudoaneurysms of the iliac, femoral and peroneal arteries is safe,
effective, and associated with few complications. It has emerged as the preferred treatment
modality for pseudoaneurysms occurring as a result of percutaneous femoral arterial
interventions (success rate: 97%). The procedure should be performed by a physician and an
ultrasonographer to enable continuous visualization of the pseudoaneurysm. Thrombin is
injected into the sac of the pseudoaneurysm away from the neck under direct ultrasound
guidance. (Option A) The ultrasound-guided compression is successful in 90% of case and was
the treatment of choice previously; however, this approach is not favorable anymore because it
needs prolonged compression time (up to 120 minutes), makes the patient uncomfortable, is
associated with early recurrence and has limited success in treating large pseudoaneurysms.
(Option B) Urgent surgical exploration is indicated for a threatened limb and when a
percutaneous approach is not feasible. This method was the treatment of choice before 1985.
(Option C) Vitamin K is not a treatment option. Any options suggesting cessation of
clopidogrel is incorrect as this is associated with high mortality rate in a patient who has just
undergone angioplasty. Moreover, it does not treat the aneurysm. (Option D) Angiographic
intervention via a retrograde approach from the contralateral common femoral artery is only
indicated if acute vessel occlusion due to distal emboli occurs. The event presents with the
patient complaining of pain, pallor, parenthesis or decreased movement in the respective limb.
Clinical examination may reveal a cold ischemic limb with absent pulses, an ankle-brachial
index (ABI) <0.5, or absent color flow and Doppler waveform on duplex ultrasound in the
index artery.

40. Which one of the following is the most appropriate treatment for venous ulcers of
lower limb?
A Bed rest with limb elevation.
B Antibiotics.
C Tissue debridment.
D Compression stocking and a walking program.
E Aspirin and statins.
Option D is correct Venous hypertension is associated with morphologic changes in the
capillary and lymphatic microcirculation resulting in important physiologic changes such as
capillary leak, fibrin deposition, erythrocyte and leukocyte sequestration, thrombocytosis and
inflammation. Collectively, these processes impair oxygenation of the skin and subcutaneous
tissue. Sever venous hypertension and hypoxia leads to oedema, subcutaneous fibrosis,
hyperpigmentation and ulcer formation. Continuous compression therapy that is typically
achieved with gradual compression stockings or compression bandage is a very important and
essential part of treatment of chronic venous disease with or without ulceration. The
mechanism by which compression therapy produces beneficial effects is not yet understood;
however, the following have been suggested: improvement in venous haemodynamics by
reducing venous reflux increasing deep venous flow velocity improvement in lymphatic flow
and cutaneous microcirculation decreasing ambulatory venous pressure alteration in certain
pro-inflammatory matrix metalloproteinases that are present in chronic ulcers that result in
favourable changes promoting ulcer healing Additionally, compression that is intermittent
enhances fibrinolysis, a potentially important mechanism in reducing fibrosis and enhancing
ulcer healing Compression therapy in contraindicated in patients with moderate to severe
peripheral artery disease (PAD), and should be used with extreme caution in patients with mild
to moderate PAD, evidenced by an ankle-brachial index (ABI) of <0.9 but >0.6. In the
presence of arterial disease can cause skin necrosis that in few instances may lead to
amputation. Active cellulitis is another contraindication for compression therapy.

41. A 57-year-old man presents with constipation and abdominal distention of 3-day
duration. On examination, a lump is noted in the left inguinal region. The lump has no
cough impulse, is not tender or painful. Which one of the following is the best
investigation to consider in this patient?
A Ultrasound of the lump.
B Ultrasound of the abdomen.
C Erect and supine x-ray films of the abdomen.
D CT scan of the lump.
E Biopsy.
Option C is correct The lump can be a segment of a small bowel loop trapped in the inguinal
region, namely inguinal hernia. Absent cough impulse is a pointer towards incarceration of the
hernia. There are two points in the question. The first and most important one is the presence
constipation and abdominal distention consistent with bowel obstruction. The second point is
the presence of an inguinal lump with no cough impulse, no pain and no tenderness. This lump
is very likely to be an incarcerated inguinal hernia which justifies both the lump and the
underlying cause of the obstruction. However, other differentials such as an inguinal lymph
node should be considered as well. The history and physical findings makes small bowel
obstruction (SBO) the first condition to exclude. Initial diagnostic approach to suspected SBO
is plain X-rays films. At least two views are required: upright view and supine view. Although
plain abdominal films can confirm the diagnosis in most patients without need for further
diagnostic testing, they can be equivocal in 20-30% of patients, or normal or misleading in 10-
20%. Abdominal X-rays are always the first step in diagnosis but not the best. Currently the
best and most accurate diagnostic test for SBO is abdominal CT scan (not an option) with both
intravenous and oral contrast. CT scan can simultaneously provide information about the
presence, level, severity and cause of obstruction. In addition, other abdominal pathologies can
be detected. In some cases, closed-loop or strangulating obstruction may be revealed. CT
scanning has been replaced with barium follow-through, which previously was the gold-
standard diagnostic test for SBO. Follow-through series are now reserved for cases in which
CT is not diagnostic. Abdominal ultrasound scan is another appropriate diagnostic choice. It
can be more accurate than plain films but clearly inferior to abdominal CT scan. There are
technical difficulties and limitations that make it only appropriate for selected patients. In this
question, however, the best available option is erect and supine plain films of the abdomen.

42. A 38-year-old man underwent gastric bypass surgery due to obesity (BMI>42) a few
months earlier. Now, he has presented with complaints of lightheadedness, palpitation
and diarrhea within the first hour of meals, especially after he takes milk, toast and cereal.
Which one of the following would be the most appropriate management option?
A Addition of vitamin
B12. B Endoscopy.
C Diet counselling.
D Esomeprazole.
E Ranitidine.
Option C is correct The presentation and the history of gastric bypass surgery makes dumping
syndrome the most likely diagnosis. Dumping syndrome is the effect of altered gastric reservoir
function, abnormal gastric motor function, and/or pyloric emptying mechanism. In term of
temporal relation to eating, dumping syndrome can have early or late manifestations: Early
dumping Symptoms of early dumping syndrome occur 30-60 minutes after a meal, and are
hypothesized to result from accelerated gastric emptying of hyperosmolar contents into the
small bowel. The hyperosmolarity of the bowel content leads to fluid shift from the
intravascular compartment into the bowel lumen, resulting in rapid small bowel distention and
increase in the frequency of bowel contractions. With rapid shift of fluid into the intestinal
lumen, circulating volume contraction follows, triggering a vasomotor response that presents
with tachycardia, lightheadedness, etc Late dumping Late dumping occurs 1-3 hours after a
meal, probably through development of hyperinsulinemic (reactive) hypoglycemia. Rapid
delivery of a meal to the small intestine results in an initial high concentration of carbohydrates
in the proximal small bowel and rapid absorption of glucose, hypoglycemia and a
hyperinsulinemic response. The high insulin levels stay for longer period and are responsible
for the subsequent hypoglycemia. The following diagram summarizes the mechanism through
which the dumping syndrome develops:

The clinical
presentation of dumping syndrome can be categorized as abdominal versus vasomotor
symptoms, and based on the time of onset as early versus late. Early dumping systemic
symptoms: Desire to lie down Palpitations Fatigue Faintness Syncope Diaphoresis Headache
Flushing Early dumping abdominal symptoms: Epigastric fullness Diarrhea Nausea Abdominal
cramps Borborygmi These symptoms can be collectively referred to as dyspepsia. Late
dumping symptoms: Perspiration Shakiness Difficulty to concentrate Decreased consciousness
Hunger For patients with dumping syndrome, dietary modification is first-line management
option. The following pieces of advice are appropriate: Daily energy intake is divided into 6
meals. Fluid intake during and with meals be restricted and liquids be avoided for at least half
an hour after a meal. Avoiding simple sugars. Milk and milk products are generally not
tolerated and should be avoided. Because carbohydrate intake is restricted, protein and fat
intake should be increased to fulfill energy needs. Additional points to consider:
Supplementation with dietary fiber has proven effective in the treatment of hypoglycemic
episodes. Many medical therapies have been tested, including pectin, guar gum, and
glucomannan. These dietary fibers form gels with carbohydrates, resulting in delayed glucose
absorption and prolongation of bowel transit time. Dietary change to a low-carbohydrate, high
protein diet, as well as the use of alpha-glucosidase inhibitors, may be useful to control the
symptoms of dumping. This is preferential to subtotal or total pancreatectomy in those persons
with severe symptoms. Most patients have relatively mild symptoms and respond well to
dietary manipulations. In some patients with postprandial hypotension, lying supine for 30
minutes after meals may delay gastric emptying and increase venous return, thereby
minimizing the chances of syncope.
43. A 67-year-old man presents to your GP clinic with complaint of fresh rectal bleeding
covering the outer surface of his stool for the past 3 weeks. He is otherwise in good health.
Which one of the following is the most appropriate next step in evaluation?
A Digital rectal exam (DRE).
B Proctoscopy.
C Flexible sigmoidoscopy.
D Rigid sigmoidoscopy.
E Colonoscopy.
Option A is correct Rectal bleeding is often caused by benign conditions such as
haemorrhoids or anal fissures. However, it is possible that this assumption masks more
proximal serious causes such as colorectal cancer. To avoid missing patients with sinister
conditions either as the cause of bleeding or a concomitant problem, all such patients should be
assessed even if based on the history the aetiology is clear. Approach to patients with rectal
bleeding starts with a thorough history followed by physical exam. The physical examination
should be directed at identifying possible or definite sources of bleeding and at finding
worrisome distal lesions that may be detectable on examination. A detailed physical
examination must include external inspection of the anus and a DRE. Having the patient
bear down during the examination may induce prolapse of a haemorrhoid or bleeding of a
superficial lesion. Moreover, when available, office-based anoscopy or proctoscopy should be
Performed in patients who present with acute rectal bleeding. These are simple measures not
requiring bowel preparation. Sigmoidoscopy or colonoscopy is indicated in patients in whom
initial assessment suggests more proximal lesions as the cause.

44. A 56-year-old man presents to your practice with complaint of fresh rectal bleeding
covering his stool. He has long-standing history of constipation. He mentions no pain on
defecation. Which one of the following can be the most likely cause?
A Colorectal cancer.
B External hemorrhoid.
C Internal hemorrhoid.
D Anorectal fistula.
Option C is correct Fresh blood covering the stool in the background history of prolonged
constipation is most likely caused by internal hemorrhoid. Typical pattern of rectal bleeding in
internal hemorrhoids is usually associated with a bowel movement. Bright-red blood typically
coats the stool at the end of defecation. Blood may also drip into the toilet bowl or stain toilet
paper. Hemorrhoidal bleeding is almost always painless, and painful defecation prompts
investigation for other causes such as anorectal fissures. Although the pattern of rectal bleeding
in this man is more likely to have been caused by hemorrhoids, overt or occult recta, bleeding
should not be merely attributed to hemorrhoids until other other less common but more serious
causes of rectal bleeding have been excluded by endoscopic testing. Flexible sigmoidoscopy or
anoscopy in low-risk younger patients or colonoscopy in most other patients establishes the
diagnosis of hemorrhoids, and excludes other causes of bleeding such as colitis, tumors, polyps,
or vascular malformations. (Option A) Presence of rectal bleeding in this patient should also
raise suspicion against colorectal cancer. However, this diagnosis is less likely because firstly
there are no other suggestive findings such as weight loss, altering bowel habits, etc and
secondly, hemorrhoids are more common than cancers statistically. (Options B and E)
External hemorrhoid and anal fissure are associated with significant anal pain and are very
unlikely to be the diagnosis in this patient. (Option D) Anorectal fistulae are associated with
soiling rather than rectal bleeding. It is unlikely that painless fresh rectal bleeding is the only
presentation of anorectal fistulae.

45. A 45-year-old refugee Somali man presents with rectal bleeding. He describes the
bleeding as bright blood covering the stool. Which one of the following should be the next
investigation?
A Rectal examination.
B Fecal occult blood test.
C CT scan of the abdomen and pelvis.
D MRI.
E Colonoscopy.
Option A is correct In approach to every patient with rectal bleeding, a complete history
should be taken and physical examination performed. Inspection of the anus and a digital
rectal exam must be carried out for every patient presenting with rectal bleeding. If
available, anoscopy or proctoscopy is a good means of inspecting the anorectal area,
especially during active bleeding. Although often benign in nature, rectal or occult bleeding
should always be attributed to more serious causes until proven otherwise by endoscopic
testing. Flexible sigmoidoscopy in low-risk younger patients or colonoscopy in most other
patients establishes the diagnosis of hemorrhoids, and excludes other causes of bleeding such
as colitis, tumours, polyps, or vascular malformations. (Option B) Occult blood testing, in the
presence if overt bleeding, is not indicated. (Options C and D) Imaging modalities such as CT
scan or MRI are not routinely used for evaluation of rectal bleeding. (Option E) Colonoscopy
is considered if the source of bleeding is suspected to be more proximal. With colonoscopy the
entire colon can often be visualized.

47. A 65-year-old woman presents to the emergency department with a painful discrete
swelling below and lateral to the pubic tubercle. On examination, the swelling is non-
reducible and mildly tender to touch. Coughing or straining does not make any changes
in the swelling. Which one of the following is the most appropriate next step in
management of this woman?
A Surgical exploration.
B Ultrasonography of the swelling.
C NPO, nasogastric tube and bowel rest.
D Fine needle aspiration (FNA) of the swelling.
E Excisional biopsy of the swelling.
Option A is correct The clinical picture is suggestive of a femoral hernia as the most likely
diagnosis. Irreducibility and the fact the swelling is not altered by cough or straining suggests
that the hernia has been incarcerated. Tenderness to palpation, if present, indicates
strangulation. Femoral hernias account for 2-4% of groin hernias, are more common in women,
and are more likely to present with strangulation requiring emergency surgery. An incarcerated
hernia needs watchful monitoring for signs of strangulation while semi-elective surgery is
arranged. Where clinical picture suggests strangulation, emergency surgical intervention should
be performed without delay. For most hernias (as in this case), the location will be obvious on
physical examination. Femoral hernias most commonly present inferior to the inguinal
ligament and medial to the femoral artery. They present with a swelling below and lateral to the
pubic tubercle, in contrast to inguinal hernias that present as a swelling medial to the pubic
tubercle. Most of the time, the diagnosis is evident by physical exam; however, in situations
when the physical exam is uncertain, such as in obese patients, additional imaging studies are
recommended. Ultrasonography of the groin region is particularly useful if confusion exists as
to the location of the hernia. This is particularly important owing to the fact that femoral
hernias are approached differently from inguinal hernias. While watchful waiting may be an
option for asymptomatic or minimally symptomatic inguinal hernia, this is not recommended
for femoral hernias due to the high risk of complications. (Option A) Ultrasonography is used
when the diagnosis is uncertain clinically. CT scan is used if ultrasound results are equivocal
and non-diagnostic. MRI has the highest accuracy but is not readily available and costeffective.
(Option C) Femora hernias have a high rate of incarceration and strangulation and it is unlikely
that an incarcerated femoral hernia resolves with bowel rest. (Options D and E) The diagnosis
is certain in this case scenario. Fine needle or excisioanl biopsies are not necessary when hernia
is suspected.

47. A 40-year-old woman presents to the emergency department with sudden-onset chest
pain, dyspnea and a fever of 39°C. She has arrived from Canada to Sydney 2 days ago by
air. Which one of the following would be the most appropriate next step in management?
A Chest X-ray (CXR).
B ABG.
C Computed tomography pulmonary angiography (CTPA).
D V/Q scan.
E D-dimer assay.
Option A is correct Travel, especially by air, longer than 3 hours is a risk factor for
development of thromboembolism either in form of deep vein thrombosis (DVT) or pulmonary
embolism (PE). Sudden onset dyspnea is the most common presenting symptom in PE.
Pleuretic chest pain, cough, hemoptysis, hypoxia and even collapse and hemodynamic
instability can be present as well. Fever isa less common, yet a possible presenting sign.
However, in the presence of fever, more common causes should be excluded before PE is
considered as a likely diagnosis. A CXR is always required as one of the very initial measure to
investigate other likely causes other than PE. This is even more important in the presence of
fever that makes an infectious etiology more likely, or at least as likely as, PE. (Option B)
ABG is performed in initial assessment. Typical finding for PE is concomitant presence of
hypoxia and hypocapnia; however, this pattern is seen in only a minority of patients. Hypoxia
is frequently present but may be due to a wide variety of other cardiopulmonary complications.
(Options C and D) CTPA and V/Q scan are used for definite diagnosis of PE somewhere
down the line but not as the next best step here. (Option E) D-dimer assays are used when PE
is considered as a likely diagnosis in patients with low to moderated pretest probability for PE,
after other potential causes are excluded.

48. A 48-year-old man presents to your practice several months after gastrectomy with
complaint of dyspepsia. This problem occurs mostly 30 minutes after meals. Dyspepsia is
worse when he takes toast and cereal for his breakfast. Which one of the following will be
the most important advice to give?
A He should use more pillows when sleeping.
B Revision of gastrectomy.
C Ranitidine.
D Esomeprazole.
E Advice dietary changes.
Option E is correct With gasterectomy in history, dumping syndrome is the most likely
underlying cause to this presentation. Dumping syndrome is the effect of altered gastric
reservoir function, abnormal gastric motor function, and/or pyloric emptying mechanism. In
term of temporal relation to eating, dumping syndrome can have early or late manifestations:
Early dumping: Symptoms of early dumping syndrome occur 30-60 minutes after a meal and
are believed to result from accelerated gastric emptying of hyperosmolar contents into the small
bowel. This leads to fluid shift from the intravascular compartment into the bowel lumen,
resulting in rapid small bowel distention and increase in the frequency of bowel contractions.
Even in healthy persons without gastric surgery, rapid instillation of liquid meals into the small
bowel has shown to induce dumping syndrome. Rapid shift of fluid into the intestinal lumen
results in decreased circulating volume, triggering a vasomotor response presenting with
tachycardia and lightheadedness. Late dumping: Late dumping occurs 1-3 hours after a meal.
The pathogenesis is thought to be related to the early development of hyperinsulinemic
(reactive) hypoglycemia. Rapid delivery of a meal to the small intestine results in an initial
high concentration of carbohydrates in the proximal small bowel and rapid absorption of
glucose. This is replied by a hyperinsulinemic response. The high insulin levels stay for longer
period and are responsible for the subsequent hypoglycemia. The following diagram
summarises the mechanism through which the dumping syndrome develops: (Next Page)
The clinical presentation of dumping syndrome can be categorized as abdominal versus
vasomotor symptoms, and based on the time of onset as early versus late. Early dumping
systemic symptoms: Desire to lie down Palpitations Fatigue Faintness Syncope Diaphoresis
Headache Flushing Early dumping abdominal symptoms: Epigastric fullness Diarrhea Nausea
Abdominal cramps Borborygmi These symptoms can be collectively referred to as dyspepsia.
Late dumping symptoms: Perspiration Shakiness Difficulty to concentrate Decreased
consciousness Hunger For patients with dumping syndrome, dietary modification is the first-
line management option. The following pieces of advice are appropriate: Daily energy intake is
divided into 6 meals. Fluid intake during and with meals be restricted and liquids be avoided
for at least half an hour after a meal. Avoiding simple sugars. Milk and milk products are
generally not tolerated and should be avoided. Because carbohydrate intake is restricted,
protein and fat intake should be increased to fulfill energy needs. Additional points to consider:
Supplementation with dietary fiber has proven effective in the treatment of hypoglycemic
episodes. Many medical therapies have been tested, including pectin, guar gum, and
glucomannan. These dietary fibers form gels with carbohydrates, resulting in delayed glucose
absorption and prolongation of bowel transit time. Dietary change to a low-carbohydrate, high
protein diet, as well as the use of
alpha-glucosidase inhibitors, may be useful to control the symptoms of dumping. This is
preferential to subtotal or total pancreatectomy in those persons with severe symptoms. Most
patients have relatively mild symptoms and respond well to dietary changes. In some patients
with postprandial hypotension, lying supine for 30 minutes after meals may delay gastric
emptying and also increase venous return, thereby minimizing the chances of syncope.

49. 47 A 70-year-old woman presents to the emergency department with progressive


abdominal distention for the past 3 days. She has not passed any stool or flatus. She has
vomited several times since this morning. She relates that symptoms started developing 3
days ago after she fell off her bed. On examination she, has a blood pressure of
100/75mmHg, pulse of 90bpm, respiratory rate of 21 breaths per minute and temperature
of 37.6°C. There is no tenderness, rebound or guarding on abdominal examination.
Which one of the following is most likely to be the cause of this presentation?
A Sigmoid volvulus.
B Cecal volvulus.
C Pneumatosis coli.
D Pseudo-obstruction.
E Toxic megacolon.
Option D is correct Of the options, the most consistent one with the history of fall (trauma)
and the given clinical presentation is acute colonic pseudo-obstruction, also known as Ogilvie
syndrome. Acute colonic pseudo-obstruction is a disorder characterised by gross dilatation of
the cecum and the right hemicolon in the absence of an anatomic lesion obstructing intestinal
flow. Acute colonic pseudo-obstruction is associated with an underlying disease in 95% of
patients, including: Non-operative trauma Infection Myocardial infarction/ heart failure
Obstetric or gynecologic disease Abdominal/pelvic surgery Neurological problems e.g.
Parkinson disease, spinal cord injury, multiple sclerosis, Alzheimer disease, etc Orthopedic
surgery Miscellaneous medical conditions e.g. metabolic, cancer, respiratory failure, renal
failure Miscellaneous surgical conditions e.g. urologic, thoracic, neurosurgery Studies has
suggested that non-operative trauma is among the most common causes of acute colonic
pseudo-obstruction. Acute colonic pseudo-obstruction is more common in men and in patients
over the age of 60. Nausea, vomiting, abdominal pain, constipation, and, paradoxically,
diarrhea are the primary manifestations. Abdominal distention is always present and can cause
labored breathing. On physical examination, the abdomen is tympanic; however, bowel sounds
are present in almost 90% of patients. Peritoneal signs are absent in the early stages of the
disease; if they develop, they suggest impending perforation. (Options A and B) Volvulus
(cecal or sigmoid) is associated with acute onset of pain and obstruction, but a volvulus of
three-day duration would have resulted in a very ill and toxic patient due to ischemia of the
affected part of the colon, signs of peritoneal irritation, etc. (Option C) Pneumatosis coli is
presence of gas in the large intestinal wall. It is a condition not a diagnosis. (Option E) Patients
with toxic megacolon typically appear very ill with fever, tachycardia and abdominal
tenderness. There is often a history of bloody diarrha or other signs and symptoms of chronic
inflammatory bowel disease.

50. During endoscopic evaluation of a 65-year-old man for dyspepsia, he is found to have
an abnormal-appearing patch of mucosa in his distal part of the esophagus. Biopsy result
is significant for Barrett;s esophagus and metaplasia. Which one of the following is
correct regarding his condition?
A He shoudl be monitored every 2-3 years by endoscopy and biopsy.
B He should be monitored by endoscopy every third month.
C There is 70-80% risk of progression to adenocarcinoma.
D Hiatus hernia has no association with Barrett’s esophagus.
E Barrett's esophaus is an indication of urgent surgical intervention.
Option A is correct With metaplasia present in this patient, surveilance with endoscopy and
biopsy should be performed every 2-3 years. Other options are incorrect.. Barrett’s esophagus
is a condition in which metaplastic columnar epithelium, replaces the stratified squamous
epithelium of the distal esophagus. The metaplastic epithelium is acquired as a consequence of
chronic gastro-esophageal reflux disease (GERD). Hiatus hernia predisposes to GERD and
indirectly to Barrett’s esophagus. Barrett’s esophagus is a premalignant condition with a 40- to
125-fold increase in the incidence of adenocarcinoma of the lower esophagus; however, the
absolute risk relatively small and approximately 0.5% per year. Every patient with the alarming
symptoms of weight loss, anemia, heme-positive stool, and dysphagia, or GERD for more than
5 years should undergo endoscopy and biopsy. Further management plan depends on the
histological findings and is summarized in the following table: Endoscopic Findings
Management Barrett’s esophagus (metaplasia) Repeat endoscopy every 2-3 years for lesions ≥
3cm and every 3-5 years for lesions <3 cm. Low-grade dysplasia Repeat endoscopy 6 months.
High grade dysplasia Distal esophagetomy.

51. A 57-year-old man presents to the emergency department with fever and right upper
quadrant pain. On examination, he has a blood pressure of 80/60mmHg, pulse rate of 110
bpm, respiratory rate of 22 breaths per minute and temperature of 38.9°C. The right
upper quadrant is tender to palpation. Laboratory studies reveal high white cell count
and mildly elevated bilirubin. Based on the findings you suspect acute cholangitis as the
diagnosis. Which one of the following organisms is most likely to have caused sepsis?
A Staphylococcus aureus.
B Pseudomonas aeruginosa.
C Enterococcus fecalis.
D Escherichia coli.
E Treponema pallidum.
Option D is correct Acute cholangitis is a clinical syndrome characterized by fever, abdominal
pain and jaundice, developing as a result of stasis and infection in the biliary tract. It can
rapidly progress to sepsis. Biliary system obstruction is the most common predisposing factor.
Culture of bile, ductal stones, and blocked biliary stents are positive in over 90% of cases,
yielding a mixed growth of gram negative and gram positive bacteria. The most common
bacteria isolated have colonic origin. Escherichia coli is the major bacterium isolated (25 to
50%), followed by Klebsiella (15-20% percent) and Enterobacter species (5-10%).
Enterococcus species (e.g. enterococcus fecalis) are the most common gram positive bacteria
(10-20%). Anaerobes, such as bacteroides and Clostridia, are usually present as isolated
pathogens , but they are rarely the sole infecting organisms and it is not clear if they play a role
in acute cholangitis.

52. A 57-year-old man presents to the emergency department with absolute constipation
for 2 days and ongoing vomiting for the past 8 hours. On examination, the abdomen is not
distended or tender. The rest of the exam is unremarkable except signs of mild
dehydration. Gastrograffin studies confirm the presence of small bowel obstruction.
Which one of the following can be the most common cause of small bowel obstruction?
A Postoperative adhesions.
B Bowel cancer.
C Crohn’s disease.
D Hernias.
E Hypoklemia.
Option A is correct Post-operative adhesions are the most common cause of small bowel
obstruction (SBO). The risk of developing an obstruction after surgery from postoperative
adhesions is estimated to be 9% within the first year after abdominal surgery, 19% by 4 years,
and 35% by 10 years. The incidence of adhesive SBO seems to be higher in open surgeries than
in laparoscopic surgeries. The cause for the differences in the rates of postoperative adhesions
between open and laparoscopic surgeries is not clear, but is likely to be related to the degree of
manipulation and trauma to the intraabdominal tissues. Malignant tumors are the second most
common cause of SBO, accounting for about 20% of cases. SBO has been described in as
many as 42% of women with ovarian carcinoma and 28% of patients with colorectal
carcinomas. Hernias are the third leading cause of SBO and account for approximately 10% of
all cases. Ventral and inguinal hernias are most commonly assoiciated with obstruction;
however, internal hernias, femoral, obturator and parastomal hernias also can contribute to
SBO. Other less common causes of small bowel obstruction include: Strictures e.g. caused by
Crohn’s disease Small bowel tumors Trauma Intussusception Bezoars Gallstone ileus Superior
mesenteric artery syndrome Hypoklemia can cause temporary paralysis of ileum in children,
but it is a rare cause of small bowel obstruction in adults.

53. A 56-year-old man is brought to the emergency department with upper


gastrointestinal bleeding evident by bright-red hematemesis. He is a known case of
alcoholic liver disease. Two endoscopic attempts to stop the bleeding fail. Which one of
the following management options should be considered if the least likelihood of hepatic
encephalopathy is desired?
A Octreotide.
B Repeat endoscopy.
C Transjugular intrahepatic portosystemic shunt (TIPS).
D Distal splenocaval shunt.
E Renocaval shunt.
Option D is correct The history of alcoholic liver disease and the bright red bleeding indicates
variceal bleeding with high certainty. Varicose esophageal veins are the most common cause of
variceal bleeding. Although at occasion gastric varices can be the source. Currently, endoscopic
therapy is the definitive treatment for active variceal bleeding. It can be performed at the same
time of the diagnostic endoscopy. Two forms of endoscopic treatment are commonly used:
Sclerotherapy - Sclerotherapy includes injection of a sclerosant solution into the varices
through an injection needle that is placed through the endoscope. Variceal band ligation – The
procedure is similar to hemorrhoidal banding and involves placing small elastic bands around
varices in the distal 5 cm of the esophagus. When first endoscopic attempt fails to stop the
bleeding a second attempt is tried. These attempts are successful in the cessation of the
bleeding in 85-90% of patients. In the minority of patients with recurrent or persisting,
transjugular intrahepatic portosystemic shunting (TIPS) is the preferred method in emergency
setting: a needle catheter is passed through the jugular vein and into the hepatic vein from there.
With the needle a hole is poked into the hepatic vein. The needle is then extruded and advanced
through the live parenchyma to the intrahepatic portion of the portal vein. By doing so, the
blood pressure in the portal vein is decreased and the portal blood flow bypasses the liver. A
notorious complication of this procedure is hepatic encephalopathy because the blood skips the
liver parenchyma and ammonium content of blood increases. Surgical intervention is another
option. Surgical procedures fall into either of these two categories: 1. Shunt surgeries – a shunt
is created between portal and systemic venous system 2. Non-shunt surgeries - the two
procedures used are (a)esophageal transection and (b) devascularization of gastro-esophageal
junction Shunt surgeries can be categorized as: Nonselective - those that decompress the entire
portal tree and divert all flow away from the portal system, such as portacaval shunts Selective
- those that compartmentalize the portal tree into a decompressed variceal system, while
maintaining sinusoidal perfusion via a hypertensive superior mesenteric-portal compartment,
such as a distal splenorenal shunt, or a distal splenocaval shunt Partial - those that incompletely
decompress the entire portal tree and thereby, maintain some hepatic perfusion ! Selective
shunting maintains near normal portal flow while decompresses the varicose veins. Since the
portal flow is normal the risk of hepatic encephalopathy is minimal compared to TIPS, non-
selective, or partial shunting. Of the options, the only one that can be applied as the next best
step in management to both stop the bleeding and decrease the risk of hepatic encephalopathy
is splenocaval shunting. Of all patients with selective shunting, only 10-15% develop
encephalopathy; however, since retroperitoneal lymphatics are diverted, ascites is a common
complication. It should be noted though that the routine procedure after endoscopic measures
failed is TIPS in spite of the risk of hepatic encephalopathy. Attempting a third endoscopy or
medical treatment with octreotide are not likely to stop the bleeding and are incorrect options.
Renocaval shunting does not lead to decompression of portal system.

54. A 32-year-old man presents to the emergency department with abdominal pain and
fever since yesterday, which now has localized in the pelvis. On examination, he has a
blood pressure of 110/85mmHg, pulse rate of 98 bpm, respiratory rate of 22 breaths per
minute and fever of 38.2°C. Palpation of the abdomen elicits no tenderness, rebound or
guarding, but there is mild tenderness on rectal exam. Which one of the following would
be the next best step in management of this patient?
A Laparoscopic drainage and appendectomy.
B Open drainage.
C CT scan of the pelvis and the abdomen.
D Ultrasonography of the pelvis and the abdomen.
E Intravenous antibiotics.
Option D is correct The clinical picture is suggestive of pelvic appendicitis. The most
common symptom of appendicitis is abdominal pain. Typically, symptoms begin as
periumbilical or epigastric pain migrating to right lower quadrant of the abdomen. This pain
migration is the most characteristic feature in the patient’s history. However, in up to 30% of
the time, the appendix may be hidden from the anterior peritoneum by being in pelvic,
retroileal or retrocolic (retroperitoneal retrocaecal) position. The hidden position of the
appendix may significantly alter the clinical manifestation of the appendicitis. A patient with a
pelvic appendix may show no abdominal signs, but rectal examination may cause tenderness in
cul-de-sac. Additionally, an obturator sign (pain on passive internal rotation of the flexed right
thigh) may be present in a patient with pelvic appendix. With typical presentation of
appendicitis, imaging studies are not necessary and the patient will be taken to the operating
room for appendicectomy, but when diagnosis of appendicitis is not certain (such as in this case)
imaging studies are required for confirmation. CT scan, specially the technique of appendiceal
CT, is the most accurate imaging study (more accurate than ultrasonography). Appendiceal CT
gives the highest diagnostic yield. If appendiceal CT is not available, standard
abdominal/pelvic CT with contrast remains highly useful and may be more accurate than
ultrasonography. However, because of concerns about patient exposure to radiation during CT
scans, ultrasonography has been suggested as a safer primary diagnostic modality for
appendicitis, with CT scanning used secondarily when ultrasounds are negative or inconclusive.
A healthy appendix usually cannot be viewed with ultrasonography. When appendicitis occurs,
the ultrasonogram typically demonstrates a noncompressible tubular structure of 7-9 mm in
diameter. CT scan is the next step if ultrasound is non-diagnostic. NOTE - Pelvic appendicitis
may present with: An absence of abdominal wall rigidity and tenderness Tenderness in the
rectovesical pouch and Pouch of Douglas on rectal examination Spasm of psoas muscle on
right-hand-side Diarrhea due to rectal irritation by the inflamed appendix Frequency of
micturition due bladder irritation by the inflamed appenix Hypogastric pain on flexing and
internally rotating the hip - due to contact of inflamed appendix with obturator internus muscle

55. A 56-year-old man presents to the emergency department with acute-onset severe
epigastric pain, fever, nausea and vomiting. Past medical history includes type 2 diabetes
mellitus, cardiomyopathy and hypertension. Abdominal ultrasound shows a stone in the
bile duct, multiple gallstones in the gallbladder and a dilated common bile duct of 6mm.
Laboratory studies are significant for elevated conjugated bilirubin, elevated white cell
count and a normal hemoglobin level. Liver function tests show a 6-time rise in alanine
aminotransferase and aspartate aminotransferase. Lipase and amylase are also elevated.
Which one of the following options is the most appropriate next step in management?
A Abdominal CT scan.
B ERCP.
C MRCP.
D Endoscopy.
E Monitor liver function tests.
Option B is correct The clinical and laboratory picture suggests acute pancreatitis secondary
to gallstones and common bile duct obstruction that has led to LFT abnormalities as well.
Gallstone pancreatitis necessitates specific therapeutic consideration in addition to conventional
treatment of acute pancreatitis. Multiple studies suggest that early endoscopic retrograde
cholangiopancreatography (ERCP), within 24-48 hours, with papillotomy, or surgical
intervention to remove bile duct stones is beneficial in patients with acute biliary pancreatitis
and the presence of the following: Concomitant acute cholangitis Persisted common bile duct
obstruction evident by worsening signs or symptoms of obstruction (clinical deterioration), or
deteriorating liver function tests Endoscopic papillotomy is typically accompanied by
placement of a plastic biliary stent to decrease the risk of post-ERCP pancreatitis. With
increased liver function tests (LFTs), especially the elevated unconjugated bilirubin, this patient
should be considered to have concomitant cholangitis until proven otherwise. Considering the
likelihood of acute cholangitis caused by CBD obstruction, this patient should undergo early
ERCP as the most appropriate next step in management, after rehydration, analgesia, and
antibiotics. LFTs are used to monitor the response to treatment. An ultrasound scan (not an
option) is the very first imaging investigation for patients with signs and symptoms of
pancreatitis or pnacreatic cancer if manifestations of biliary obstruction are also present.
Otherwise, Abdominal CT scan is used; However, in the absence of clinical features of biliary
obstruction, CT scan is the preferred diagnostic modality. CT scan is the most important
imaging test for the diagnosis of acute pancreatitis, its severity and potential complication
Patients with clinical and biochemical features of acute pancreatitis. CT scan is indicated if:
there is no improvement with conservative therapy complications are suspected other diagnoses
than pancreatitis is suspected

56. Which one of the following is the most common presenting symptom of a patient with
duodenal stricture secondary to duodenal ulcer?
A Vomiting 3-4 times a day.
B Abdominal distension.
C Right upper quadrant pain.
D Vomiting after one hour of meals.
E Vomiting early in the morning.
Option D is correct Gastric outlet obstruction (GOO), is the consequence of any disease
producing a mechanical obstruction to gastric emptying. Clinical entities that can cause GOO
are generally categorized into benign and malignant. Peptic ulcer disease (PUD) is among the
benign causes of GOO; however, the incidence of ulcer-induced GOO has dramatically
declined owing to adequate and efficient treatment of PUD. The mechanism of obstruction by
PUD can be either edema around the ulcer, or scar formation after the ulcer heals. The most
common symptoms of GOO, regardless of the underlying etiology are, bloating, anorexia,
nausea and vomiting. Vomiting usually is described as nonbilious, and characteristically
containing undigested food particles. Patients with gastric outlet obstruction from a duodenal
ulcer or incomplete obstruction typically present with symptoms of gastric retention, including
early satiety, bloating or epigastric fullness, indigestion, anorexia, nausea, vomiting, epigastric
pain, and weight loss. They are frequently malnourished and dehydrated and have a metabolic
insufficiency. Weight loss is frequent when the condition is chronic and is most significant in
patients with malignant disease. Abdominal pain is not frequent and usually relates to the
underlying cause, e.g. PUD, pancreatic cancer. The time of vomiting can suggest the site of
obstruction. In pyloric obstruction (more proximal) the time of vomitus is usually within the
first hour of eating, whereas in pyloric stenosis, or duodenal stenosis or obstruction (more distal)
the vomiting occurs after one hour because normally it takes 45 minutes to 1 hour for the food
to reach the duodenum. (Option A) If the patient has 3 or 4 meals a day he may vomit 3 to 4
times within one hour after each meal, but vomiting 3 to 4 times a day by itself is not a
common presentation of gastric outlet obstruction. (Option C) Right upper quadrant abdominal
pain is seen in hepatobiliary diseases and is not a characteristic feature of gastric outlet
obstruction. (Option B) Abdominal distension is a feature of bowel obstruction, particularly
obstruction of the large bowel. (Option E) Early morning vomiting is associated with raised
intra-cranial pressure and pregnancy and is not a feature of gastric outlet obstruction.

57. A 65-year-old man is being evaluated for assessment of the source of gastrointestinal
bleeding after he presented with complaint of melena at several occasions and was found
to be anemic. Due to the very low level of hemoglobin and symptoms such as severe
weakness and chest pain on exertion, he received several transfusions; however he was
not able to maintain a satisfactory hemoglobin level. Upper and lower endoscopy and
small bowel barium studies were performed without being able to find the source of
bleeding. Which one of the following is the next best step in management?
A Capsule endoscopy.
B Interventional angiography.
C CT angiography.
D Repeat endoscopy.
E Radionuclide imaging.
Option B is correct The case scenario an example of obscure gastrointestinal(GI) bleeding.
Obscure GI bleeding is defined as bleeding from the gastrointestinal (GI) tract that persists or
recurs without an obvious etiology found on upper endoscopy and colonoscopy and radiologic
evaluation of the small bowel (such as by small bowel follow-through or enteroclysis). Obscure
GI bleeding accounts for approximately 5% of patients with GI bleeding. In 75% of these
patients, the source of bleeding is in small bowel. The remainder of cases are due to missed
lesions in either upper or lower GI tract. Obscure bleeding is subdivided into overt or occult,
depending upon the presence or absence of clinicallyevident bleeding. Obscure occult bleedin –
This is manifested as iron deficiency anemia or recurrent positive Fecal Occult Blood Test
(FOBT) results. Obscure overt – This is manifested as recurrent episodes of clinically evident
bleeding (e.g. melena or hematochezia), or persistent blood loss. patients with ongoing blood
loss so significant to necessitate transfusion are considered to have life-threatening active
bleeding. Further management of obscure GI bleeding depends on whether the bleeding is
clinically active (overt) or inactive (occult): Active obscure GI bleeding: In patients presenting
with active bleeding, radiological investigations are the mainstay of diagnositc approach, given
their high sensitivity and non-invasiveness. CT Angiography (CTA) can detect bleeding rates
of 0.5 ml/min and above. A negative CTA signifies that catheter angiography is not indicated
since the latter is less sensitive than the CTA and has been shown to detect the source of
bleeding only if the rate of blood loss is greater than 0.5 ml/min. With a negative CTA and
continuous bleeding, technetium 99m – labeled RBC nuclear scan is used as the second-line of
investigation at some institutions while others may use this as the first-line investigation.
Among available modalities, RBC nuclear scan is most sensitive for active GI bleeding and can
detect bleeding rates as low as 0.1 mL/min. NOTE - Life-threatening hemorrhage should
indicate catheter angiography as first-line management. A hemorrhage so significant is very
likely to light up on catheter angiography. With catheter angiography, the site of bleeding can
be found ans controlled at the same time. Inactive obscure GI bleeding: Capsule endoscopy
is the option of choice. However, a negative test demands CT angiography or other radiologic
modalities (radionuclide scan, etc) for further assessment. With low hemoglobin despite several
transfusions, this patient has severe life-threatening active obscure bleeding, making catheter
angiography the most important next step in management to localize and stop the bleeding at
the same time. (Option A) Capsule endoscopy is the first option for patients with occult
obscure GI bleeding i.e. iron deficiency anemia or recurrent positive Fecal occult blood test
(FOBT) results. (Option C) CT angiography is the option of choice if the bleeding is active
(overt) but not life-threatening such as in this patient. A patient with recurrent melena or
hematochezia exemplifies a case for which CT angiography is the most appropriate next step in
management. (Option D) Repeating the endoscopic studies is unlikely to add any information
regarding the site of bleeing, as this measure has already failed to do so. (Option E)
Radionuclide imaging is used when there is obscured GI bleeding of unknown origin despite
CT angiography or capsule endoscopy. The test takes a long time to perform. This patient with
brisk and signficant blood loss is not a good candidate for this test.

58. A 45-year-old man presents to your clinic with hand and arm swelling and pain which
developed a few hours after working with a chainsaw. He has no family or personal
history of thrombophilia or deep vein thrombosis. On physical exam, he is in good health
otherwise. Generalized non-pitting edema of the right hand and arm up to the shoulder is
noted. The arm is slightly cyanosed, but not warm to touch. The remainder of the exam is
inconclusive. Which one of the following is the most likely diagnosis?
A Lymphedema.
B Subclavian vein thrombosis.
C Brachial plexus injury.
D Cellulitis.
E Lymphangitis.
Option B is correct The clinical picture is suggestive of upper extremity deep vein thrombosis
(UEDVT) (thrombosis of axillary or subclavian vein). UEDVT is rare but increasing in
incidence with considerable morbidity. Pulmonary embolism may occur in as many as one-
third of the patients. The increased incidence of UEDVT in the recent decades is directly
related to increasing use of central catheterization for chemotherapy, bone marrow
transplantation, dialysis and parenteral nutrition. UEDVT has been reported in 25% of those
with central catheters. UEDVT is classified as primary or secondary on the basis of
pathogenesis: PRIMARY UEDVT Primary UEDVT is a rare disorder (2 per 100 000 persons
per year) which is subdivided into two other categories: Effort thrombosis (the so-called Paget-
Schroetter Syndrome) - Patients with Paget-Schroetter Syndrome develop spontaneous UEDVT,
usually in their dominant arm, after strenuous activity such as rowing, wrestling, weight lifting,
or baseball pitching, but are otherwise young and healthy. The heavy exertion causes
microtrauma to the vessel intima and leads to activation of the coagulation cascade. Significant
thrombosis may occur with repeated insults to the vein wall, especially if mechanical
compression of the vessel is also present. Idiopathic UEDVT - Patients with idiopathic UEDVT
have no known trigger or obvious underlying disease. Idiopathic UEDVT can be, however,
associated with occult cancer. In one study, one fourth of patients presenting with idiopathic
UEDVT were diagnosed with cancer (most commonly lung cancer or lymphomas) within one
year of follow-up. Thoracic outlet obstruction refers to compression of the neurovascular
bundle (brachial plexus, subclavian artery, and subclavian vein) as it exits the thoracic inlet.
Although this disorder may initially cause intermittent, positional extrinsic vein compression,
repeated trauma to the vessel can result in dense, perivascular, fibrous scar tissue formation that
will compress the vein persistently. Compression of the subclavian vein typically develops in
young athletes with hypertrophied muscles who do heavy lifting or completely abduct their
arms. Cervical ribs, long transverse processes of the cervical spine, musculofascial bands, and
clavicular or first rib anomalies are sometimes found in these patients. Therefore, cervical spine
and chest plain films should be obtained in all patients undergoing evaluation for thoracic
outlet syndrome. Regardless of the etiology, UEDVT can be associated with the following
complications: Pulmonary embolism (in one-third of patients) Persistent upper extremity pain
and swelling Superior vena cava syndrome Loss of vascular access. SECONDARY UEDVT
Secondary UEDVT develops in patients with central venous catheters, pacemakers, or cancer
and accounts for most cases of UEDVT. Catheter-related thrombosis is caused by several
factors including damage to the vessel wall during catheter insertion or infusion of medication,
or the catheter impeding the blood flow through the vessel and stasis. Regardless of the
etiology, UEDVT can have the following clinical features: Vague shoulder or neck discomfort
Pain and discomfort of the arm Non-pitting edema of the hand and arm Extremity cyanosis
Dilated cutaneous veins Supraclavicular fullness Fever (often low-grade) – higher fevers may
be caused by thrombophlebitis or the underlying malignancy Facial edema and cyanosis if there
superior vena cava syndrome has occurred If the thoracic outlet syndrome is the etiology of the
UEDVT the additional following findings are other possible findings: Brachial plexus
tenderness Pain radiating to the arm and forearm Arm or hand atrophy Hand weakness
Referred pain from the brachial plexus to the fourth and fifth fingers NOTE (1) – At occasions,
UEDVT may be asymptomatic. NOTE (2) – Thrombophilia seems to be associated with
increased risk of UEDVT. (Option A) Lymphedema can be a differential diagnosis, but there is
no predisposing factor in history favoring this diagnosis. (Option C) With brachial plexus
injury, neurologic symptoms would have been present. However, UEDVT caused by thoracic
outlet syndrome may be associated with concurrent neurologic dysfunctions of the brachial
plexus. (Options D and E) Lymphangitis and cellulitis are often associated with more systemic
symptoms e.g. fever. The patient may look ill. Erythema and warmth of the limb are other
features that are absent here.

59. A 67-year-old man presents to the emergency department with facial puffiness and
swelling of the right arm and upper chest. He is moderately short of breath. On
examination, his face, arm and upper chest are slightly cyanosed and puffy. The rest of
the exam is inconclusive. Which one of the following would be the most appropriate next
investigation?
A Chest X-ray.
B Echocardiography.
C ECG.
D RAST.
E Angiography.
Option A is correct The clinical picture is consistent with superior vena cava (SVC) syndrome.
SVC syndrome results from any condition that leads to obstruction of blood flow through the
SVC. Obstruction can be caused by external compression of the SVC by adjacent pathologic
processes involving the right lung, lymph nodes, and other mediastinal structures, or by
thrombosis within the SVC. In some cases, both external compression and thrombosis coexist.
Conditions that can lead to SVC syndrome can be malignant or non-malignant: Malignant
causes - malignant mediastinal tumors are the most common cause (>80%). Of malignant
tumors bronchogenic carcinoma accounts for 75-80% of cases, with most of these being small
cell carcinomas. Non-Hodgkin lymphoma is responsible for 10-15% of mediastinal
malignancies resulting in SVC syndrome. Rare malignant causes include Hodgkin lymphoma,
metastatic cancers, primary leiomyosarcoma of the mediastinal vessels and plasmocytomas.
Non-malignant causes - some of these causes are: Mediastinal fibrosis Vascular diseases, such
as aortic aneurysm, vasculitis, and arteriorvenous fistulas Infections, such as histoplasmosis,
tuberculosis, syphilis, and actinomycosis Benign mediastinal tumors such as teratoma, cystic
hygroma, thymoma, and dermoid cyst Cardiac causes, such as pericarditis and atrial myxoma
Thrombosis related to the presence of central vein catheters Early in the clinical course of SVC
syndrome, partial obstruction of SVC may be asymptomatic, or the symptoms are so minor that
are overlooked, but as it advances toward complete SVC obstruction, the classic symptoms and
signs become more obvious: Dyspnea - the most common symptom (63% of patients) Facial
swelling Head fullness Cough Arm swelling Chest pain Dysphagia Orthopnea Distorted vision
Hoarseness Stridor Headache Nasal stuffiness Nausea Pleural effusion Lightheadedness The
characteristic physical findings include venous distention of the neck and chest wall, facial
edema, upper extremity edema, mental status changes, plethora, cyanosis, papilledema, stupor
and even coma. Bending forward or lying down can aggravate the signs and symptoms. Since
most cases of SVC are due to mediastinal malignancies, a chest X-ray is always the most
appropriate initial investigation). Chest X-ray may reveal a widened mediastinum or a mass in
the right side of the chest. One study showed that only 16% of patients with SVC syndrome
had a normal chest X-ray. CT scan, MRI, and angiography (option E) can be use for more
detailed evaluation, but not as initial assessment. RAST (option D) stands for
radioallergosorbent test and is a blood test for finding the allergen a patient is allergic to, and
has no role in diagnosis of SVC syndrome. ECG (option C) and echocardiography (option B)
are not diagnostic for SVC.

60. A 72-year-old woman presents to the emergency department with sudden onset severe
pain on righ lower quadrant and and back pain. She has the history of deep vein
thrombosis 2 months ago, for which she was started on warfarin. On examination, she has
a blood pressure of 110/75mmHg, pulse rate of 140 bpm and temperature of 37.2°C.
There is a tender mass in right iliac fossa (RIF). Plain X-ray of her abdomen is obtained
and is shown in the following photograph. Which one of the following is the most likely
diagnosis?
A Abscess of the appendix.
B Rectus sheath hematoma.
C Cecal volvulus.
D Leaking abdominal aortic aneurysm.
E Fecal impaction.
Option C is correct The X-ray shows an extensively air-filled dilated proximal colon arising
from the right side and extending to the left. With these X-ray findings along with the history
and physical findings, cecal volvulus is the most likely diagnosis. Volvulus occurs when a
segment of viscus twists around its axis and results result in obstruction as well as blood supply
compromise. The most common types of volvulus are sigmoid volvulus (more common)
and cecal volvulus (less common). Cecal volvulus tends to occur in younger patients (30-
60 years) compared to sigmoid volvulus that often is seen in elderly patients, who are
bedridden or nursing home residents.
61. The following photograph is one cut from an abdominal CT scan in a 68-year-old man.
Which one of the following is the most likely diagnosis?
A Gastric outlet obstruction.
B Cancer of the stomach.
C Simple hepatic cyst.
D Pancreatic pseudocyst.
E Hepatic hemangioma.
Option A is correct The picture shows a contrast CT scan of the abdomen. On the right side of
the picture (left side of the patient) a distended contrast-filled stomach is seen. Duodenum is on
the left side of the picture. It also contains contrast media. The right kidney is another structure
seen on the CT. In front of the vertebral body and slightly to the left, the aorta (containing
contrast material) is noticed. Other structures seen on this cut of the CT scan are lower part of
the right hepatic lobe, left hepatic lobe, spleen and parts of the pancreas. The distended and
fluid filled stomach is suggestive of gastric outlet obstruction (GOO). The distended duodenum
indicates that the obstruction has occurred at the duodenum level. Clinical entities that can
result in GOO generally are categorized into 2 groups of causes – benign and malignant. In the
past, peptic ulcer disease (PUD) was the most common cause of GOO, but currently, 50- 80%
of GOOs are due to malignancies such as pylorus adenocarcinoma, lymphoma and
gastrointestinal stromal tumour (GIST). Benign causes of GOO include pancreatic pseudocysts,
gastric varices, infections such as tuberculosis and rarely gall stones. At this level no pancreatic
or hepatic lesion is seen. Cancer of the stomach would have given an obstruction in upper parts
if it involves the pylorus. Besides, this might be the underlying cause of the CT scan findings
not the interpretation.

62. A 45-year-old man presents to the emergency department with acute onset epigastric
and right upper quadrant abdominal pain that he describes as constant and severe. He is
nauseous and has vomited 3 times since the pain started. He admits to chronic heavy
alcohol consumption. On examination, he has a blood pressure of 140/90mmHg, pulse
rate of 110bpm and temperature of 38.4°C. The epigastric area is mildly tender to
palpation. Which one of the following is the most likely diagnosis?
A Acute cholecystitis.
B Acute pancreatitis.
C Acute gastritis.
D Acute gastro-esophageal reflux disease.
E Alcohol intoxication.
Option B is correct The clinical picture and the history of heavy alcohol use suggest acute
pancreatitis as the most likely diagnosis. The cardinal symptom of acute pancreatitis is
abdominal pain. The pain is characteristically described as dull, boring and constant. The pain
is often sudden-onset and gradually becomes worse. Most often, the pain is felt in the upper
abdomen usually in the epigastric region, but sometimes is perceived more on the right or left
side, depending on which part of the pancreas is involved. In about 50% of the patients, the
pain directly radiates through the abdomen to the back. The duration of pain is variable but
typically lasts more than 1 day. Restless and agitation may be noted. At times of pain patients
bend forward to the so-called “pancreatic position” or lie in the knee-chest position. Anorexia,
nausea and vomiting, and diarrhea are other likely symptoms. Acute pancreatitis secondary to
alcohol, frequently occurs 1 to 3 days after a binge drinking.It can also occur after cessation of
drinking. The patient should be asked about recent operative or other invasive procedures such
as ERCP, family history of hypertriglyceridemia, previous biliary colic and binge alcohol
drinking as the major causes of acute pancreatitis. (Option A) Acute cholecystitis causes pain
in right upper quadrant, fever and leukocytosis. Murphy’s sign is usually positive: the patient is
asked to inspire deeply while the right subcostal area is palpated. The Murphy sign is cessation
of inspiration due to pain. Patients with acute cholecystitis may experience increased
discomfort and hold in mid-inspiration. Abdominal examination may show voluntary and
involuntary guarding. (Option B) Acute gastritis usually can present with pain in epigastrium
associated with nauseas and vomiting and epigastric tenderness but fever would not be a
feature. (Option D) Gastroesophageal reflux disease (GERD) presents with heartburn, chronic
cough, a metalic taste in the mouth. The given clinical picture is completely different from that
of GERD. (Option E) Patients with lcohol intoxication presents with slurred speech,
nystagmus, disinhibited behavior, incoordination, unsteady gate, memory impairment, stupor,
or coma depending on the severity of intoxication.

63. Which one of the following is the most important diagnostic feature of achalasia? A
Dysphagia for solids.
B Dysphagia for liquids.
C Dysphagia for both solids and liquids.
D Regurgitation.
E Weight loss.
Option C is correct Achalasia is a disease of unknown cause in which there is a loss of
peristalsis in the distal esophagus and a failure of lower esophageal sphincter relaxation with
swallowing. Both of these abnormalities impair esophageal emptying. The disease equally
affects men and women mostly between 25-60 years. Achalasia before adolescence is very rare.
Manifestations of achalasia are primarily due to the defect in lower esophageal sphincter
relaxation. Constant contraction of this sphincter in achalasia causes functional obstruction of
the esophagus that persists until the hydrostatic pressure of the retained material exceeds the
pressure generated by the sphincter muscle. Dysphagia for solids (91% percent) and liquids
(85%) is the primary clinical feature of achalasia. Although dysphagia for liquids can occur in
patients with other esophageal motility disorders (e.g. progressive systemic sclerosis), this
symptom is most characteristic of achalasia and strongly suggests the diagnosis. Difficulty
belching is another important symptom that is seen in 85% of patients. Weight loss,
regurgitation, chest pain and heart burn is seen in as many as 40% to 60% of patients. Weight
loss is often in the range of 5-10kg, but can be more significant. Although dysphagia for liquids
is found characteristically in patients with achalasia, it is almost always associated with
simultaneous dysphagia for solids. Dysphagia to liquids alone is unlikely to be caused by
achalasia. Dysphagia initially for only solids is caused by mechanical esophageal obstruction
such as in cancer, rings and webs.

64. A 39-year-old Aboriginal man presents to your practice with a 6-week history of
abdominal pain, nausea, vomiting and bowel motions that are difficult to flush down the
toilet. The abdominal pain is described as a constant and disabling radiating to back. The
pain is neither brought up, nor alleviated by eating. He is a chronic alcoholic and
continues to drink alcohol in large amounts. Which one of the following investigations is
most likely to reveal the underlying cause of his symptoms?
A Investigations for exocrine function of pancreas.
B Stools exam.
C Trans abdominal ultrasound.
D ERCP.
E Plain abdominal X-ray.
Option C is correct The clinical picture and history of excessive alcohol use makes chronic
pancreatitis the most likely diagnosis. Chronic pancreatitis presents with epigastric pain as the
most dominant feature. The natural history of pain in chronic pancreatitis is highly variable.
Most patients experience intermittent attacks of pain at unpredictable intervals, while a
minority of patients have chronic pain. In most patients, pain severity either decreases or
resolves over 5-25 years. In alcohol-induced disease, alcohol cessation may reduce the severity
of pain. The pain is constant, often radiates to back and may be associated with nauseas and
vomiting. It is often felt in the epigastric area but may be felt on the left side or even right side.
The pain may or may not be related to eating. If related, there may be weight loss due to fear of
eating. At times of pain patients bend forward to the so-called “pancreatic position” or lie in the
knee-chest position on their right or left side to decrease the pain intensity. Patients with severe
pancreatic exocrine dysfunction cannot properly digest complex foods and absorb partially
digested breakdown products. Nonetheless, clinical significant protein and fat deficiencies
does not develop until more than 90% of the pancreatic function is lost. However, fear of
eating due to pain brought up by eating, can result in early weight loss. With severe chronic
pancreatitis, insulin deficiency and diabetes mellitus may also develop. Abdominal CT scan,
MRI, ultrasound, plain X-ray films and ERCP can be used for diagnosis, but CT scan is the
best initial test used for imaging studies when chronic pancreatitis is suspected. CT scans
are 75-90% sensitive and 85% specific. Ultrasound is the second-line initial imaging study
with a sensitivity of 60-70% and specificity of 80-90% and the best among option in the
absence of CT scan. Calcifications within the pancreatic duct are present on plain films in
approximately 30% of cases, making palne films a less desirable option. The characteristic
finding on imaging is pancreatic duct calcifications, ductal dilation, enlargement of the
pancreas and fluid collection. Calcium deposition is most commonly seen in alcoholic
pancreatitis, but is also present hereditary and tropical forms of the disorder. It is rare in
idiopathic pancreatitis. Magnetic resonance cholangiopancreatography (MRCP) is becoming
the diagnostic test of choice since it can show calcification, without any radiation risks, but is
expensive and not readily available. For this reason, it is not an ideal option for initial
assessment. Endoscopic retrograde cholangiopancreatography (ERCP) is reserved for situations
where non-invasive modalities are not available, are equivocal and for intervention.
65. A 51-year-old Aboriginal male with history of acute pancreatitis presented with
persistent abdominal pain and loss of appetite. On examination, an abdominal mass is
found. A contrast abdominal CT scan is performed with one cut shown in the following
photograph. Which one of the following is the most likely diagnosis?

A Chronic pancreatitis.
B Acute pancreatitis.
C Pancreatic pseudocyst.
D Gastric adenocarcinoma.
E Gastric lymphoma.
Option C is correct The round lesion with hypodense homogenous content and the thin
smooth wall in the vicinity of and obliterating the pancreas is highly suggestive of a pancreatic
cyst or pseudocyst. With the history of previous acute pancreatitis, a pseudocyst would be the
most likely diagnosis. A pancreatic pseudocyst is a collection of pancreatic juice encased by
reactive granulation tissue (and not epithelial tissue) in or around the pancreas. Pseudocysts can
be single or multiple, small or large, and can be located either within or outside of the pancreas.
Most pseudocysts communicate with the pancreatic ductal system and contain high
concentrations of digestive enzymes such as amylase and lipase. The walls of pseudocysts are
formed by adjacent structures such as the stomach, transverse mesocolon, gastrocolic omentum,
and the pancreas. The lining of pancreatic pseudocysts consists of fibrous and granulation
tissue. Lack of an epithelial lining distinguishes pseudocysts from true pancreatic cysts. The
mechanism by which a pseudocyst is formed is necrosis and liquefaction of the pancreatic
necrosis of pancreatic or peripancreatic tissue. Pseudocyst can be seen in the following
situations: After an episode of acute pancreatitis (in 10% of patients) - necrosis of
peripancreatic tissue progresses to liquefaction and pseudocyst formation. Alternatively, a
pseudocyst may result from parenchymal necrosis leading to the complete ductal disruption,
and gross leakage of pancreatic juice. In patients with chronic pancreatitis – pseudocysts
may develop after acute attacks of pancreatitis or after the pancreatic duct is obstructed. The
latter causes increased Intraductal pressure and leakage of pancreatic juice. After abdominal
blunt or penetrating trauma (including iatrogenic injuries such as pancreatic surgery – injury
can directly disrupt the duct and causes leakage. Clinical manifestations and complications of
pancreatic pseudocysts include: Expansion of the pseudocyst can produce abdominal pain,
duodenal or biliary obstruction, vascular occlusion, or fistula formation into adjacent viscera,
the pleural space, or pericardium Spontaneous infection Digestion of an adjacent vessel can
result in a pseudoaneurysm, which can produce a sudden, painful expansion of the cyst or
gastrointestinal bleeding due to bleeding into the pancreatic duct Pancreatic pleural effusion -
can result from disruption of the pancreatic duct with fistulisation to the chest Pancreatic
peritonitis - can be caused by disruption of the pancreatic duct with fistulisation to the
Abdomen

66. A 34-year-old alcoholic man, who survived an episode of acute pancreatitis 4 weeks
ago, has presented with mild discomfort in the epigastrium for the past few days. He is
concerned that the disease might have recurred. He denies nausea and vomiting and
describes the pain as constant and nagging. Physical examination is unremarkable. A
contrast abdominal CT scan is performed showing a 4-cm pancreatic pseudocyst. Which
one of the following is the next best step in management?
A Endoscopic decompression.
B Open surface decompression.
C Observation.
D Percutaneous catheter drainage.
E Urgently take him to the operating room.
Option C is correct A pancreatic pseudocyst is a collection of pancreatic juice encased by
reactive granulation tissue (and not epithelial tissue) in or around the pancreas. Pseudocysts can
be single or multiple, small or large, and can be located either within or outside of the pancreas.
Most pseudocysts communicate with the pancreatic ductal system and contain high
concentrations of digestive enzymes such as amylase and lipase. The walls of pseudocysts are
formed by adjacent structures such as the stomach, transverse mesocolon, gastrocolic omentum,
and pancreas. The lining of a pancreatic pseudocysts consists of fibrous and granulation tissue.
Lack of an epithelial lining distinguishes pseudocysts from true pancreatic cysts. The
mechanism by which a pseudocyst is formed is necrosis and liquefaction of the pancreatic
necrosis of pancreatic or peripancreatic tissue. Pseudocyst can be seen in the following
situations: After an episode of acute pancreatitis (in 10% of patients) - necrosis of
peripancreatic tissue progresses to liquefaction and pseudocyst formation. Alternatively, a
pseudocyst may result from parenchymal necrosis leading to the complete ductal disruption,
and gross leakage of pancreatic juice. In patients with chronic pancreatitis – pseudocysts may
develop after acute attacks of pancreatitis or after the pancreatic duct is obstructed. The latter
causes increased Intraductal pressure and leakage of pancreatic juice. After abdominal blunt or
penetrating trauma (including iatrogenic injuries such as pancreatic surgery – injury can
directly disrupt the duct and causes leakage. Clinical manifestations and complications of
pancreatic pseudocysts include: Expansion of the pseudocyst can produce abdominal pain,
duodenal or biliary obstruction, vascular occlusion, or fistula formation into adjacent viscera,
the pleural space, or pericardium. Spontaneous infection. Digestion of an adjacent vessel can
result in a pseudoaneurysm, which can produce a sudden, painful expansion of the cyst or
gastrointestinal bleeding due to bleeding into the pancreatic duct. Pancreatic pleural effusion -
can result from disruption of the pancreatic duct with fistulisation to the chest. Pancreatic
pleural effusion: can be caused by disruption of the pancreatic duct with fistulisation to the
abdomen. NOTE - up to 40% of pseudocysts resolve without intervention; however, they can
produce a wide range of clinical problems depending on the location and extent of the fluid
collection and the presence of infection. Pancreatitis pseudocysts are diagnoses with CT or
ultrasound scan. If the diagnosis is in doubt, contents can be aspirated and examined. The
dictum mentioning that intervention is needed if the cysts are larger than 6 cm or persist
beyond 6 weeks is no longer in use, and surgical intervention should be considered in (even one
criterion is sufficient): Compression of large vessels (clinical symptoms or seen on CT scan)
Gastric or duodenal outlet obstruction Stenosis of the common bile duct due to compression
Infected pancreatic pseudocysts Hemorrhage into pancreatic pseudocyst Pancreatic-pleural
fistula Pancreatic pseudocysts and symptoms: Early satiety Nausea and vomiting Pain Upper
gastrointestinal bleeding Asymptomatic pancreatic pseudocyst AND either of the following:
Pseudocysts> 5cm, unchanged in size and morphology for more than 6 weeks Diameter> 4cm
and extrapancreatic complications in patients with chronic alcoholic pancreatitis Suspected
malignancy Surgical drainage (laparotomy and internal and external drainage) is the criterion
standard against which all other interventions are measured in terms of success rate, mortality
and recurrence rate. In recent years however endoscopic drainage has been introduced and can
be applied provided that the cyst is near the stomach or duodenal wall: There are two main
types of endoscopic drainage: Transmural drainage: in this method, using endoscope a small
incision is made in the stomach (endoscopic cystgastrostomy [ECG]) or in duodenum
(endoscopic cystduodenostomy [ECD]) to let the pseudocyst drain into the stomach or
duodenum. ECD is preferred over ECG. Transpapillary drainage: this method is safer and more
effective than transmural drainage, but requires that the cyst communicates with pancreatic
duct. Using ERCP and through the pancreatic duct, the cyst is reached and poked so that its
content can drain into the pancreatic duct. Stents may be used to facilitate drainage. NOTE –
pseudocysts should have a mature capsule (wall thickness>3mm and <1cm), bulge the lumen
and have minimum size of 5-6 cm to become eligible for endoscopic drainage. Generally,
endoscopic drainage methods are preferred over open surgical treatment if eligibility is met and
there is no contraindication, because these methods are less invasive and associated with fewer
complications. Laparotomy with cyst excision and internal and external drainage is still the
gold standard management option; however it is considered first-line therapy for surgical
intervention if: Complicated pseudocysts i.e. infected and necrotic pseudocysts Pseudocysts
associated with pancreatic duct stricture and a dilated pancreatic duct Suspected cystic
neoplasia Presence of pseudoaneurysm unless it has been embolised before the procedure
Coexistence pseudocysts and bile duct stenosis Complications such as compression of the
stomach or the duodenum, perforation or pseudoaneursyms NOTE – pseudoaneurysm is an
absolute contraindication to endoscopic drainage unless it is embolised prior to the procedure.
Generally, patients with symptomatic pseudocysts should undergo interventional measures for
pseudocyst drainage. The procedure of choice is endoscopic drainage, but as mentioned before,
for a pseudocyst to be amenable to this procedure, it has to be at least 5-6 in size as well as
bulging into the lumen. This cyst with 4 centimeters in size is unlikely to be drained by
endoscopic measures; hence, open decompression (surgical drainage) should be considered;
however, since the pain is mild, a watchful observation would be best management here in an
attempt to avoid the high rates complications associated with surgical drainage. If the
symptoms were more pronounce, intolerable or indicative of more serious complications
surgical drainage (open surface drainage) would have been the option of choice. Percutaneous
catheter drainage has low success rate and high recurrent rates. It is never considered for
treatment of a pancreatic pseudocyst. However, in infected pseudocysts it is the procedure of
choice for sampling and examining the material as the most appropriate initial step.

67. A 57-year-old man presents to your practice complaining of abdominal discomfort


and pain for the past 6 months. The pain is predominantly felt in the epigastric area. He
does not smoke but admits to chronic alcohol use. On examination, no abdominal
tenderness is elicited. The remainder of the exam is inconclusive. An abdominal
ultrasound scan is arranged that reveals the presence of a 10-cm cystic lesion in the
epigastric area. Which one the following is the most appropriate management option?
A Endoscopic gastrostomy.
B Laparotomy.
C Percutaneous drainage.
D Drainage through ERCP.
E Conservative management and re-evaluation in 6 months.
Option D is correct A pancreatic pseudocyst is a collection of pancreatic juice encased by
reactive granulation tissue (not epithelial tissue) in or around the pancreas. Pseudocysts can be
single or multiple, small or large, and can be located either within or outside the pancreas. Most
pseudocysts communicate with the pancreatic ductal system and contain high concentrations of
digestive enzymes such as amylase and lipase. The walls of pseudocysts are formed by adjacent
structures such as the stomach, transverse mesocolon, gastrocolic omentum, and pancreas. The
lining of pancreatic pseudocysts consists of fibrous and granulation tissue; the lack of an
epithelial lining distinguishes pseudocysts from true pancreatic cysts. The mechanism by which
a pseudocyst is formed is necrosis and liquefaction of the pancreatic necrosis of pancreatic or
peripancreatic tissue. Pseudocyst can be seen in the following situations: After an episode of
acute pancreatitis (in 10% of patients) - necrosis of peripancreatic tissue progresses to
liquefaction and pseudocyst formation. Alternatively, a pseudocyst may result from
parenchymal necrosis leading to the complete ductal disruption, and gross leakage of pancreatic
juice. In patients with chronic pancreatitis – pseudocysts may develop after acute attacks of
pancreatitis or after the pancreatic duct is obstructed. The latter causes increased intraductal
pressure and leakage of pancreatic juice. After blunt or penetrating abdominaltrauma (including
iatrogenic injuries) – The injury can directly disrupt the duct and causes leakage. Clinical
manifestations and complications of pancreatic pseudocysts include: Expansion of the
pseudocyst can produce abdominal pain, duodenal or biliary obstruction, vascular occlusion, or
fistula formation into adjacent viscera, the pleural space, or pericardium. Spontaneous infection.
Digestion of an adjacent vessel can result in a pseudoaneurysm, which can produce a sudden,
painful expansion of the cyst or gastrointestinal bleeding due to bleeding into the pancreatic
duct. Pancreatic pleural effusion -can result from disruption of the pancreatic duct with
fistulization into the chest. Pancreatic peritonitis - can be caused by disruption of the pancreatic
duct with fistulisation to the abdomen. NOTE - up to 40% of pseudocysts resolve without
intervention; however, they can produce a wide range of clinical problems depending on the
location and extent of the fluid collection and the presence of infection. Pancreatitis
pseudocysts are diagnoses with CT or ultrasound scan. Where the diagnosis is in doubt, the
content can be aspirated (under endoscopic ultrasonography or CT scan) and examined. The
old rule mentioning that intervention is needed if the cysts are larger than 6 cm or persist
beyond 6 weeks is no longer in use, and surgical intervention should be considered if any of the
following is present: Compression of large vessels (clinical symptoms or seen on CT scan)
Gastric or duodenal outlet obstruction Stenosis of the common bile duct due to compression
Infected pancreatic pseudocysts Hemorrhage into pancreatic pseudocyst Pancreatico-pleural
fistula Pancreatic pseudocysts and symptoms: Early satiety Nausea and vomiting Pain Upper
gastrointestinal bleeding Asymptomatic pancreatic pseudocyst AND either of the following:
Pseudocysts> 5cm, unchanged in size and morphology for more than 6 weeks Diameter> 4cm
and extrapancreatic complications in patients with chronic alcoholic pancreatitis Suspected
malignancy Surgical drainage is the criterion standard against which all other interventions are
measured in terms of success rate, mortality and recurrence rate. In recent years, however,
endoscopic drainage has been introduced and can be applied provided that the cyst is near the
stomach or duodenal wall: There are two main types of endoscopic drainage: Transmural
drainage: in this method, using endoscopy, a small incision is made in the stomach (endoscopic
cystgastrostomy [ECG]) or in duodenum (endoscopic cystduodenostomy [ECD]) to let the
pseudocyst drain into the stomach or duodenum. ECD is preferred over ECG. NOTE –
pseudocysts should have a mature capsule (wall thickness>3mm and <1cm), bulge the lumen
and have minimum size of 5-6 cm to become eligible for endoscopic drainage. Transpapillary
drainage: this method is safer and more effective than transmural drainage, but requires that the
cyst communicates with the pancreatic duct because this method includes entering the
pancreatic duct by ERCP, and from there, into the pseudocyst. Stents may be left in place to
facilitate drainage. Generally, endoscopic drainage methods are preferred over open surgical
treatment if eligibility is met and there is no contraindication because these methods are less
invasive and associated with fewer complications. Laparotomy with cyst excision and internal
and external drainage is still the gold standard management option; however it is ONLY
considered first-line therapy for surgical intervention in the following conditions: Complicated
pseudocysts i.e. infected or necrotic Pseudocysts associated with pancreatic duct stricture and a
dilated pancreatic duct Suspected cystic neoplasia Presence of pseudoaneurysm, unless it has
been embolised before the procedure Coexistence pseudocysts and bile duct stenosis
Complications such as compression of the stomach or the duodenum, perforation or
pseudoaneursyms NOTE – pseudoaneurysm is an absolute contraindication to endoscopic
drainage unless it is embolised prior to the procedure. Generally, patients with symptomatic
pseudocysts should undergo interventional measures for pseudocyst drainage. The procedure of
choice is endoscopic drainage. This cyst is 10 cm in size (>5cm) and amenable to endoscopic
drainage either by endoscopic transmural or transpapillary drainage. Transpapillary drainage
has the lowest complication rate of all the mentioned procedures and is the method of choice if
the pseudocyst communicates with the pancreatic duct. Fortunately, 80% of pseudocysts
communicate with the pancreatic duct. (Option A) Endoscopic cystgastrostomy (ECG) or
duodenostomy (ECD) are methods of choice if the pseudocyst is not communicating with the
pancreatic duct. (Option B) Laparotomy and surgical removal of the cyst is considered if
endoscopic methods fail or there is a contraindication. (Option C) Percutaneous catheter
drainage has low success rate and high recurrent rates. It is never considered for treatment of a
pancreatic pseudocyst. However, in infected pseudocysts it is the procedure of choice for
sampling and examining the material as the most appropriate initial step. (Option E)
Conservative management is not an appropriate option for symptomatic pseudocysts.
68. Which one of the following is the most common early complication of hemorrhagic
pancreatitis?
A Pseudocyst.
B Infection.
C Obstructive jaundice.
D Pancreatic fistula.
E Renal failure.
Option E is correct Acute pancreatitis can be classified into acute interstitial (most
common) and acute hemorrhagic (least common). In the first type, the gland architecture is
preserved but is edematous. Inflammatory cells and interstitial edema are prominent within the
parenchyma. In hemorrhagic type, there is marked necrosis, hemorrhage, and fat necrosis.
There is marked pancreatic necrosis along with vascular inflammation and thrombosis.
Hemorrhagic pancreatitis can rapidly result in severe hemorrhage, hypovolemia, shock and
acute renal failure. Other options are also potential complications of acute pancreatitis but often
do not occur as early as acute renal failure.

69. You are about to perform a femoral venepuncture and you should take precaution not
to damage the adjacent structures. Which one of the following is the order of structures in
the groin under the inguinal ligament from medial to lateral? A Lacunar ligament, femoral
artery, femoral vein, femoral nerve.
B Lacunar ligament, femoral vein, femoral nerve, femoral artery.
C Lacunar ligament, femoral vein, femoral artery, femoral nerve.
D Femoral vein, femoral artery, femoral nerve, lacunar ligament.
E Femoral vein. Femoral artery, lacunar ligament, femoral nerve.
Option C is correct Femoral triangle consists of three borders: Upper border: inguinal
ligament Medial border: lateral border of adductor longus Lateral border: medial border of
sartorius Femoral trinagle and its Contents

The contents of the femoral triangle from


medial to lateral are: 1. Lacunar ligament and deep femoral lymph nodes 2. Femoral vein 3.
Femoral artery 4. Femoral nerve Remember LEVAN: Lymph node chain / lacunar ligament,
Empty space, Vein, Artery and Nerve for the order of contents from medial to lateral. The
lacunar ligament is a ligament in the inguinal region that connects the inguinal ligament to the
pectineal ligament near the point where they both insert on the pubic tubercle. This ligament
comprises the medial border of the femoral canal.

70. A
55-year-old man presents with a 6-month history of increasing dysphagia for solids foods.
He has a previous history of gastroesophageal reflux going back many years. He has
managed his reflux with antacids but since the dysphagia started, his reflux has not been
so troublesome. Which one of the following is the most likely diagnosis?
A Esophageal cancer.
B Achalasia.
C Gatro-esophageal junction stricture.
D Para-esophageal hernia.
E Ulcerative esophagitis.
Option C is correct Dysphagia to solid food is more likely to be caused by mechanical
obstruction due to strictures, tumors, rings or webs. With the history of protracted gastro-
esophageal reflux disease (GERD), the most likely cause of this presentation would be stricture.
Inflammation and scarring of the esophagus result in stenosis of the esophagus most often at
the site junction to stomach. Resolution of GERD symptoms supports the diagnosis: when the
stricture develops, the amount of acid reflux is decreased. (Option A) Esophageal cancer is
another important differential diagnosis that has to be excluded, but not the most likely
diagnosis here. (Option B) Achalasia causes dysphagia to both solids and liquids. Dysphagia to
solids, but not to liquids rules out achalasia as a probability diagnosis. (Option D) Para-
esophageal hernias may present with GERD. Althogu GERD can result in this clinical picture
over time, para-esophageal hernias to not present like this per se. (Option E) Ulcerative
esophagitis is very rarely complicated by strictures, especially in patients with HIV; hence, an
unlikely diagnosis.

71. A 66-year-old man presents with intermittent right upper quadrant pain. An
ultrasound, performed for revealing the cause gallstones, reveals a lesion in the liver.
Triple phase CT scan is performed for more evaluation showing a 35 mm subcortical
lesion with early prominent dense enhancement, which spreads through the lesion in the
late portal venous phase. Which one of the following conditions would fit this description
best?
A Isolated metastatic lesion.
B Hemangioma.
C Hepatocellular carcinoma.
D Hepatic cyst.
E Hydatid cyst.
Option B is correct The patient has the provisional diagnosis of the biliary colic, most likely
due to biliary stones. The ultrasound scan – as the best initial diagnostic tool –has been used to
confirm the diagnosis. Furthermore, any associated inflammation would be evaluated.
Inflammation of the gallbladder manifests as the thickening of the wall of the gallbladder and
the presence of pericholecystic fluid. Stones might be seen in the common bile duct as well;
however, the sensitivity of ultrasound for detection of ductal stones is low (30%-50%). As a
routine procedure, when scanning for biliary problems, the sonographer will scan the liver as
well. In this case the sonographer has encountered an incidental finding, irrelevant to the
presenting symptoms, for which a triphasic CT scan of the liver has been performed. Early
prominent dense enhancement of the lesion during the arterial phase is characteristic of liver
hemangioma (the most common benign liver tumor). Hemangiomas are seen in approximately
20% of the general population. They may be solitary or multiple. The lesions typically show
intense enhancement during the arterial phase of triphasic CT scan and retain a blush of
contrast during the portal venous phase. (Options A and C) Most malignant liver tumours
(primary or metastatic) are hypovascular and will not have the early enhancement during the
arterial phase; rather, they become more pronounced during the portal venous phase. (Option
D) Cystic lesions in the liver may be simple, multiple (polycystic liver disease), neoplastic or
infective (hydatid cysts). Simple cysts are extremely common and usually asymptomatic. On
imaging, these cysts have a low-density homogenous appearance. With polycystic disease, the
number and size of the cysts often lead to symptoms. (Option E) Hydatid cysts have a
characteristic septate appearance and heterogenous appearance if they contain daughter cysts.
Liver abscesses are usually symptomatic and more likely to have a heterogenous appearance.

72. A 34-year-old woman presents to your clinic complaining of abdominal pain and
diarrhea one week after she returned from a trip to Thailand. While she was on the trip,
she first noticed abdominal pain in the right iliac fossa which resolved subsequently. On
examination, the abdomen is non-tender and soft with no rigidity or guarding. However,
digital rectal exam is tender. Which one of the following is the most likely diagnosis?
A Giardiasis.
B Celiac disease.
C Rotavirus infection.
D Urinary tract infection.
E Appendicitis.
Option E is correct Although giardiasis, viral gastroenteritis and celiac disease can all present
with diarrhea and abdominal pain, no rectal tendernesson is elicitedon rectal exam. Urinary
tract infection is not associated with tenderness on rectal exam either. Of the given options, the
only one that can justify the clinical presentation is acute pelvic appendicitis. Clinical features
of pelvic appendicitis are: Absence of abdominal wall rigidity and tenderness Tenderness in the
rectovesical pouch and/or pouch of Douglas on rectal examination Right-sided spasm of psoas
muscle Diarrhea due to irritation of the rectum by the inflamed appendix Increased frequency
of urination caused by irritation of bladder due to an inflamed appendix Hypogastric pain
brought on by internal rotation of a flexed hip due to contact of the inflamed appendix with the
obturator internus muscle

73. Which one of the following complications of acute diverticulitis carries the highest
mortality rate?
A Bleeding.
B Abscess formation.
C Peritonitis.
D Perforation.
E Intestinal obstruction.
Option D is correct Diverticular disease carries a number of potential complications including:
Bleeding, especially in the elderly Bowel perforation Intra-abdominal abscess formation
Peritonitis Fistula formation Intestinal obstruction Rupture of an inflamed diverticulum with
fecal contamination of the peritoneum occurs in only 1 to 2% of cases but is associated with a
20% mortality rate, which is the highest when compared to other complications of diverticulitis.
Perforation of diverticula into the abdominal cavity presents with the following features:
Abdominal distention Diffuse tenderness of the abdomen even to light palpation. Guarding
Rigidity Rebound tenderness Absent bowel sounds

74. You are one of the senior residents in surgery. You are called to see Mr. Kingsley, a
67-yearold man, who has just been diagnosed with acute cholangitis. Which one of the
following statements is not correct regarding the management of acute cholangitis?
A Plan for immediate decompression if the patient does not respond to initial measures.
B Plan for biliary decompression on semi-urgent basis (<72 hours) if the patient is responding
to initial resuscitation.
C Plan for urgent decompression (within 24-48hrs) if the patient is older than 70 years.
D The most appropriate method of biliary decompression is ERCP, sphincterectomy and
stenting.
E Initial aggressive resuscitation and antibiotics usually fail to get good response in majority of
cases.
Option E is correct Acute ascending cholangitis is initially managed with aggressive fluid
resuscitation and intravenous antibiotics followed by biliary decompression. Since the
infectious organisms responsible for acute ascending cholangitis are enteric gram negative
bacteria, the selected antibiotic of choice should provide appropriate coverage against these
germs. All patients with ascending cholangitis require biliary drainage. In about 85-90% of
patients, there is respond to medical therapy. In this group decompression may be performed
semi-electively during the same admission (and ideally within 72 hours); however for the
following patients urgent decompression may be considered: Patients older than 70 years
Patients with diabetes Patients with other comorbid conditions Approximately 10% to 15% of
patients (not the majority) fail to respond within 12 to 24 hours or deteriorate after initial
medical therapy and need urgent biliary decompression. Delay to do so increases the chance of
an adverse outcomes.
75. A 69-year-old woman is diagnosed with the carcinoma of the cecum. Which one of the
following is more likely to have been her initial presenting symptoms?
A Right iliac fossa (RIF) mass.
B Altered bowel habit.
C Weakness and fatigue.
D Melena.
E Bleeding.
Option C is correct Colorectal cancers may present with a wide variety of symptoms. The
presenting symptoms, to a great extend, depends on the location of the tumor. A change in
bowel habits is a less common presenting symptom for right-sided tumors because feces is
liquid in the proximal colon and the lumen caliber is larger. Right-sided tumors present with
anemia and fatigue due to chronic blood loss, while tumors of the left side are associated with
altered bowek habits and rectal bleeding. (Option A) If a right-sided tumor is large enough, a
right iliac fossa mass may be palpated. But a tumor that large has already caused significant
symptoms for which the patient ha ‫ ی‬already sought medical attention. (Option B) Altered
bowel habit is more commonly seen in left-sided colon cancers including rectal
tumours.Melena is associated with upper gastrointestinal (GI) bleeding with prolonged passage
time of the blood through the GI tract. Colorectal tumours are very unlikely to cause melena.
(Option D) Melena is associated with upper gastrointestinal bleeding. (Option E) Rectal
bleeding is more often caused by a rectal than colon cancer.

76. The photograph is one of a barium swallow series that has been performed in a 78-
year-old man. He has presented with symptoms that had been persisted for 12 months.
Which one of the following could be the most likely presenting symptom?
A Recurrent chest infection.
B Progressive weight loss.
C A retrosternal burning sensation.
D Gurgling in the neck.
E Food regurgitation.
Option E is correct The photograph shows a pocket of contrast material at the root of the neck,
as well as the contrast in the esophagus characteristic of retropharyngeal pouch (Zenker’s
diverticulum). The condition is most commonly found in the elderly population. Patients with
Zenker’s diverticulum usually have dysphagia because the primary problem is an overactive
upper esophageal sphincter, which fails to relax. Despite long-standing dysphagia, patients
usually do not have significant weight loss. When the pharyngeal pouch becomes large enough
to retain contents such as mucus, pills, sputum and food, the patient may complain of
pulmonary aspiration and recurrent chest infections, foul-smell breath, gurgling in the throat,
appearance of a mass in the neck, or regurgitation of food into the mouth. Of these symptoms,
regurgitation of food is the most distressing symptoms for which the patients seek medical
attention. A retrosternal burning sensation is a characteristic feature of gastro-esophageal reflux
disease (GERD) and is not an associated symptom in Zenker’s diverticulum.
77. 47 Which one of the following is the most common cause of acute bile duct obstruction
in tertiary care hospitals?
A Choledocholithiasis.
B Benign strictures.
C Tumors.
D Post-biliary access/manipulation by ERCP or PTC.
E Acute Cholecystitis.
Option D is correct The most common cause of bile duct obstruction in community is
choledocholithiasis. Gallstones contribute to approximately 70% of cases presenting with
biliary tree obstruction. Benign strictures and tumors are responsible for 15% of cases with
obstructed bile passage. However, it should be noted that the question does not ask about the
most contributing factor to obstruction in tertiary hospitals rather than the community. The
most common cause of bile duct obstruction in tertiary hospitals is biliary access/manipulation
by endoscopic retrograde cholangiopancreatography (ERCP) or percutaneous transhepatic
cholangiography (PTC).

78. Which one of the following is a sign of pyloric stenosis due to peptic ulcer?
A Vomiting within the first 1 hour of eating.
B Vomiting immediately after eating.
C Vomiting after 2 hours of eating.
D Regurgitation.
E Epigastric pain radiating to the back.
Option A is correct Pyloric obstruction also known as gastric outlet obstruction (GOO) is the
consequence of any disease producing a mechanical barrier to gastric emptying. Clinical
entities that can cause GOO are generally categorized into benign and malignant. Peptic ulcer
disease (PUD) is among the benign causes of GOO. The incidence of ulcer-induced GOO has
dramatically declined owing to the adequate and efficient treatment of PUD. The mechanism of
obstruction by PUD can be either edema around the ulcer, or scar formation after the ulcer
heals. The most common symptoms of GOO, regardless of the underlying etiology, are
bloating, anorexia, nausea and vomiting. Vomiting is usually nonbilious, and characteristically
contains undigested food particles. Patients with gastric outlet obstruction from a duodenal
ulcer or incomplete obstruction typically present with symptoms of gastric retention, including
early satiety, bloating or epigastric fullness, indigestion, anorexia, nausea, vomiting, epigastric
pain, and weight loss. They are frequently malnourished and dehydrated and have metabolic
insufficiency. Weight loss is frequent when the condition is chronic and is most significant in
patients with malignant disease. Abdominal pain is not frequent and usually relates to the
underlying cause, e.g. PUD, pancreatic cancer. The time of vomiting can suggest the site of
obstruction. In pyloric obstruction (more proximal) the time of vomitus is usually within the
first hour of eating, whereas in pyloric stenosis, or duodenal stenosis or obstruction (more distal)
the vomiting occurs after one hour because normally it takes 1 hour for the food to reach the
duodenum.Within 2-4 hours of eating the food is in the small intestine. Vomiting after 2-4
hours should raise suspicion against another cause.

79. A 65-year-old woman presents to your practice becauseshe has noted streaks of blood
on her stool on different occasions during the past week. This occasions were preceded by
a period of constipation. The only medication she is currently on is panadeine for a back
pain that started 3 weeks ago. She is otherwise healthy. Which one of the following would
be the most likely diagnosis?
A Ulcerative rectal cancer.
B Colorectal cancer metastasizing to the lumbar spine.
C Internal hemorrhoid.
D External hemorrhoid.
E Ulcerative colitis.
Option C is correct Blood on stool can be caused by several conditions, some being benign
and others malignant. Colorectal cancer, particularly if arsining from rectum can give blood
covering the stool. The blood may be bright or dark red, depending on the site of the tumor.
The darker the blood, the more proximal the tumor. Proximal colon cancers tend to present
with lethargy and fatigue rather than blood-stained stool because blood mixes with the stool
and is barely visible. Rectal bleeding may be the only symptom of colorectal cancer; however,
weight loss, abdominal pain or discomfort, bloating, anorexia, and other symptoms may be
seen. By the time a colorectal is clinically evident, it has often already metastasized to the liver
(the most common site) and lymph nodes in most of the time. Lungs and bones are rarely
involved several months after the tumor has metastasized to the liver and/or lymph nodes.
Internal hemorrhoids are one of the most common causes of benign rectal bleeding. Patients
may notice blood covering the stool, as streaks on the stool, dripping in the toilet bowl or
staining the toilet paper. The most common predisposing factor for development of an internal
hemorrhoid is chronic constipation. This woman has been on panadeine (paracetamol 500mg +
codeine 8 mg). This can justify the constipation and internal hemorrhoid as the most likely
cause of this presentation. Ulcerative rectal cancer can cause rectal bleeding (often painless),
but a rectal tumor so large to cause constipation is expected to be associated with more
pronounced presentation including weight loss, anemia, fatigue, or decreased stool cilber. As a
general rule external hemorrhoids do not bleed but are painful, so less likley to be the dianosis.
NOTE - Although internal hemorrhoid is the most likely diagnosis, colorectal cancer should be
excluded by thorough investigation.
80. Which one of the following is the most common cause of a hyperechoic mass on liver
ultrasonography?
A Hepatoma.
B Simple cyst.
C Hemangioma.
D Echinococcal cyst.
E Metastatic liver disease.
Option C is correct Hemangiomas, benign proliferation of vascular tissue, are the most
common cause of a hyperechoic liver mass on ultrasound. Hepatic hemangiomas (also known
as hepatic venous malformations) are benign non-neoplastic hypervascualr lesions. They are
frequently diagnosed as an incidental finding on imaging in asymptomatic patients. It is very
important to differentiate hemangiomas from hepatic neoplasms. On ultrasound, they typically
manifest as well-defined hyperechoic lesions; however, a small proportion (10%) are
hypoechoic, which may be due to a background of hepatic steatosis, where liver parenchyma
has increased echogenicity On CT scan, most hemangiomas are relatively well defined.
Features of typical lesions on three phasic CT scan include: Noncontrast: often hypoattenuating
relative to liver parenchyma Arterial phase: typically show discontinuous, nodular, peripheral
enhancement (small lesions may show uniform enhancement) portal venous phase: progressive
peripheral enhancement with more centripetal fill in Delayed phase: further irregular fill in and
therefore iso- or hyper-attenuating to liver parenchyma

81. A 35-year-old woman presents to your practice with complaint of right upper
quadrant discomfort for the past 3 months. She smokes 10 cigarettes a day and drink
alcohol at weekends. On examination, she is otherwise healthy, with no palpable
abdominal mass or tenderness. An abdominal CT scan is arranged and obtained, which is
shown in the accompanying photograph. Which one of the following is the most likely
diagnosis?
A
Liver abscess.
B Simple hepatic cyst.
C Hepatic hemangioma.
D Hepatocellular carcinoma.
E Hydatid cyst.
Option B is correct The homogenous hypoattenuating (darker than the surrounding liver
parenchyma) oval-shaped lesion in the photograph is characteristic of a simple hepatic cyst.
Simple hepatic cysts are common benign liver lesions and have no malignant potential. They
can be diagnosed on ultrasound, CT, or MRI. Simple hepatic cysts are one of the most common
liver lesions, occurring in approximately 2-7% of the population. It is slightly more common in
women. Hepatic cysts are typically discovered incidentally and are almost always
asymptomatic, unless they are large enough to cause symptoms (such as in this patient). Simple
hepatic cysts may be isolated or multiple and may vary from a few millimeters to several
centimeters in diameter. Simple hepatic cysts are benign developmental lesions that do not
communicate with the biliary tree. They can occur anywhere in the liver, but there may be a
greater predilection for the right lobe of the liver. Certain diseases are associated with multiple
hepatic cysts and include: Polycystic liver disease Autosomal dominant polycystic kidney
disease (ADPKD) - hepatic cysts may be seen in ~40% of those with ADPKD Von-Hippel-
Lindau disease Findings on ultrasonography include: Round or ovoid anechoic lesion (may be
lobulated) Well-marginated with a thin or imperceptible wall and a clearly defined back wall
May show posterior acoustic enhancement if large enough A few septa may be possible, but no
wall thickening is present A small amount of layering debris is possible No internal vascularity
on color Doppler On CT scan, a hepatic cysts is characterized by its homogenous
hypoattenuation (water attenuation). The wall is usually imperceptible, and the cyst is not
enhanced after intravenous administration of contrast material. (Option A) Liver abscess is
associated with fever, leukocytosis and more pronounced symptoms. They are solid and
hyperattenuated on CT scan. (Option C) hemangiomas have less homogenocity and well-
demarcation compared to hepatic simple cysts. (Option D) Hepatocellular carcinoma (HCC)
present with less demarcated hepatic lesions that are often hypoattenuated on CT and
hypoechoic on ultrasonography. The radiologic findings are inconsistent with HCC as a
possible diagnosis. (Oprion E) Hydatid cysts presents as a multiloculated cyst (daughter cysts
within the main cyst)

82. A 72-year-old man presents to the emergency department with complaint of perianal
pain for the past 2 days. His anal area is illustrated in the accompanying photograph.

Which one of the following is the most likely diagnosis?


A Thrombosed internal hemorrhoid.
B Thrombosed external hemorrhoid.
C Rectal carcinoma.
D Crohn’s disease of the anus.
E Perianal abscess.
Option A is correct Traditionally, hemorrhoids are classified as internal and external; however,
some authors believe that since these two have different origins and mechanisms of
development, they are better termed hemorrhoids (instead of internal hemorrhoids) and perianal
hematoma (instead of external haemorrhoids). Perianal hematoma and external hemorrhoids are
often interchangeably used. Hemorrhoid (internal hemorrhoid): The anus is mainly lined by
discontinuous masses of spongy vascular tissue termed anal cushions, which contribute to anal
closure and differentiating flatus from stool. Viewed from the lithotomy position, these
cushions are located at 3, 7, and 11 o’clock. These cushions are attached together and to the
surrounding structures by supporting fibromascular tissue. Hemorrhoids occur when these
structures become bulky and protruded due to gravity, straining, or increased tone of anal
sphincter (unlike the common belief, hemorrhoid is not a varicous anal vein). Bulky and
protruded cushion are at risk of trauma from hard stool and bright red bleeding from the
capillaries of the underlying lamina propria. Constipation and straining are the most common
causes of hemorrhoids; however, bowel habit is normal in many patients. Congestion from a
pelvic tumor, pregnancy, congestive heart failure, nephrosis, or portal hypertension plays a role
in only a minority of patients. Hemorrhoids are classified as following: 1 degree – remains in
the rectum. 2 degree – prolapses through the anus on defecation but it reduces spontaneously. 3
degree – like 2 degree but needs digital reduction. 4 degree – remains prolapsed persistently.
Hemorrhoids are painless unless acute thrombosis superimposes, in which case it may become
painful. Thrombosed hemorrhoids are managed conservatively with analgesics, stool softeners,
bed rest (elevation of the bed foot) and ice packs for 2-3 weeks. The drawback of this method is
the long time off work. Hemorrhoidectomy is the second option if conservative management is
not an option due to any cause. Perianal hematoma (external hemorrhoid) Perianal hematoma is
not hemorrhoid; however, it is usually called an external haemorrhoid. It presents as a painful
tense blue swelling at the anal verge caused by a recent thrombosis of an anal vein, often after
straining at stool. The picture in the question shows a fleshy red lesion protruding out of the
anus consistent with an (internal) hemorrhoid. Presence of pain suggests acute thrombosis.
(Option B) External hemorrhoid has a different appearance. (Option C) Although hemorrhoids
can be associated with rectal carcinoma, but the lesion itself is not a carcinoma. Moreover, a
carcinoma this large would have been associated with more pronounced systemic and local
symptoms. (Option D) It should always be remembered that for every anorectal lesion
thorough investigation should be conducted in an attempt to exclude serious underlying
pathologies such as inflammatory bowel disease or malignancies. (Option E) Perianal
abscesses present with painful red tender perianal swelling and induration not a lesion
protruding out of the anus.

83. A 40-year-old man presents to the emergency department after sudden onset of the
right calf pain and paralysis. The right dorsal pedis pulse is not perceptible. The limb
feels cold and is pale. The patient is given analgesics. After starting the patient on heparin,
which one of the following would be the most appropriate step in management?
A Abdominal CT.
B CT angiogram.
C Doppler Duplex ultrasound of the calf veins.
D Pelvic ultrasound.
E Echocardiography.
Option B is correct This patient has developed critical acute limb ischemia ‫ و‬requiring urgent
vascular surgery for revascularization and restoration of blood supply. Clinical signs of acute
arterial occlusion include (6 P's): Pain Paralysis Pulselessness Pallor Paresthesia Poikilothermia.
With any of the above signs or symptoms the patient is considered to have acute limb ischemia.
To confirm the diagnosis of acute arterial occlusion and the extent of obstruction, the patient
should have an urgent CT angiogram as the road map to the surgery. Magentic resonance
angiography with contrast is an alternative to CT angiography with about the same sensitivity
and specificitiy. (Option A) Abdominal CT scan is not required as this patient did not present
primarily with abdominal problems. (Options C and D) Pelvic ultrasound or Doppler Duplex
ultrasound of the calf veins adds nothing to the management strategy because firstly the
diagnosis is already made, and secondly these modalities do not provide adequate information
regarding the anatomical site of the occlusion. Furthermore, Doppler Duplex ultrasound is
highly operator-dependent. (Option E) Echocardiography is the investigation to consider once
acute phase of arterial occlusion has been managed. Thombi from the heart is a main source of
acute limb ischemia.

84. 16 A-79-year-old woman undergoes total hip replacement surgery. After four days,
she develops a swollen right leg. The circumference of the right calf is 4 cm greater than
that of the left leg. The right leg feels warm to touch and is slightly tender. Which one of
the following is most appropriate diagnostic tool to use for a diagnosis?
A Duplex Doppler ultrasound.
B X-ray. C Contrast venography.
D CT angiography.
E D-dime assay.
Option A is correct This patient has suspected diagnosis of deep vein thrombosis (DVT) and
requires urgent Doppler ultrasound to establish the diagnosis. Doppler ultrasound is more than
90% sensitive and more than 95% specific for thromboses within the femoral and popliteal
veins, but less accurate for iliac or calf vein thromboses. D-dimer is used for assessment of
patients with Well score of <2. A patient with symptoms and signs consistent with DVT should
be assessed and the clinical (pretest) probability of acute DVT established by using a validated
scoring system. If the clinical probability of DVT is low (<2), a D-dimer assay should be
performed; otherwise, the next step would be Doppler Duplex sonography. Modified Wells
score for predicting probability of deep vein Thrombosis check table page 2541 CLINICAL
CHARACTERISTIC SCORE Active cancer (treatment ongoing, administered within previous
6 months or palliative) 1 Pralysis, paresis or recent plaster immobilization of the lower
extremities 1 Recently bedridden >3days or major surgery within previous 12 weeks requiring
general or regional anesthesia 1 Localized tenderness along the distribution of the deep venous
system 1 Swelling of entire leg 1 Calf swelling >3cm larger than asymptomatic side (measured
10 cm below tibial tuberosity) 1 Pitting edema confined to the symptomatic leg 1 Collateral
superficial veins (nonvaricose) 1 Previously documented DVT 1 Alternative diagnosis at least
as likely as DVT -2 A score of ≥2 indicates that the probability of DVT is likely; a score of <2
indicates that the probability of DVT is unlikely In patients who have symptoms in both legs,
the more symptomatic leg is used This patient has been bedridden for more than 3 days (1
point), has localized tenderness (1 point) and the circumference of the affected leg is more than
3 cm greater than the unaffected leg (1 point). With a Wells score or 3, this patient has high
probability diagnosis of DVT and needs Doppler ultrasonography of the leg as the next step. If
the Wells score was <2 the next best step would have been D-dimer assay (option E). A
negative D-dimer test excludes DVT (and pulmonary embolism), but positive values is only
suggestive of DVT and further investigations would be needed. Previously, contrast
venography (option C) was the definitive test for the diagnosis of DVT but has been largely
replaced by ultrasonography, which is noninvasive, more readily available; however, it still
remains the gold-standard for diagnosis of DVT (not in practice). While a Doppler ultrasound
can establish the diagnosis, there is no need for CT angiography (option D) unless accurated
anatomical site of the thrombus is required for surgical interventin (very rarely indicated).
85. An 82-year-old man presents to your office complaining of an ulcer over the medial
side of his leg just above the internal malleolus. He mentions that the wound has been
there for the past 6 months, and worsened recently. The examination confirms the
diagnosis a venous ulcer. The ipsilateral ankle-brachial index is 0.7. Which one of the
following is correct regarding the management of this patient?
A Compression stockings are unsafe for this patient.
B Prescribe compression stockings and refer him to a vascular surgeon.
C Give antibiotics and review in two weeks.
D Check fasting glucose.
E Check HbA1C.
Option A is correct Venous leg ulcers are the most common chronic wounds seen in general
practice. They account for 70-90% of chronic wounds. They tend to develop over the medial
distal leg (just above the internal malleolus), and are usually painful. The most important risk
factor for developmet of a venous ulcer is previous deep vein thrombosis (DVT) with
consequent damage to venous structure and insufficiency. Most venous ulcers respond
adequately to a conservative management called Bisggard regimen. This regimen has four
components: 1. Patient education 2. Elevation of the foot 3. Elastic compression 4. Evaluation
Elastic compression is considered for very patient unless: ABI is below 0.8 The patient is
diabetic In the above situations, compression stocking causes more compromised arterial flow
and makes the limb ischemia worse; therefore, specialist advice and close monitoring is
strongly recommended. Antibiotics are not indicated in uncomplicated venous ulcers. Diabetes
is one of the most important predisposing factors in development of arterial ulcers. Although
fasting glucose level may be considered for screening of diabetes, it is not a step in
management here. HbA1C is used to monitor efficacy of or adherence to anti-hyperglycemic
regimens, which is not the case here.

86. A 73-year-old man presents with swelling of his left leg for the past three hours.
Evaluation establishes the diagnosis of a deep vein thrombosis (DVT) in the left lower leg.
A Doppler ultrasound reveals deep vein thrombosis in the calf as well as popliteal and
femoral veins. His past medical history is negative for previous DVT. Which one of the
following is the next best in management of this patient?
A Angiography.
B Angioplasty.
C Heparin.
D Warfarin.
E Aspirin.
Option C is correct All patients with DVT are initially managed by injectable heparin (either
intravenous unfractionated or subcutaneous low molecular weight heparin). Warfarin can be
started within 24-48 hours with close monitoring of INR. Once INR is in the therapeutic range
of 2-3, heparin can be withheld. Warfarin should be continued for 6 months at least. Patients
should see their GP on a regular basis for close monitoring of the INR. (Option A) DVT is the
presence of clot in the deep veins. Arteriography evaluates arterial structure and thromboses
within arteries. (Option B) Angioplasty is performed for revascularizaion in arterial problmes
with no role in DVT. (Option D) Warfarin is indicated in this patient after initial
anticoagulation with heparin. (Option E) Antiplatelet therapy with aspirin or clopidogrel does
not provide adequate anticoagulation for prevention of DVT.
87. Five days after surgical resection of a colonic tumor, a 73-year-old man develops pain,
swelling and tenderness in his right calf. Physical examination reveals swelling of the
ankle and the calf and tenderness over the gastrocnemius muscle. Which one of the
following is the most appropraite diagnostic tool to establish the diagnosis?
A CT angiography of the leg.
B MRI of the leg.
C Conventional venography of the leg.
D Serum D-dimer.
E Duplex Doppler ultrasonography.
Option E is correct With the clinical picture suggestive of deep vein thrombosis, Duplex
Doppler ultrasonography should be used to confirm the diagnosis. Although pain, swelling, and
tenderness are suggestive of DVT, but treatment should never be started solely on clinical basis.
(Option A) CT angiography is used for visualization of clots in arteries not veins. (Option B)
MRI is being used increasingly for assessment of DVT, and probably is the preferred
investigation for suspected caval and iliac venous thrombosis, particularly in pregnancy, but is
expensive, time-consuming and not always available. (Option C) Conventional venography is
the criterion standard for diagnosis of DVT, but is rarely used now because less invasive and
convenient diagnostic modalities are widely available. (Option D) D-dimer assay, although
sensitive, lacks specificity. Increased levels of D-dimer can be seen in several conditions, with
post-surgery state as one. It is basically used to rule out DVT or pulmonary embolism in low
risk patients (Wells score<2). A positive test demands more specified evaluation, but a negative
one excludes the possibility of DVT or PE.

88. you are assessing the risk of thromboembolism in a 75-year-old woman, who is booked
for elective surgical replacement of her left knee joint. Which one of the following, if
present in history, is the most significant risk factor for thromboembolism during her
peri-operative stay?
A History of colon resection due to toxic megacolon.
B Irritable bowel syndrome.
C Stage II kidney disease.
D Transient ischemic attack with full recovery three years ago.
E Heart failure.
Option E is correct Peri-hospital risk assessment of a venous thromboembolic events (VTE)
such as deep vein thrombosis or pulmonary embolism is stratified according to the following
table: Stratification of deep vein thrombosis and/or pulmonary embolism during hospital stay
Check table on page2557 *major surgery: any intra-abdominal operation and all other
operations lasting more than 45 minutes ** infectious diseases, varicose veins, obesity or
general immobility ***deficiency of antithrombin, protein C, protein S, Factor V Leiden
mutation, hyperhomocysteinemia, and prothrombin 20210A Among the options, congestive
heart failure and the surgery are of the highest risk for venous thromboembolism. None of the
other options are risk factors for VTE.

89. A 56-year-old man comes to your practice with an eight-month history of left leg pain,
which is brought on by walking, and used to relieved with rest. Recently, he gets the pain
even at rest, particularly at night. He has to dangle his leg for relative pain relief. He has
smoked for the past 30 years and been on hydrochlorothiazide for the past 10 years. On
examination, he is an overweight man with a BMI of 29, blood pressure of 145/100mmHg
and pulse of 98bpm. His left leg is hairless with shiny and hyper-pigmented skin. The left
femoral pulse is barely perceptible and the right dorsal pedis pulse cannot be felt. He has
an ankle-brachial index (ABI) of 0.4 on the left side. Which one of the following is the
most appropriate management?
A Advise him to stop smoking, do more exercise and review in three months.
B Advise him to stop smoking, do more exercise and start him on statin.
C Arrange Duplex Doppler ultrasonography and review.
D Urgent referral for vascular surgery.
E Arrange for arteriography and review.
Option D is correct The clinical picture is consistent with chronic peripheral artery disease
(PAD) and limb ischemia suggested by calf claudication and the presence of risk factors
(hypertension, smoking and obesity). The rest pain and ABI of 0.4 suggests a critical ischemia,
for which surgical intervention is required to be planned; otherwise he will end up gangrene
and amputation. This patient should be urgently referred to a vascular surgeon for assessment
and treatment. (Options A and B) Smoking cessation is the most appropriate advice that
should be given to patients with PAD. It increases the exercise tolerance and walking distant
and slows progression to more advanced disease. Statins should be started for patients with
PAD as well as those with coronary artery disease, diabetes mellitus, aortic diseases due to
atherosclerosis, and carotid artery disease. Pentoxifyline (not an option) might have a role, but
trials have shown moderate benefit with this drug. (Option C) Duplex Doppler
ultrasonography has already been used for determination of the ABI. It is unable to locate the
anatomy of the lesion and a road map for surgery. Duplex Doppler ultrasonography is used to
assess the blood flow to the leg and calculation of ABI, and the presence of a clot in the venous
system when DVT is suspected. It cannot accurately spot the site of occlusion in an artery.
(Option E) Once surgical intervention is considered by a surgeon, the presence and site of
occlusion should be indentified and documented prior to the surgery. This is achieved by either
less invasive CT angiography or MRI angiography, or through more invasive procedure of
conventional arteriography.

90. A 60-year-old man presents with leg pain for the past 6 months. The pain becomes
worse with walking and is relieved when he rests. There is no pain at rest. He has smoked
20 cigarettes per day for the past 30 years. On examination, he is obese with a BMI of 31
and has a blood pressure of 160/110mmHg on two readings 20 minutes apart. Distal
pulses of the left lower limb, including dorsal pedis, are barely perceptible. The skin of
the legs is shiny and hairless. Mild muscular atrophy of the leg is noted. Ankle-brachial
index (ABI) is 0.7. Which one of the following would be the most appropriate
management and advice?
A Smoking cessation, exercise and follow-up in three months.
B Smoking cessation, statins and ACE inhibitors.
C Duplex Doppler venous ultrasonography.
D Aspirin, metoprolol and statin.
E Referral for vascular surgery.
Option B is correct With intermittent calf claudication, presence of the risk factors and an ABI
of 0.7, the diagnosis of chronic peripheral arterial disease is almost established. Management of
peripheral arterial disease includes the following: Smoking cessation - smoking is the most
important predisposing factor for peripheral arterial disease (PAD). Smoking cesation alone is
associated with an improvement in the distance of pain-free walk, doubled 5-year survival and
better post-op outcomes. Exercise – exercise on an as tolerated basis, improves the pain-free
walking distance and time and should be advised for all patients. ACE inhibitors – evidence
suggests that ACE inhibitors may improve walking ability in patients with intermittent
claudication. The ACE inhibitor with greatest evidence of benefit is ramipril. It is unknown if
the improvement in walking distance associated with ramipril is due to a class effect of ACE
inhibitors or whether it is specific to this medicine. The ABI does not seem to improved though.
Statins – statins improve revascularisation, pain-free walking distance and survival. Of all
lipid-lowering agents, only statins have been proved to lower the mortality in patients with
vascular diseases due to atherosclerotic changes. It should be started for patients with coronary
artery disease, PAD, aortic disease (e.g. abdominal aortic aneurysm), carotid artery disease and
diabetes mellitus. Clopidogrel and aspirin – they are often prescribed to reduce the overall risk
of myocardial infarction and stroke, but are not associated with improvement of PAD
symptoms. Beta blockers (e.g. metoprolol) are not indicated in the absence of cardiac disease.
They have no effect on PAD. More detailed management of PAD is highly dependent on the
severity of the disease. ABI is the mostly used predictor for this purpose. ABI is interpreted as
follow: check table on page 2563 Among the given option, smoking cessation, statins, and
ACE inhibitors are the best possible management and the advice for the patient.

92. A 76-year-old man comes to your office for evaluation. He mentions that he has
difficulty walking because of the left leg pain. The pain is brought on after walking two
blocks and gets better when he stops to rest. On examination, the leg skin is shiny and
dark. The legs hair is lost and the muscles are atrophied. Distal pulses are difficult to
palpate. Which one of the following is the most likely diagnosis?
A Acute limb ischemia.
B Deep vein thrombosis.
C Superficial vein thrombosis.
D Chronic obstructive arterial disease.
E Spinal canal stenosis.
Option D is correct Leg pain brought up by walking and relieved by rest and weak or absent
distal pulses are characteristic of chronic limb ischemias as a result of chronic obstructive
arterial disease. Atrophied muscles and shiny hairless skin supports the diagnosis. The clinical
findings in chronic limb ischemia include: Weak or absent distal pulses – the hallmark finding
Shiny and hyperpigmented skin Hair loss and leg ulcers Thickened nails Muscular atrophy
Vascular bruits (Option A) Acute limb ischemia presents with sudden onset pain, pallor,
paralysis, paresthesia, pulselessness and poikilothermia. This patient has features of chronic
limb ischemia. (Option B) Leg pain due to deep vein thrombosis can be brought on by walking
and relieved by rest (similar to chronic limb ischemia), but other features such as sparse leg
hair, pigmentation, muscle atrophy, etc are not feautres of DVT. DVT presents with leg pain
and tenderness, swelling and warmth. (Option C) Superficial thrombophlebitis presents with
pain, erythema, induration and tenderness along the course of a superficial vein. (Option E)
Leg pain caused by neurogenic claudication due to spinal canal stenosis tends to get worse with
erect posture and relieved by recumbency. Absence of neurological deficits makes this
diagnosis less likely.
92. A 65-year-old man comes to your practice with complaint of pain in buttock and leg.
He has the pain for the past three months, and describes it as cramping felt in the left
buttock. The pain is only present when he starts walking and comes on after five minutes
or so every time. It radiates to the left thigh and goes away few minutes after he stops
walking. Which one of the following is the most likely diagnosis?
A Neurogenic claudication resulting from spinal canal stenosis.
B Stenosis of his left common iliac artery.
C Stenosis of the left superficial femoral artery.
D Osteoarthritis of the right hip joint.
E L4/L5 disc prolapse.
Option B is correct The nature of the pain (cramping) and its pattern (brought on by walking,
relieved by rest) is characteristic of chronic limb ischemia. Exertional buttock pain that is
ischemic in nature (buttock claudication) is due to obstruction of either common iliac or
external iliac artery. Stenosis of superficial femoral artery leads to ischemic pain of lower leg
on walking (calf claudication), not the buttock. (Option A) Spinal canal stenosis usually affects
the mid or lower lumbar spine and can cause nerve root impingement presentation, which is
pain brought on by standing and relieved by recumbency. Pain distribution is related to the
nerve root distribution rather than muscle groups supplied by an involved artery. (Option C)
Stenosis of the superficial femoral artery gives rise to leg rather than buttock claudication.
(Option D) Osteoarthritis of the hip is associated with pain on activity and relif by rest. The
pain is felt in the hip and can radiat to the knee. Buttock pain is not common. (Option E)
L4/L5 disc prolapse can result in unilateral nerve root entrapment leading to neurological
findings, not necessarily related to walking. The pain of L4/L5 disc prolapse is worse on lifting
heavy objects, coughing, sneezing or straining at stool. Numbness and parsthesia are features
that are commonly present in nerve root impingement, but not in chronic limb ischemia.
93. A 55-year-old man is on heparin after he developed an acute limb ischemia. Which
one of the following tests is used for monitoring the response to anticoagulation therapy
with heparin?
A Fibrinogen level.
B Prothrombin time (PT).
C INR.
D Activated partial thromboplastin time (aPTT).
E Bleeding time (BT).
Option D is correct Activated partial thromboplastin time (aPTT) is used to assess the
integrity of the following coagulation pathways: Intrinsic pathway that includes prekallikrein,
high molecular weight kininogen and factors VIII, IX, XI, and XII. Common pathway
including factors II, V, X, and fibrinogen. aPTT is also used for monitoring the effects of
maintenance therapy with heparin. Usually the goal of heparin maintenance therapy is to keep
aPTT level 1.5- 2.5 times above patient’s aPTT baseline. When starting heparin therapy, aPTT
is measured 6-hourly until it is within therapeutic range of 65-100 seconds, and 24 hourly
afterwards. INR is used for monitoring warfarin therapy. It assesses extrinsic coagulation
pathway, as does PT. Normal range of PT in an individual not on blood thinners is 11-13.5
seconds. PT and INR are prolonged if the patient is on warfarin, or deficient in factors II
(prothrombin), III, IX, X, or fibrinogen, or has vitamin K deficiency. BT is used as a platelet
function test with a normal range of 1-9 minutes. It is prolonged if the patient is on antiplatelet
medications (aspirin, clopidogrel), has thrombocytopenia, or in those with platelet aggregation
disorders e.g. Von-Willebrand disease. BT is not used routinely anymore.

94. A 55-year-old man presents to the emergency department after sudden onset of pain
and paresthesia in his left leg. On examination, the distal pulses of the left lower limb are
absent and the limb is cold and painful. A CT angiogram is performed and establishes the
presence of femoral artery embolism. The patient was started on intravenous heparin and
embolectomy was carried out. Which one of the following is the next best step in
management?
A Aspirin and warfarin for 6 months.
B Lifelong aspirin.
C Warfarin for 3-6 months.
D Low molecular weight heparin for six months and lifelong warfarin afterwards.
E Heparin for one week.
Option C is correct Every patient with acute limb ischemia must be started on anticoagulation
therapy with warfarin after embolectomy is performed. Heparin cover should not be withheld
until the INR is in the therapeutic range of 2-3. Long-term warfarin should be considered
following embolism from a cardiac source; otherwise 3-6 months warfarin may suffice. Aspirin
is indicated in patients with atherosclerotic changes of the peripheral arteries and chronic limb
ischemia. It has no role in management of acute limb ischemia. Combination of aspirin and
warfarin is not shown to benefit the patient more than warfarin alone in terms of preventing
further thromboembolic events. Furthermore using these two drugs in combination increases
the risk of bleeding. Heparin is only used until the INR is in the therapeutic range and withheld
afterwards. This goal is usually achieved in 2-3 days.
95.A 70-year-old man presents with acute pain and paralysis of the right leg diagnosed to
have been caused by acute leg ischemia. Heparin is started immediately. After emergency
imaging, he is transferred to the operating room for embolectomy. Surgical intervention
successfully restored blood supply to the affected limb after 3 hours. This patient is at risk
of developing reperfusion injury as a result of prolonged ischemia. Which one of the
following is not a characteristic feature of reperfusion injury?
A Hyperkalemia.
B Hypokalemia.
C Metabolic acidosis.
D Myoglobinuria.
E Elevate creatine kinase.
Option B is correct Reperfusion injury is a complication of blood restoration to a limb, which
has been ischemic for a while. Characteristic features of reperfusion injury are all related to
ischemia and its impacts on tissue (especially muscle cells). Features of reperfsuion syndrome
include: Metabolic acidosis (lactic acidosis) Elevated creatine kinase Hyperkalemia
Myoglobinemia and myoglobinuria These findings are caused by hypoxemia resulting in
metabolic acidosis, and muscle cell breakdown and release of its cell into the blood.
Hypokalemia is not a characteristic feature of reperfusion injury. The extent of the reperfusion
injury depends on the following: Duration and the site of arterial blockage The extent of
collateral flow to the affected area The previous health of the affected limb Approximately one
third of all deaths from arterial occlusions are due to metabolic complications after
revascularisation.
96. A 65-year-old man presents to the emergency department with sudden onset right-
sided leg pain and paresthesia. On physical examination, the distal pulses of the right leg
are absent and the limb is cold. A CT angiography is arranged showing thrombotic
occlusion of the right femoral artery. Which one of the following is the next best step in
management?
A Intravenous infusion of heparin and emergency embolectomy.
B Intravenous infusion of heparin for 24 hours and review.
C Complete bed rest with compression bandage and leg elevation.
D Start the patient on warfarin.
E Start the patient on aspirin and clopidogrel.
Option A is correct The clinical and imaging findings are quite consistent with acute limb
ischemia. Paralysis, paresthesia and compartment syndrome are ominous signs that demand
emergency surgical intervention after heparin has been started. The golden time for surgical
intervention is 4 hours. Signs of ischemiaare are reversible if prompt action is taken within this
period. Prolonged acute ischemia(>6 hours) leaves irreversible and permanent deficits. Heparin
cover should be maintained and warfarin started. Heparin then can be safely withheld once the
INR is 2-3. Warfarin alone is not recommended as it is pro-coagulation at the beginning.

97. Which one of the following signs mandates emergency surgical intervention in acute
limb ischemia?
A Pallor.
B Coldness.
C Paralysis.
D Pain at rest.
E Pulselessness.
Option C is correct Paralysis is present when a patient with acute limb ischemia is unable to
wriggle the toes or fingers. It is the most reliable sign indicating acute limb ischemia requiring
emergency surgical intervention. Paresthesia of the affected limb is another indicator of acute
limb ischemia and can be used in the absence of paralysis. (Option A) Pallor is among the
signs suggesting acute limb ischemia; however, the color of the limb may be affected by the
ambient light. (Option B) Coldness of the limb is the least reliable sign as it can be due to
decreased body temperature of the patient to cold weather. (Option D) Pain (either with
exertion or at rest) and pallor can be caused by chronic ischemia as well, and are not reliable
indicators of the need for emergency surgical treatment. (Option E) Pulselessness can be a sign
of acute limb ischemia, but paralysis remains the most critical indicator.

98. Which one of the following is the most common cause of acute limb ischemia?
A Congestive heart failure.
B Thrombosis.
C Vasculitis.
D Embolism.
E Smoking.
Option B is correct Thrombotic occlusion is the most common cause of acute lower limb
ischemia accounting for more than 80% of cases. (Option D) Occlusion from embolus is not as
common, partly due to decline in rheumatic heart disease and prompt management of patients
with atrial fibrillation with prophylactic anticoagulation. Atrial fibrillation accounts for two-
thirds of acute limb ischemias due to embolism. (Options A, C and E) Congestive heart failure,
vasculitis and smoking are less common risk factors of acute limb ischemia. 97Question ID
260 Title Mortality rate associated with ruptured aortic aneurysm Last revision 2016-10-16
11:36:44 You are called to see a 58-year-old patient, who has presented with complaints of
weakness and shortness of breath. A blood test reveals hemoglobin of 85g/L (120-160). As a
part of the work up for finding the cause of his anemia, an abdominal CT scan is performed
showing a 10.5cm infra-renal abdominal aortic aneurysm as an incidental finding. You tell the
patient that he needs surgery for treatment of his aneurysm. He wants to know how dangerous
it could be if he does not have the surgery. Which one of the following is the mortality rate of a
ruptured abdominal aortic aneurysm? A 10%. B 25%. C 50%. D 80%. E 100%. Option D is
correct A ruptured AAA is associated with an overall mortality rate of 80-90%.

99. A 72-year-old man comes to the emergency department complaining of epigastric pain
that radiates to the lower back. While waiting for further evaluation in the emergency
department, he suddenly collapses. Physical examination shows a blood pressure of 80/50
mmHg and pulse of 110 bpm. He is pale, cold and sweaty. Which one of the following is
the next best step in management?
A Take the patient to the operating theatre immediately.
B Arrange for CT angiography.
C Non-contrast abdominal CT scan.
D ECG.
E Bedside ultrasonography.
Option E is correct The location of the pain and its radiation to the lower back, followed by
signs of shock is consistent with a ruptured abdominal aortic aneurysm as the most likely
provisional diagnosis. Before emergent transfer of the patient for surgery, vascular surgeon
should be contacted and the diagnosis should be confirmed with ultrasonography. In the
meanwhile, resuscitative measures such as intravenous access, fluid administration, and
supplemental oxygen should be taken in the emergency department. An ECG must be obtained
pre-operatively.

100. Which one of the following is not an indication for surgical repair of an abdominal
aortic aneurysm (AAA)?
A AAA more than 5.0cm in a male patient.
B AAA more than 5.0cm in a female patient.
C AAA growth more than 1.0cm/year.
D AAA with back pain.
E AAA with distal thromboembolism.
Option A is correct The following are indications for surgical repair of an aortic
aneurysm: 1.Asymptomatic abdominal aortic aneurysm greater than over 5.5cm in men
2.Asymptomatic abdominal aortic aneurysm over 5.0cm in women 3.Thoracic aortic aneurysm
over 6.0cm 4.Growth of more than 1.0cm/year 5.Symptomatic abdominal aortic aneurysm
(abdominal/back pain, distal thromboembolism) An asymptomatic aortic aneurysm of 5 cm in a
male is not an indication for surgical repair.
101. Abdominal ultrasonography of a 68-year-old man for renal colic reveals an
abdominal aortic aneurysm 4.5cm in size as a chance finding. Which one of the following
is the most appropriate management of his aneurysm?
A Refer him for elective surgical repair of the aneurysm.
B Repeat sonography in 6 months.
C Repeat sonography in 12 months.
D Repeat sonography in 24 months.
E Arrange for CT angiography.
Option C is correct Ultrasonography is the preferred method for screening and surveillance of
asymptomatic AAA. Recommended screening intervals for an asymptomatic AAA depends on
the size of the aneurysm and is according to the following table: AAA diameter (cm)
Surveillance interval (months) 3-3.9 24 4-4.5
12 4.6-5 6 >5 3 Given the
size of the AAA (4.5 cm), it should be monitored every 12 months with ultrasonography.

102. A 65-year-old man presents with history of two episodes of abdominal pain after
meals. As a part of workup plan, an abdominal ultrasonography is performed, which
showed gallstones in the gallbladder and a 3.8 cm abdominal aortic aneurysm (AAA).
Which one of the following is the recommended management of this asymptomatic AAA?
A Ultrasonography every 12 months.
B CT scan every 12 months.
C Ultrasonography every 24 months.
D CT scan every 24 months.
E CT scan or ultrasonography every 12 months.
Option C is correct Ultrasonography is the preferred method for screening and surveillance of
asymptomatic AAA. The screening intervals depend on the size of the AAA and are according
to the following table: AAA diameter (cm) Surveillance interval
(months) 3-3.9 24 4-4.5
12 4.6-5 6 >5
3 Given the size of the AAA (3.8 cm), it should be monitored every 24 months
with ultrasonography

103. A 55-year-old man presents with concerns about developing abdominal aortic
aneurysm (AAA), as his father was diagnosed with the same disease at the age of 57 years.
He has smoked 20 cigarettes a day for the past 15 years. On examination, he has a blood
pressure of 130/75mmHg. Which one of the following is the best advice to give?
A Reassure as AAA has no genetic predisposition.
B Arrange for regular screening with ultrasonography. C
Do a CT angiogram.
D Start him on aspirin.
E Start him on anti-hypertensive medications.
Option B is correct Patients with a first- degree relative with AAA are have a 20% increased
risk of developing AAA compared to normal population; for this reason, he has to be screened
for AAA. Screening is performed with regular ultrasonography as the best investigation with
very high index of sensitivity and specificity. NOTE - Advancing age is the most common risk
factor for AAA. Other risk factors include male gender, family history, smoking,
hypercholesterolemia, diabetes mellitus and chronic hypertension. (Option A) Reassurance is
inappropriate because he has increased risk of AAA due to the family history. (Option C) CT
angiogram is indicated once the surgical repair is planned. (Option D) Aspirin is used neither
for prophylaxis, nor treatment of AAA. (Option E) Anti-hypertensive medications are of no
benefit because the patient is normotensive.

104. A 55-year-old man with history of smoking and hypertension has been diagnosed
with a 4.2cm abdominal aortic aneurysm (AAA). Which one of the following is the
investigation of choice for monitoring and surveillance of his AAA?
A MRI angiography.
B CT angiography.
C Ultrasonography.
D Digital subtraction angiography.
E Non-contrast abdominal CT scan.
Option C is correct Ultrasonography is the most accurate and the best investigation for
diagnosing, screening and surveillance of aortic abdominal aneurysms. Ultrasonography is a
noninvasive, readily available test with about 100% sensitivity and specificity. CT angiography
is performed if surgical correction is planned, once the diagnosis is made by ultrasonography.

105. A 60-year-old man presents to the emergency department with complaint of vague
abdominal pain. On examination, there is mild abdominal tenderness in the epigastric
area with a vaguely palpated abdominal mass. On auscultation, a bruit is heard over the
mass. Which one of the following is the next step in management?
A Aortogram.
B Emergency bedside ultrasonography.
C Non-contrast abdominal CT scan.
D Erect abdominal X-ray.
E Immediate transfer of the patient to the theatre.
Option B is correct The findings are consistent with provisional diagnosis of abdominal aortic
aneurysm (AAA). Physical examination is only moderately sensitive for diagnosis, therefore a
bedside ultrasonography for confirming the diagnosis of AAA is the next best step in
management. Ultrasonography is the preferred diagnostic method for AAAs with high accuracy.
Spiral CT scan gives visualisation of the aneurysm and surrounding structures. It is only
indicated prior to elective surgical repair. It is never considered for diagnosis. CT angiography
with intravenous contrast can be used for assessment, but non-contrast CT and abdominal X-
rays are never part of the diagnostic plan.

106. Which one of the following is the most important diagnostic feature of achalasia? A
Dysphagia for solids.
B Dysphagia for liquids.
C Dysphagia for both solids and liquids.
D Regurgitation.
E Weight loss.
Option C is correct Achalasia is caused by lack of a relaxation of the lower esophageal
sphincter and aperistalsis of the esophageal body. The condition is characterized by
degeneration of the nerves innervating the esophagus of unknown cause. Symptoms of
achalasia include the following: Dysphagia – the most common symptom: like other motility
disorders of the esophagus there is dysphagia for both liquids and solids at the same time.
Regurgitation – regurgitation of achalasia is nocturnal, but that of retropharyngeal pouch
(Zenker’s diverticulum) occurs even when the patient is standing. Chest pain Heart burn
Weight loss – weight loss can occur with all types of dysphagia Of the symptoms, dysphagia to
both liquids and solids is the cardinal and the most common symptom of achalasia. (Option A)
Dysphagia for solids as the prominent feature is characteristic of conditions that cause a
narrowing within the lumen. (Option B) Dysphagia for liquids as the cardinal symptom is seen
in oropharyngeal dysphagia such as in Zenker’s diverticulum, Parkinsonism, stroke and
myasthenia gravis. (Option D) Regurgitation in recumbency is also a feature of achalasia but it
is not as common as dysphagia (~40% vs. 80%). It seen otther conditions such as
retropharyngeal pouch. (Option E) Weight loss can be a manifestation of all esophageal
diseases. It is not a exclusive feature of achalasia.

107. Which one of the following conditions is not associate with dysphagia?
A Achalasia.
B Parkinson's disease.
C Esophageal varices.
D Esophageal cancer.
E Esophageal stricture.
Option C is correct
Dysphagia means difficulty in swallowing and can be classified as three types: Oropharyngeal
dysphagia – difficulty in passing liquids more than solids due to neurologic or muscular
problems such as: Parkinsonism Myasthenia gravis – the upper one-third of the oesophagus has
striated muscle. If myasthenia gravis affects this part, dysphagia ensues. Prolonged intubation
Zenker’s diverticulum Esophageal dysphagia – difficulty in passing solids more than liquids
due to local obstruction in the following conditions: Strictures Schatzki ring Webs Carcinoma.
Motility-related dysphagia – difficulty in passing both solids and liquids at the same time, seen
in conditions such as: Achalasia Scleroderma At occasions, there is odynophagia that is painful
swallowing. As a matter of fact, in odynophagia the difficulty in swallowing is caused by the
pain, not buy structural abnormalities. Monilial (candida) esophagitis and pill esophagitis are
two examples of conditions leading to dynophagia. Of the given options, the only condition
which is not associated with dysphagia (or odynophagia) is esophageal varices.

108. Which one of the following is the most common type of esophageal malignancy in
Australia?
A Squamous cell carcinoma.
B Lymphoma of the esophagus.
C Adenocarcinoma.
D Sarcoma.
E Metastatic tumors.
Option C is correct The epidemiology of esophageal carcinoma has markedly changed over
the past several decades. Previously, squamous cell carcinoma was the most common type of
esophageal malignancies worldwide (90-95% of cases); however, over the past 4 decades and
in developed countries such as United States of America, UK and Australia, the incidence of
adenocarcinoma of the distal esophagus and gastro-esophageal junction has progressively
increased. Currently, adenocarcinomas account for more than 70% of all cases of esophageal
cancers in Australia. Barrett’s esophagus caused by gastroesophageal reflux disease (GERD) is
the most significant risk factor for development of esophageal carcinoma of adenocarcinoma
type. NOTE - Squamous cell carcinoma remains the leading type of esophageal carcinoma
worldwide with smoking and chronic alcohol as the most significant risk factors.

109. Which one of the following is the most common type of esophageal cancer worldwide?
A Adenocarcinoma.
B Squamous cell carcinoma.
C Lymphoma.
D Metastatic lesions.
E Sarcoma.
Option B is correct The epidemiology of esophageal carcinoma has markedly changed over
the past several decades. Worldwide, squamous cell carcinoma is the most common type of
esophageal malignancies. Smoking and alcohol are the most significant risk factors for this
type of esophageal cancer. However, in developed countries such as United States of America,
UK and Australia, the incidence of adenocarcinoma of the distal esophagus and gastro-
esophageal junction has progressively increased and accounts for over 70% of all cases of
esophageal cancer. gastroesophageal reflusx disease (GERD) can result in Barrett’s esophagus
which is the most significant risk factor for adenocarcinoma of the esophagus.

110. Which one of the following factors has the greatest impact on development of
esophageal adenocarcinoma?
A Achalasia.
B Barrett’s esophagus.
C Low-fiber diet.
D Smoking.
E Gastro-esophageal reflux disease.
Option B is correct Barrett's esophagus is the most significant risk factor for development of
esophageal adenocarcinoma. (Option A) Achalasia has been associated with esophageal cancer
but is not as significant as other factors such as smoking and alcohol in squamous cell
carcinoma and Barret's esophagus in adenocarcinoma of the esophagus. (Option C) Although
implicated in some studies, low-fiber diet is not as significant as other risk factors for
esophageal cancer. (Option D) Smoking and chronic alcohol ingestion are the most common
and important risk factors for development of esophageal squamous cell carcinoma. (Option E)
Gastro-esophageal reflux disease is not a risk factor for esophageal adenocarcinoma unless it
results in Barrett’s esophagus (indirect effect). NOTE - Obesity is considered another
important risk factor, mostly because it predisposes to reflux disease. However, the outcome of
these conditions, namely the Barrett’s oesophagus, remains the main and most significant risk
factor through producing metaplastic changes that can progress to dysplasia and eventually
cancer

111. A 71-year-old man presents with progressive jaundice, pale stool and dark urine. On
examination, a mass is palpated in the right upper quadrant that moves with respiration
Ultrasonography shows a dilated common bile duct (CBD) and no masses in the head of
the pancreas. Which one of the following could be the most likely diagnosis?
A Chronic pancreatitis.
B Carcinoma of the tail of the pancreas.
C Peri-ampullary tumor.
D Biliary cirrhosis.
E Budd-Chiari syndrome.
Option C is correct The clinical picture is highly suggestive of common bile duct obstruction
(CBD). CBD obstruction presents with obstructive jaundice and often a palpable mass
(distended gallbladder) in the right upper quadrant that can be tender or non-tender depending
on the underlying etiology. The following are the most common causes of CBD obstruction:
Stones – the most common cause Strictures Periampullary tumors – these tumors arise within
2cm of the ampula of Vater in the duodenum and include the following: Pancreatic head /
uncinate process tumors: includes pancreatic ductal adenocarcinoma involving head and
uncinate process of the pancreas Lower common bile duct tumors: includes types of
cholangiocarcinoma involving the intrapancreatic distal bile duct Ampullary tumors: those
originating from the ampula of Vater Periampullary duodenal carcinoma The pancreatic head
tumors are the most common periampullary tumors, but not in this case, as sonography shows
that the pancreatic head is clear; therefore, other types of periampullary tumors should be
considered as the most likely diagnosis. (Option A) Chronic pancreatitis presents with
abdominal pain, malabsorption and diarrhea. Obstructive jaundice is not a presentation.
(Option B) Anatomically, tumors of pancreatic tail are far from biliary system and do not
cause biliary obstruction. (Option D) Biliary cirrhosis does not cause CBD obstruction.
(Option E) Budd-Chiari syndrome is thrombotic occlusion of hepatic veins, presenting with a
different clinical picture.

112. 35 years old, presents to the emergency department with pain and swelling of his left
thigh since this morning. Investigations establish the diagnosis of deep venous thrombosis
for which he is started on heparin in hospital. He has diabetes and hypertension and his
wife mentions that is very busy and distracted and always forgets to take the drugs he is
prescribed for treatment of his hypertension and diabetes. Which one of the following
options would be the most appropriate management for him after the course of heparin is
completed?
A No more treatment is needed.
B Warfarin for 6 months.
C Aspirin for 6 months.
D Surgical intervention.
E Caval filter.
Option E is correct Treatment of DVT starts with either unfractionated or low molecular
weight heparin. Warfarin could be started at the same day (or within 48 hours). Heparin therapy
should be continued for 5 days and stopped once INR is above 2 in two consecutive days.
NOTE – Since anticoagulation is contraindicated in the presence of a bleeding diathesis, the
following tests should be performed prior to heparinization: Activated partial thromboplastin
time (APTT) International normalised ratio (INR) Platelet count Thrombophilia screen
including: activated protein C resistance, fasting plasma homocysteine, prothrombin G20210A,
antithrombin III, protein C, protein S, lupus anticoagulant, anticardiolipin antibody and lupus
anticoagulant Warfarin should be continued for at least 3 months or more depending on the
patient’s risk of recurrent venous thromboembolism. The objectives of anticoagulation therapy
are treating the current DVT and prevention of pulmonary embolism. Studies have shown that
as many as 33% of patients may develop PE while receiving adequate anticoagulation therapy.
Cava filters are an alternative to systemic anticoagulation with warfarin (or heparin) in the
following situation: DVT or PE in patients with contraindications to anticoagulation therapy;
these patients include those with: Hemorrhagic stroke Recent neurosurgical procedure or other
major surgery Major or multiple trauma Active internal bleeding (e.g. upper or lower
gastrointestinal bleeding, hematuria, hemobilia) Intracranial neoplasm (either primary or
metastatic) Bleeding diathesis (e.g. secondary thrombocytopenia, idiopathic thrombocytopenic
purpura, hemophilia) Pregnancy Unsteady gate or tendency to fall (as seen in patients with
previous stroke, Parkinson disease) Poor patient compliance with medications DVT or PE in
patients with a complication of anticoagulation therapy (e.g. bleeding) Failure of
anticoagulation therapy – the patient develops venous thromboembolism while on therapeutic
dose of anticoagulant. Free-floating iliofemoral or caval thrombus PE prophylaxis - IVC filter
placement has been advocated as a means of preventing PE in patients at high risk for
thromboembolic events. Traditionally, such patients have included the following populations:
Patients with DVT who are about to undergo surgery (lower-extremity orthopedic surgery,
major abdominal surgery, neurosurgery) Patients with chronic pulmonary hypertension and a
marginal cardiopulmonary reserve Patients with cancer Trauma patients, including those with
(1) severe head injury with prolonged ventilator dependence, (2) major abdominal or pelvic
penetrating venous injury, (3) spinal cord injury with or without paralysis, (4) severe head
injury with multiple lower-extremity fractures, or (5) pelvic fracture with or without lower-
extremity fractures As this patient is known be noncompliance with his medications he should
have a inferior vena cava filter for prevention of PE.

113. A construction worker comes to you for advice. He and other fellow-workers had not
been wearing masks during renovations and now he is worried about asbestosis and
asbestosrelated cancers. Which one of the following would be the most appropriate
management?
A Refer him to respiratory specialist.
B Chest X-ray now and in 5 years’ time.
C Inform the authorities about the breach of workplace safety.
D Do nothing.
E Inform him that asbestos carries a very low risk of cancer.
Option B is correct Asbestos has been linked to an increased risk of bronchogenic carcinoma,
mesothelioma, gastrointestinal cancer and asbestosis among occupationally exposed workers.
Adequate screening for asbestos-related diseases does not exist at present. However, some tests,
particularly chest X-rays and pulmonary function tests, may indicate that an individual has
been overexposed to asbestos. It is important for physicians to become familiar with the
operating conditions that are associated with potential asbestos exposure. This is particularly
important in evaluating medical and work histories and in conducting physical examinations.
The current recommendations include taking a through history, performing physical exam with
close attention to respiratory system and the following diagnostic measures: Chest X-ray – A
chest X-ray should be taken for all patients with exposure to asbestos; however, initially, the
chest X-rays are often inconclusive because it takes approximately years for radiographic signs
of asbestos exposure to be evident on X-rays. It is recommended that chest X-rays be repeated
at 3-5 years intervals. Pulmonary function tests (PFTs) – simple PFTs available at general
practice level should be applied to recognize the possible restrictive patterns caused by
asbestosis. If the condition is work-related, involvement of a respiratory or occupational
physician is needed to consider factors such as the effect of the causative exposure on other
workers, how to control the exposure at the workplace and appropriate use of personal
protective equipment. When a current worker has been identified to have been overexposed to
asbestos ,measures taken by the employer to eliminate or mitigate further exposure should also
lower the risk of serious long-term consequences. The employer is required to institute a
medical surveillance program for all employees who are or will be exposed to asbestos at or
above the permissible exposure limit (0.1 fibre per cubic centimeter of air). NOTE – although
most patients with asbestos-related lung disease have a strong exposure history, significant
disease can occur with minimal exposure; hence, even minor or accidental exposures
necessiates evaluation.

114.A 70-year-old man presents with difficulty in swallowing for the past 6months and a
4- kilogram weight lost in this period. He describes that the most difficult part of
swallowing for him is when he tries to start to get the food down from his mouth. He had
been a smoker for most of his adult life but has quit 10 years ago. Which one of the
following would be the most appropriate management option at this point?
A Endoscopy.
B Surgery.
C Upper series barium study.
D Helicobacter pylori testing.
E Manometry.
Option C is correct No matter what the clues point towards, every patient with dysphagia
should undergo appropriate investigation. Just because of weight loss, the patient’s cannot be
told to have esophageal cancer. Although the patient’s age is a red flag for dysphagia, the fact
that it occurs at initiation of swallowing makes oropharyngeal dysphagia the better bet. On the
other hand, every patient with dysphagia, regardless of the etiology, may have weight loss due
to decreased calorie intake; nonetheless, a thorough and judicious assessment should be
considered for every patient with dysphagia. The best initial step in management of dysphagia
depends on provisional diganosis based on the history and clinical fidnings. When esophageal
cancer is suspected, evaluation starts with upper endoscopy and biopsy. With oropharyngeal
and motility-related dysphagia, barium studies would be the most appropriate initial approach.
In this scenario, ororpharyngeal dysphagia e.g. caused by a retropharyngeal pouch (Zenker's
diveticulum) is the most likely diagnosing; therefore barium swallow would be the best initial
assessment tool. If a retropharyngeal pouch is diagnosed on barium studies, endoscopy should
be avodied, as there is significant risk of the scope perforating the pouch. (Option A)
Endoscopy is the initial investigation when cancer is suspected based on history and clinical
features. (Option B) Surgery is indicated if the cause of dysphagia is found to be cancer or
Zenker’s diverticulum. Achalasia unresponsive to conservative measures may eventually need
surgical intervention. (Option D) Helicobacter pylori can cause peptic ulcer and consequently
strictures of gastric outlet (more common) or inlet (less common). Stricture at the junction of
the oesophagus to the stomach may cause dysphagia, but not described as difficulty in initiation
of swallowing. (Option E) Manometry can be used once barium meal study suggests a motility
disorder such as achalasia.
115.A 58-year-old woman presents to your practice with the complaint that she loses
control of her ‘waterworks’ when she puts her key in the front door. She denies urinary
leakage associated with laughing, coughing, or sneezing. Which form of urinary
incontinence is she most likely to have?
A Incontinence from a urinary fistula.
B Stress incontinence.
C Urge incontinence.
D Incontinence secondary to a neuropathic bladder.
E Overflow incontinence.
Option C is correct The patient’s description of incontinence is consistent with urge
incontinence as the most likely diagnosis. Urge incontinence occurs when the patient is unable
to delay micturition. The condition may be precipitated by various triggers such as the sound of
running water or placing a key in the door when arriving home. (Option A) urinary fistula is
associated with continuous dribbling or leakage of small amounts of urine on effort. (Option B)
Stress incontinence is associated with leakage of usually small amounts of urine during
activities that cause the intra-abdominal pressure to increase e.g., coughing or sneezing, which
this patient denies. (Option D) Neuropathic (neurogenic) bladder refers to bladder dysfunction
secondary to neurologic damage. Symptoms can include overflow incontinence, frequency,
urgency, urge incontinence, and urinary retention. Although a neurogenic bladder may lead to
urge incontinence, it is the cause rather than the type of the incontinence. (Option E) Overflow
incontinence is due to obstruction and may be secondary to uterovaginal prolapse or a
hypotonic bladder as in a neuropathic bladder.

116. A 65-year-old woman presents to your practice with complaint of urinary


incontinence that has restricted her social activities. She feels the urge to urinate while
outside and often cannot make it to a bathroom. She also mentions that every time she
puts the key into the door, it is like that her bladder wakes up and get irritated. Which
one of the following would be the most appropriate advice for her at this stage?
A Pelvic floor muscles exercises.
B Anticholinergic medications.
C Bladder training.
D Vaginal cones.
E Bladder neck suspension.
Option C is correct The case scenario represents a typical case of urge urinary incontinence.
For women with mild to moderate stress, urge and mixed urinary incontinence, behavioral and
physical therapy should be the initial treatment. It also can be an option for patients with more
severe symptoms. These therapies include bladder training and pelvic floor muscle exercise
(Kegel exercise). Although these two are often used in combination, pelvic floor muscle
exercises are recommended initially for genuine stress incontinence and the stress component
of mixed incontinence. On the other hand, bladder training is the first line-treatment for urge
incontinence. It should be noted, however, that these two are usually advised in combination.
This woman has urge incontinence; therefore, bladder training is first-line option for her
treatment. The goals of bladder training are: Establishing a baseline with a bladder diary
Establishing a voiding interval Gradually increasing the voiding interval in increments such as
2–5 minutes over a prolonged time period aiming to void every 3 hours (Option A) Pelvic
floor muscles exercise is also an important part to consider in conservative management in
combination with bladder training; however, not as effective as bladder training in urge
incontinence. (Option C) Anticholinergics, combined with continued bladder training, are an
option when urge incontinence does not respond to physical or behavioral therapies. A trial of 4
to 6 weeks is used. For those who respond, risk and benefits are balanced at 6 months to
determine if treatment should be continued. Patients should be advised about the adverse
effects of anticholinergics such as dry mouth and constipation, and how to manage them.
(Option D) Vaginal cones are used as supplement to pelvic muscle exercise for treatment of
stress urinary incontinence. The patient inserts the cone in her vagina and uses pelvic muscle
contractions to hold it in place during activity. This method is not effective for treatment of
urge incontinence. It does not seem effective in treatment of stress incontinence either if used
alone. (Option E) Bladder neck suspension is a more invasive method for treatment of stress
incontinence in patients who do not respond to more conservative measures. It is not used for
treatment of patients with urge incontinence.

117. A 75-year-old woman presents with leakage of urine when she laughs, sneezes,
coughs or lifts heavy weights. She also complains that she feels the urge to pass urine 10-
12 times a day, and half of time she cannot make it to the bathroom. Urgency appears to
be more troubling to her. A urine examination is negative for infections. Ultrasound scan
of the bladder, ureter and kidneys is inconclusive except a residual volume of 125 . Which
one of the following is the most appropriate treatment option for her?
A Anticholinergic medications.
B Anterior colporrhaphy.
C Retropubic suspension of the bladder.
D Pelvic floor muscle exercise.
E Bladder training.
Option E is correct This woman has signs and symptoms of both stress incontinence and urge
incontinence, namely mixed incontinence. However, she is annoyed mostly by the urge. For
women with urge continence as the most bothersome symptom, bladder training would be the
most appropriate management option to consider first. The goals are: Establishing a baseline
with a bladder diary Establishing a voiding interval Gradually increasing the voiding interval in
increments such as 2–5 minutes over a prolonged time period aiming to void every 3 hours
Other important measures to consider are lifestyle modifications such as decreased fluid intake,
weight reduction and avoiding certain foods and beverages that have diuretics effects (e.g.,
alcoholic beverages, caffeine, etc). (Option A) Anticholinergics, combined with continued
bladder training, are an option when urge incontinence does not respond to physical or
behavioral therapies. A trial of 4 to 6 weeks is used. For those who respond, risk and benefits
are balanced at 6 months to determine whether treatment should be continued. Patients should
be advised about the adverse effects of anticholinergics such as dry mouth and constipation,
and how to manage them. (Option B) Anterior coloprrhaphy has not a role in treatment of
urinary incontinence. (Option C) Retropubic suspension of the bladder is more invasive
procedure used for treatment of stress urinary incontinence in patients who has failed to
respond to more conservative measures such as pelvic floor exercise. (Option D) Pelvic floor
muscle exercise (e.g. Kegel exercise) is the most important initial conservative management to
consider for patients with genuine stress incontinence, and mixed (both stress and urge) urinary
incontinence when the stress incontinence is the more prominent complaint. Badder training
can be aadded fotr the urge.
118. A 70-year-old lady attends your clinic with complaint of urinary incontinence. She
describes her problem as leakage of small amounts urine when she laughs, sneezes or
coughs as well as during exercise. Which one of the following is the most appropriate next
step in management?
A Pelvic floor mucles exercise.
B Tension-free vaginal tape.
C Burch's colposuspension.
D Vaginal weighted cones.
E Topical estrogen cream.
Option A is correct The description is typical for genuine stress urinary incontinence (SUI) as
the most likely diagnosis. For women, who present with SUI or mixed incontinence, pelvic
floor exercise is the most appropriate management option to consider first. Bladder training is
the most important component of treatment in mixed incontinence with pronounced symptoms
related to an overactive bladder. Pelvic floor muscle exercise strengthens the muscular
components of the urethral closure mechanism. The basic recommended regimen is 3 sets of 8
to 12 slow velocity contractions sustained for 6 to 8 seconds each, performed 3 or 4 times a
week and continued for at least 3 months. (Options B and C) Tension free vaginal tape and
Burch’s colposuspension are more invasive procedures considered for patients who fail to
respond to conservative management strategies. Tension free vaginal tape is a minimally
invasive procedure that is performed by transcutaneous placement of a mesh around and under
the mid-urethra. It does not elevate the urethra, rather forms a resistant platform against
intraabdominal pressure. In Burch’s procedure, an intraabdominal approach is used to lift the
bladder neck upwards using stitches. (Option D) Weighted vaginal cones are used as
supplement to pelvic muscle exercise. The patient inserts the cone in her vagina and uses pelvic
muscle contractions to hold it in place during activity. Although small trials show efficacy
compared with no treatment in younger women, it is unclear if outcomes are comparable to
other therapies. . A 2013 systematic review and meta-analysis of 23 small randomized trials
found some evidence that cones have increased efficacy over no active treatment, but
inconclusive evidence that they provide increased efficacy over standard pelvic floor muscle
exercise. (Option E) Topical estrogen therapy has been used, as a supplement to pelvic floor
exercise, in postmenopausal women with urge and mixed incontinence. It is possible that
urinary urgency and frequency in urge incontinence are symptoms of urogenital atrophy in
older postmenopausal women. There is good evidence that low-dose topical vaginal estrogen
therapy may reverse the symptoms and cytologic changes of urogenital atrophy. However, oral
estrogens (with or without progesterone) have been shown to worsen urinary incontinence, and
should be avoided. Evidence supporting the use of estrogen for patients with stress urinary
incontinence is insufficient.

119. An 85-year-old woman presents with complaint of leakage of small amounts of urine
when she laughs, coughs, or sneezes. On examination, she has a reducible rectocele. As the
most appropriate initial step in management you advise bladder training and pelvic floor
muscle exercise that fails to improve her symptoms. Which one of the following is the
most appropriate next step in management?
A Vaginal cones.
B Imipramine.
C Surgical repair of the rectocele.
D Bladder neck suspension.
E Vaginal estrogen cream.
Option D is correct The scenario is a typical case of stress urinary incontinence. Stress
incontinence is the most common cause of urinary incontinence in younger women and the
second most common cause in older women (after mixed incontinence), and occurs when an
increase in intraabdominal pressure overcomes sphincter closure mechanisms in the absence of
a bladder contraction. Pelvic floor muscles exercise is the initial therapy in women with stress
and mixed incontinence, and also useful, in combination with bladder training, for women with
urge incontinence. If pelvic floor muscle training fails, the mainstay of treatment is surgery.
Surgery has high cure rates for stress urinary incontinence, even in elderly women. A vaginal
or abdominal approach can be used for surgical treatment. Vaginal procedures include
midurethral sling (preferred), bladder neck sling (suspension), or injection of urethral bulking
agents. Burch retropubic colposuspension is an abdominal procedure for stress urinary
incontinence. (Option A) Methods such as supervision of the pelvic floor exercise to check if
the appropriate pelvic muscles are contracted efficiently, biofeedback, and vaginal weighted
cones are used in combination with pelvic floor muscles exercise to achieve a better therapeutic
outcome. Vaginal cones are weighted devices designed to increase the strength of the pelvic
floor muscles. The cones are available in sets of 5, with identical shape and volume but
increasing weights (i.e., 20 g, 32.5 g, 45 g, 60 g, and 75 g). As part of a progressive resistive
exercise program, a single cone is inserted into the vagina and held in place by tightening the
levator ani muscles for as long as 15 minutes. As the levator ani muscles become stronger, the
exercise duration may be increased to 30 minutes. The intravaginal weight provides the sensory
feedback for the desired pelvic muscle contraction. A 2013 systematic review and meta-
analysis of 23 small randomized trials found some evidence that cones have increased efficacy
over no active treatment, but inconclusive evidence that they provide increased efficacy over
standard pelvic floor muscle exercise. An option suggesting addition of vaginal cones to the
therapy might be correct but with failed pelvic floor muscle exercise, this method alone is not
very likely to add therapeutic benefits. Moreover, vaginal cones are not effective in
management of stress incontinence associated with pelvic organ prolapse. (Option B) The role
of drug therapy in stress incontinence is very limited. Duloxetine (a serotonin and
noradrenaline reuptake inhibitor) has some effects on increasing bladder outlet resistance and is
effective in controlling mild urinary stress incontinence in women, but it is not approved for
this indication in Australia. (Option C) Currently, the complaint of this patient is urinary
incontinence not the rectocele, which is an incidental finding. Women with pelvic organ
prolapse, who are asymptomatic, do not require surgical repair. Moreover, in many women
with pelvic organ prolapse and stress urinary incontinence, surgical repair of the pelvic organ
prolapse (e.g., rectocele) results in worsening of the urinary incontinence. (Option E) Topical
estrogen therapy has been used, as a supplement to pelvic floor exercise and bladder training, in
postmenopausal women with urge and mixed incontinence. It is possible that urinary urgency
and frequency in urge incontinence are symptoms of urogenital atrophy in older
postmenopausal women. There is good evidence that low-dose topical vaginal estrogen therapy
improves or even may reverse the symptoms and cytologic changes of urogenital atrophy.
However, oral estrogens (with or without progesterone) should not be used to treat urinary
incontinence because it can worsen the condition. Evidence supporting use of estrogen in
patients with stress urinary incontinence is insufficient.
120. 70-year-old man presents to the emergency department with complain of loin pain
that he describes as on and off for the past 48 hours. On examination, he is afebrile and
there is no tenderness over the loin, flank or costovertebral angle. A urine test shows 3+
hematuria with negativity for protein and nitrite. Which one of the following is the most
appropriate step in management?
A Abdominal X-ray.
B IVP.
C Abdominal CT scan.
D Retrograde pyelography.
E Urine culture.
Option C is correct The scenario is highly suggestive of urinary stone(s) (nephrolithiasis) as
the most likely diagnosis. Nephrolithiasis affects one in 10 persons in Australians. Although
most stones pass spontaneously, some conditions require intervention. These conditions include:
Continuous pain Renal impairment Infection, mandate intervention For every patient presenting
with a clinical picture indicative of urinary stones investigations should be considered to
determine the location, number and size of the stone(s), which influence its likelihood of
spontaneous passage. Non-contrast CT scan of the abdomen for assessment of the kidneys,
ureters and bladder (CT-KUB) is the investigation of choice to consider. CT-KUB has a
sensitivity and specificity of approximately for evaluation of urinary stones irrespective of
stone type. It also provides an opportunity for an alternative diagnosis if no stone(s) is
visualized. It is important to note that an additional KUB plain film (x-ray) should be obtain at
the same presentation. If the stone is visible on plain KUB, this can be used for follow up
imaging, with a lower radiation dose. Only calcium containing stones are visible on plain KUB
X-ray, and this information also has implications for stone management. (Option A) As
mentioned above, a KUB x-ray is performed for reducing the radiation exposure associate with
CT scanning if the stone is visualized. Otherwise, this modality is not sensitive enough to
establish a diagnosis. (Option B) Intravenous pyelogram (IVP) is used rarely in the diagnosis
of ureteric colic due to the sensitivity and specificity of non-contrast CT KUB. However, a
contrast study (CT urogram) may sometimes be helpful for treatment planning purposes.
Retrograde pyelography (option C) serves the same purpose as does IVP but the contrast
material is pushed up through a urinary catheter. This test has no role for evaluation of patients
with real colic. (Option E) This patient has no signs, symptoms, or dipstick urine exam
findings suggestive of urinary tract infection. If that was the case, a urine culture and sensitivity
would have been indicated for further management. NOTE - Ultrasound is generally reserved
for the assessment of loin pain in pregnant women or for follow-up of patients with uric acid
stones, which cannot be seen on a plain KUB X-ray due to their radiolucency. Ultrasound may
also be used in follow up to ensure that signs of obstruction (such as hydronephrosis) have
resolved after conservative management of ureteric colic, or to monitor stone size in
asymptomatic patients. There is no role for magnetic resonance imaging (MRI) in the
assessment or management of urolithiasis.

121.A 32-year-old woman presents to your GP office for advice regarding breast cancer
after her maternal aunt was diagnosed with breast cancer 3 months ago. She wants to
know about the risk factors for breast cancer. She is obese and has a body mass index
(BMI) of 45. She smokes 20 cigarettes a day and is oral contraceptive pills (OCPs). The
rest of examination, including breast exam, is unremarkable. Which one of the following
pieces of advice is the best for reducing the risk of breast cancer for her?
A Smoking cessation.
B Weight loss.
C Regular exercise.
D Using low-estrogen dose OCPs.
E Using non-hormonal methods for contraception.
Option B is correct It has been estimated that approximately 20%of all cancers are caused by
excess weight. The Million Women Study, the largest study of its kind on women has shown
that approximately half of cancers can be attributed to obesity in postmenopausal women. The
International Agency for Research into Cancer (IARC) and the World Cancer Research Fund
(WCRF) reports show that common cancers in obese people are predominantly endometrial,
esophageal adenocarcinoma, colorectal, postmenopausal breast cancer, prostate and renal. Less
common malignancies associated with obesity are melanoma, thyroid cancer, leukemia, non-
Hodgkin’s lymphoma, and multiple myeloma. The role of obesity in development of breast
cancer in women is hypothesized to be due increased production of endogenous estrogen in the
fat tissue and excess exposure of the breast tissue to unopposed endogenous estrogen. For this
patient, weight reduction is the most appropriate advice both for general health and risk
reduction for breast cancer. (Option A) Smoking have a positive link to a number of
malignancies. Some reports advocate tobacco smoking as a risk factor for the development of
breast cancer; however, this risk is not as significant as that of obesity. (Option C) regular
exercise is associated with improved overall health as well as decreased incidence of
malignancies. The effect does not seem to be superior to weight reduction for breast cancer.
(Options D and E) Use of contraceptive pill has been associated with only a slightly increased
risk of breast cancer after 5 years of use. Changing the contraception method or reducing the
dose of estrogen is not likely to significantly decrease such risk. TOPIC REVIEW Many risk
factors for breast cancer have been identified, including genetic, environmental, and lifestyle
factors. Of these risk factors some are modifiable while others not. The physician should take a
detailed a through family and personal history to estimate the patient's risk for breast cancer.
After risk assessment and establishment, physician and patient can discuss the best preventive,
screening and management plan involving measures such as addressing modifiable risk factors
or genetic counseling. Screening is important, because early detection may lead to better
outcomes. The most important risk factors for development of breast cancer include the
following: Factors associated with the highest risk for development of breast cancer (relative
risk [RR] >4.0) are as follows: Advanced age (65 years and older) Atypical hyperplasia of
breast (biopsy proven) Certain inherited genetic mutations ( BRCA1, BRCA2, TP53, ATM,
CDH1); RR 4-8 Ductal or lobular carcinoma in situ (DCIS/LCIS); RR 8-10 Family history of
early ovarian cancer (age <50 years) Multiple first-degree relatives with breast cancer Ionizing
radiation exposure before age 30 (RR 22-40) Personal history of early breast cancer (age <40)
Factors associated with RR 2.1-4.0 for breast cancer are as follows: High endogenous estrogen
or testosterone level (postmenopausal) First full-term pregnancy after age 35 years Very dense
breasts (>50%, compared with 11-25% mammographically) One first-degree relative with
breast cancer Proliferative breast diseases (eg, atypical ductal hyperplasia) Certain inherited
genetic mutations (eg, CHEK2, PTEN) Factors associated with RR 1.1-2.0 for breast cancer are
as follows : Alcohol consumption Age 30-35 at first full-term pregnancy Diethylstilbestrol
exposure in utero Early menarche (age <12 years) Height (>5 feet 3 inches) High
socioeconomic class Ashkenazi Jewish heritage Personal history of breast cancer (age of
onset >40) Dense breasts (25-50%, compared with 11-25% mammographically) Benign breast
conditions:Non-atypical ductal hyperplasia, fibroadenoma, sclerosing adenosis, microglandular
adenosis, papillomatosis, radial scar Never breastfed a child Nulliparity (no full-term
pregnancies) Late menopause (age >55) Type II diabetes mellitus Obesity (post-menopausal)
Personal history of uterine, ovarian, or colon cancer Recent and long-term use of hormone
replacement therapy (HRT) containing estrogen and progestin Recent oral contraceptive use
Occupation: night shift Tobacco abuse Sedentary lifestyle Inferior cardiovascular health High
bone mineral density Factors that reduce risk of breast cancer (RR <1) include the following:
Asian, Hispanic, or Pacific islander race Breastfeeding Age <20 at first pregnancy Tamoxifen
use Prior risk-reduction breast surgery History of cervical cancer History of oophorectomy
Exercise/active lifestyle Low bone mineral density

122. Three months ago you started Mary, aged 73, on residronate 35mg weekly, after she
was diagnosed with osteoporosis confirmed with bone mineral density (BMD). Her
presenting symptom at that time was back pain. Examination revealed height decrease
and mild kyphosis, as well as tenderness over thoracic vertebrae. Despite being on
residronate, not only did her symptoms persist, she also developed pain over new areas of
her thoracic. Investigations have established new osteoporotic fractures. Which one of the
following would be the most appropriate management option for her?
A Continue residronate at the same dose.
B Switch to alendronate.
C Increase the dose of residronate.
D Switch to zoledronic acid.
E Switch to teriparatide.
Option E is correct Bisphosphonates are currently the most commonly anti-resorptive agents
prescribed as first-line treatment for most of osteoporotic patients. Of this drug family,
alendronate (10mg/day or 70mg/week, orally), residronate (5mg/day or 35mg/week, orally) and
zoledronic acid are available in Australia. Bisphosphonates should be used for at least 12
months before their efficacy on treatment of osteoporosis is assessed. However, in cases where
the patient suffers two or more minimal trauma fractures despite being on sufficient doses of an
anti-resorptive drug, e.g. bisphosphonates, commencement of teriparatide is justified as the
most appropriate option ( E is correct). Teriparatide is the synthetic parathyroid hormone that
predominantly acts by increasing the osteoblasts (boneforming cells) and by inducing new
osteoblasts formation. This drug is costly and at least 18 months of continuous use is needed
effectiveness. Based on these, this medication is only reimbursed by the PBS for patients with
severe osteoporosis and very high risk of fractures who have: A BMD T-score of =<-3 OR had
two or more minimal trauma fractures OR experienced at least one symptomatic new fracture
after at least 12-months of continuous therapy with an anti-resorptive agent at adequate doses.
(Option A) While Mary has developed new osteoporotic fractures due to severe osteoporosis,
continuation of the same agent is not a wise management. Switching to other bisphosphonates
such as alendronate (option B) or zoledronic acid (option D) does not add any benefit, as all
members of this drug family have almost the same effectiveness. Mary is already on the
recommended weekly dose for residronate and increasing the dose of residronate (option C)
does not seem to benefit her.
123. A 65-year-old patient of yours, is being assessed for osteoporosis. A Dual Energy X-
ray Absorptiometry has revealed T-scores of -2.5 and -2.7 for the femoral neck and the
vertebral column, respectively. She was diagnosed with the cancer of her right breast 6
years ago for which she underwent right mastectomy, chemotherapy and radiation
therapy. In addition to advice regarding calcium and vitamin D, which one of the
following medications would be the best option for treatment of her osteoporosis?
A Raloxifene.
B Alendronate.
C Teriparatide.
D Strontium ranelate.
E Hormone replacement therapy (HRT).
Option B is correct According to WHO criteria and based on Jane’s T-scores at vertebra and
femoral neck, she is suffering from osteoporosis. Currently, bisphosphonates are the first-line
treatment option for primary and secondary treatment of vertebral and non-vertebral fractures
due to osteoporosis. Alendronate, residronate and zoledronic acid are the available
bisphosphonates in Australia . (Option A) Raloxifene is a selective estrogen receptor
modulator (SERM) reimbursed by the Pharmaceutical Benefit Scheme (PBS) for treatment of
postmenopausal osteoporosis. SERMs also have a preventive effect on breast cancer; therefore,
an appropriate option for post-menopausal women with family or personal history of breast
cancer in whom vertebral fracture due osteoporosis is a concern. Although there is excellent
evidence (Grade A) that raloxifene reduces the risk of vertebral fractures, limited evidence
support its effect on nonvertebral fractures. Jane is osteoporotic in femur as well; hence
raloxifene would not an appropriate first-line option for her. (Option C) Teriparatide is the
synthetic parathyroid hormone that predominantly acts by increasing the osteoblasts (bone-
forming cells) and by inducing new osteoblasts formation. This drug is costly and at least 18
months of continuous use is needed effectiveness. Based on these, this medication is only
reimbursed by the PBS for patients with severe osteoporosis and very high risk of fractures
who have: A BMD T-score of =<-3 OR had two or more minimal trauma fractures OR
experienced at least one symptomatic new fracture after at least 12-months of continuous
therapy with an anti-resorptive agent at adequate doses. Jane does not any of the above criteria
to meet the need for teriparatide. NOTE – BMD starts declining approximately 12 months after
cessation of therapy with teriparatide; hence, continuation of therapy with an anti-resorptive,
generally a bisphosphonates. (Option D) Strontium ranelate is an effective second-line option
for reducing the risk of further osteoporotic fractures in postmenopausal women with prevalent
fractures. This agent should not be used in patients with previous or clinically active
cardiovascular disease or uncontrolled hypertension and should only be used when other drug
classes for treatment of osteoporosis are not appropriate. Strontium ranelate is prescribed at a
dose of 2 grams per day. (Option E) Breast cancer is estrogen dependent; therefore,
prescribing estrogen for patients with a personal or strong family history of breast cancer is not
an appropriate option. Estrogen is an option (available on PBS) for the prevention and
treatment of osteoporosis in women who are near or at menopause, in particular for those
patients suffering from menopausal vasomotor symptoms, e.g. hot flushes. When considering
this therapy, potential adverse effects i.e. increased risk of thromboembolism, cardiovascular
diseases and breast cancer should be weighed and discussed with the patient.
124. A 69-year-old man presents to your GP clinic with complaint of uriary incontinence.
He explains that he cannot make it to the bathroom once he feels the urge to pass urine.
He denies any dysuria. Which one of the following could be the most likeky underlying
cause to this presentation?
A Benign prostatic hyperplasia (BPH).
B Urinary tract infection (UTI).
C Detrusor instability.
D Increased intraabdominal pressure.
E Nephrolithiasis.
Option C is correct The incontinence described in the scenario is characteristic of urge
incontinence. In this type of incontinence, there is sudden and strong need to urinate. The
patient often is not able to make it to the bathroom and wet themselves. An overactive bladder
caused by detrusor instability is the most common etiology of urge incontinence. An irritable or
unstable bladder are synonymous terms used to describe the pathology. Patients with detrusor
instability have involuntary bladder contractions, resulting in a sudden urge to urinate. (Option
A) BPH can often cause overflow urinary incontinence rather than urge incontinence. (Option
B) Although UTI is a cause of frequency and urgency due to irritation of the bladder, detrusor
instability of unknown etiology is the most common cause of urge incontinence. (Option D)
Increased intraabdominal pressure is a cause of urine overflow in stress incontinence not urge
incontinence. (Option E) A stone in the bladder can irritate the bladder and cause frequeny,
urgency and urge incontinence but idiopathic detrusor instability remains the commonest cause
of urge incontincne.

125. Three months ago you started Mary, aged 73, on residronate 35mg weekly, after she
was diagnosed with osteoporosis confirmed with bone mineral density (BMD). Her
presenting symptom at that time was back pain. Examination revealed height decrease
and mild kyphosis, as well as tenderness over thoracic vertebrae. Despite being on
residronate, not only did her symptoms persist, she also developed pain over new areas of
her thoracic. Investigations have established new osteoporotic fractures. Which one of the
following would be the most appropriate management option for her?
A Continue residronate at the same dose.
B Switch to alendronate.
C Increase the dose of residronate.
D Switch to zoledronic acid.
E Switch to teriparatide.
Option E is correct Bisphosphonates are currently the most commonly anti-resorptive agents
prescribed as first-line treatment for most of osteoporotic patients. Of this drug family,
alendronate (10mg/day or 70mg/week, orally), residronate (5mg/day or 35mg/week, orally) and
zoledronic acid are available in Australia. Bisphosphonates should be used for at least 12
months before their efficacy on treatment of osteoporosis is assessed. However, in cases where
the patient suffers two or more minimal trauma fractures despite being on sufficient doses of an
anti-resorptive drug, e.g. bisphosphonates, commencement of teriparatide is justified as the
most appropriate option. Teriparatide is the synthetic parathyroid hormone that predominantly
acts by increasing the osteoblasts (boneforming cells) and by inducing new osteoblasts
formation. This drug is costly and at least 18 months of continuous use is needed effectiveness.
Based on these, this medication is only reimbursed by the PBS for patients with severe
osteoporosis and very high risk of fractures who have: A BMD T-score of =<-3 OR had two or
more minimal trauma fractures OR experienced at least one symptomatic new fracture after at
least 12-months of continuous therapy with an anti-resorptive agent at adequate doses. (Option
A) While Mary has developed new osteoporotic fractures due to severe osteoporosis,
continuation of the same agent is not a wise management. Switching to other bisphosphonates
such as alendronate (option B) or zoledronic acid (option D) does not add any benefit, as all
members of this drug family have almost the same effectiveness. Mary is already on the
recommended weekly dose for residronate and increasing the dose of residronate (option C)
does not seem to benefit her.

126. A 65-year-old patient of yours, is being assessed for osteoporosis. A Dual Energy X-
ray Absorptiometry has revealed T-scores of -2.5 and -2.7 for the femoral neck and the
vertebral column, respectively. She was diagnosed with the cancer of her right breast 6
years ago for which she underwent right mastectomy, chemotherapy and radiation
therapy. In addition to advice regarding calcium and vitamin D, which one of the
following medications would be the best option for treatment of her osteoporosis?
A Raloxifene.
B Alendronate.
C Teriparatide.
D Strontium ranelate.
E Hormone replacement therapy (HRT).
Option B is correct According to WHO criteria and based on Jane’s T-scores at vertebra and
femoral neck, she is suffering from osteoporosis. Currently, bisphosphonates are the first-line
treatment option for primary and secondary treatment of vertebral and non-vertebral fractures
due to osteoporosis. Alendronate, residronate and zoledronic acid are the available
bisphosphonates in Australia . (Option A) Raloxifene is a selective estrogen receptor
modulator (SERM) reimbursed by the Pharmaceutical Benefit Scheme (PBS) for treatment of
postmenopausal osteoporosis. SERMs also have a preventive effect on breast cancer; therefore,
an appropriate option for post-menopausal women with family or personal history of breast
cancer in whom vertebral fracture due osteoporosis is a concern. Although there is excellent
evidence (Grade A) that raloxifene reduces the risk of vertebral fractures, limited evidence
support its effect on nonvertebral fractures. Jane is osteoporotic in femur as well; hence
raloxifene would not an appropriate first-line option for her. (Option C) Teriparatide is the
synthetic parathyroid hormone that predominantly acts by increasing the osteoblasts (bone-
forming cells) and by inducing new osteoblasts formation. This drug is costly and at least 18
months of continuous use is needed for effectiveness. Based on these, this medication is only
reimbursed by the PBS for patients with severe osteoporosis and very high risk of fractures
who have: A BMD T-score of =<-3 OR had two or more minimal trauma fractures OR
experienced at least one symptomativ fracture after at least 12-months of continuous therapy
with an antiresorptive agent in adequate doses. Jane does not any of the above criteria to meet
the need for teriparatide. NOTE – BMD starts declining approximately 12 months after
cessation of therapy with teriparatide; hence, continuation of therapy with an anti-resorptive,
generally a bisphosphonates. (Option D) Strontium ranelate is an effective second-line option
for reducing the risk of further osteoporotic fractures in postmenopausal women with prevalent
fractures. This agent should not be used in patients with previous or clinically active
cardiovascular disease or uncontrolled hypertension and should only be used when other drug
classes for treatment of osteoporosis are not appropriate. Strontium ranelate is prescribed at a
dose of 2 grams per day. (Option E) Breast cancer is estrogen dependent; therefore,
prescribing estrogen for patients with a personal or strong family history of breast cancer is not
an appropriate option. Estrogen is an option (available on PBS) for the prevention and
treatment of osteoporosis in women who are near or at menopause, in particular for those
patients suffering from menopausal vasomotor symptoms, e.g. hot flushes. When considering
this therapy, potential adverse effects i.e. increased risk of thromboembolism, cardiovascular
diseases and breast cancer should be weighed and discussed with the patient.

127. Which one of the following conditions is the most common inherited hypercoagulable
state leading to venous thromboembolism (VTE)?
A Anti-thrombin deficiency.
B Protein C deficiency.
C Protein S deficiency.
D Factor VLeiden.
E Lupus anticoagulant.
Option D is correct Venous thromboembolism (VTE) is a multifactorial disease driven by
environmental/acquired risk factors such as age, obesity, oral contraceptives, and immobility,
as well as inherited risk factors such as genetic polymorphisms. Of genetic factors, factor V
Leiden is the most common genetic cause of VTE . Prothrombin G20210A, gene mutation and
deficiencies in protein S, protein C, and anti-thrombin account for most of the remaining cases.
It should be noted however that although factor V Leiden is the most common genetic cause of
VTE, it is associated with only a medium risk of VTE in carriers.

128. Which one of the following hereditary conditions is associated with highest risk of
venous thromboembolism (VTE)?
A Anti-thrombin deficiency.
B Protein C deficiency.
C Protein S deficiency.
D Factor 5 Leiden.
E Lupus anticoagulant.
Option A is correct In addition to conditions such as immobility, obesity, oral contraceptive
pills, etc., there are a variety of inherited factors that contribute to VTE. These factors are also
known as strong, medium and weak risk factors. Anti-thrombin deficiency, protein C
deficiency (option B), and protein S deficiency (option C), are associated with high risk of
VTE. Among this group members, anti-thrombin deficiency, formerly known as anti-thrombin
III deficiency, confers the highest risk of VTE.. (Option D) Factor 5 Leiden is a moderate risk
factor for VTE. (Option E) Lupus anticoagulant is also a factor for both VTE and arterial
thromboembolism but is an acquired condition not an inherited one. The prevalence is 1 to 5%
of population and even more in the elderly and those with a comorbid condition such as cancer.
The risk of developing VTE in an individual with lupus anticoagulant is approximately 6-8%.

129. A 55-year-old diabetic woman presents with a foot ulcer shown in the following
photograph. On examination, she looks ill and has of 38.6°C. Which one of the following
is the most appropriate next step in management after wound debridment and swabs?

A Oral amoxicillin – clavulanate.


B Intravenous ticarcillin - clavulanate.
C MRI of the foot.
D Daily wet dressing.
E Daily wound toilet with povidone iodine.
Option B is correct Therapeutic Guidelines – Antibiotics; available from: http://tg.org.au The
photograph shows an ulcer on the fifth toe of the right foot with marked inflammation and
cellulitis of the surrounding tissue. Moreover, the patient has fever and tachycardia (systemic
toxicity). In such cases, after debridment and taking swabs for cultures, antibiotics are of
significance importance as the next best step in management. The antibiotics of choice for
patients with severe infection evident by systemic toxicity is piperacillin + tazobactam or
ticarcillin + clavulanate. (Option A) Oral amoxicillin-clavulanate is used for mild infections or
ulcers without infection but odorous discharge. (Option C) MRI is indicated if osteomyelitis,
as a complication of the ulcer, is suspected. It is not a priority at this stage. (Option D) Daily
wet dressing is used as adjunct therapy to antibiotics for wound care. (Option E) Cytotoxic
antiseptics such as povidone iodine delay wound healing and should not be used.

130. A 60-year-old diabetic woman, whose diabetes is well controlled on insulin, comes to
your clinic with an ulcer on the dorsum of her right fifth toe which she noted 5 days ago.
On examination, a purulent odorless discharge is noted. There is marked inflammation of
the fifth toe, extending to the lateral aspect of the foot and ankle. She is febrile, has a
blood pressure of 110/85mmHg, and heart rate of 104bpm. Which one of the following is
the most appropriate next step in management?
A Intravenous ticarcillin + clavulanate.
B Analgesics.
C X-ray of the foot.
D MRI of the foot.
E Debridment of the ulcer.
Option E is correct In approach to diabetic-related foot ulcers (DRFU), the most important
initial step in management is debridment of the ulcer. Debridment allows comprehensive
evaluation of the wound, wound bed, and actual wound size. Debridment can convert a chronic
wound to an acute one with accelerated healing. It also removes local pressure on the wound.
Therapeutic Guidelines – Antibiotics; available from: http://tg.org.au After the wound is
debrided, swabs are taken for bacteriological assessment and culture. This should be performed
before antibiotics are started. In the next step, antibiotics should be stared if indicated. (Option
A) The decision as to empirical antibiotic treatment is guided by the characteristics of the ulcer.
The following table summarizes the types of diabetic ulcers, associated clinical features and
antibiotics of choice for antimicrobial treatment: check table on page 2811 started on
intravenous antibiotics , and ticarcillin + clavulanate is an appropriate option after debridment
and swabs. (Option B) Analgesics can be given for pain control. It is not a priority before
debridment, swabs and antibiotics. (Option C) In the presence of an infection/inflammation,
lateral, antero-posterior and oblique X-ray views of both feet should be performed with the X-
ray request specifying the anatomical location of the ulcer and mentioning the possibility of
underlying osteomyelitis, diabetic arthropathy and gas formation. However, it can be
performed after more important steps (debridement, swabs, and antibiotics) have been taken.
(Option D) MRI is indicated if osteomyelitis is suspected based on the following: Ulcers
unresponsive to antibiotic therapy Probing the wound results in bone being touched Bone is
visible in the base of the wound X-ray suggest bone involvement In this scenario, MRI is not a
first priority in management.

131. A 62-year-old diabetic man presents with a foot ulcer shown in the following
photograph. On examination, he is not febrile. Debridment is perfomred, swabs for
cultures are taken and antibiotics started. Which one of the following is the most

appropriate next step in management? A X-


ray of the foot.
B Blood culture.
C MRI.
D Daily wet dressing of the wound.
E Ultrasonography.
Option A is correct A step-wise approach to management of diabetic foot ulcers includes: 1-
Debridment Management of diabetic foot ulcers starts with debridment as a very important
initial step in management. It allows comprehensive evaluation of the wound, wound bed and
actual wound size. Debridment may also convert a chronic wound to an acute one with
accelerated healing. Reducing the local pressure on the wound is another benefit. Debridment
can be surgical or non-surgical. It must be noted, however, that in the presence of peripheral
vascular disease wound debridment should be performed after comprehensive assessment of
the vasculature status. Debridment in an ischemic limb is associated with complications. 2-
Wound swabs If infection is evident or suspected, deep wound swabs or needle aspiration of
the exudate should be taken for bacteriological analysis before starting antibiotic treatment. If
an ulcer can be probed down to bone, osteomyelitis is likely to be present (MRI is indicated). It
is usual to take a wound swab even in the absence of the clinical signs of infection or in an
ulcer which is clean, does not probe to bone, is not producing large amounts of exudate, and
has granulation tissue. The swab should be taken as localized and as deep as possible. If
pathogen is found without any clinical signs of infection, a topical bacteriocidal dressing, such
as one containing nanocrystalline silver, may help clear the wound of bacteria. 3- Antibiotics
Antibiotics are indicated if there is minimal inflammation but odorous wound secretions, or in
the presence of moderate to severe infection. Choices of antibiotics depend on the wound
characteristic and associated symptoms. The following table summarizes antibiotic therapy for
different diabetic ulcers: check page 2815 Therapeutic Guidelines – Antibiotics; available from
http://tg.org.au Severe infection Infection as above AND systemic toxicity or metabolic
instability e.g. fever, chills, tachycardia, hypotension, confusion, vomiting, leukocytosis, sever
hyperglycemia, or azotemia Ticarcillin + clavulanate or (if penicillin allergic) Ciprofloxacin
PLUS either clindamycin or lincomycin 4- Imaging When clinical signs of inflammation are
evident, lateral, antero-posterior and oblique X-ray views of both feet should be obtained, with
the X-ray request specifying the anatomical location of the ulcer and mentioning the possibility
of underlying osteomyelitis, diabetic arthropathy and gas formation. Bone infection usually has
to be present for several weeks before it is detectable on plain X-ray films, so serial X-rays at
one to four weeks intervals may be necessary if clinical infection fails to resolve and the initial
X-ray is clear. MRI of the affected area may be useful for differentiating infectious from non-
infectious inflammatory conditions. However, if MRI facilities are not available, a surgeon
should be able to make a decision regarding exploration and debridment of an ulcer based on
clinical examination. 5- Other tests Surgical biopsy of the affected area may be useful for
collection of subsequent specimens for microbiological examination if deterioration occurs
after empirical antibiotic therapy is started. Full blood count, ESR, electrolytes, blood glucose
and HbA1c, renal and liver function tests are indicated. Weekly measurement of protein titer
during treatment of a foot infection may help monitor the response to treatment. In this patient,
swabs have been taken and antibiotics started, and the most appropriate next step will be
antero-posterior, lateral and oblique X-rays of both feet as mentioned earlier. (Option B) Blood
cultures may be indicated if the patient had systemic manifestations such as fever and chills
(absent in this patient). (Option C) MRI is indicated if osteomyelitis is suspected. A probe
reaching the bone, bone being visible through the ulcer, or inadequate response to antibiotics
makes osteomyelitis likely. (Option D) Daily wet dressing of the wound is the treatment of
choice for non-infected ulcers, and adjuvant treatment to antibiotics for infectious wounds.
(Option E) Ultrasound is not recommended for initial assessment of diabetic foot ulcers.

132. A 65-year-old woman with long-standing history of type II diabetes mellitus (DM)
presents with a right foot ulcer which she noticed 7 days ago. You admit her to the
hospital for treatment based on the diagnosis of a diabetes-related foot ulcer (DRFU).
Wound debridment is carried out. The following day, examination reveals that the ulcer
is 1cm in diameter and oozing an odorless clear fluid. Which one of the following is the
most appropriate next step in management?
A Oral amoxicillin-clavulanate + metronidazole.
B Intravenous ticarcillin + metronidazole.
C MRI of the foot.
D Daily wet dressing of the wound and follow-up.
E Wound toilet with povidone iodine.
Option D is correct Foot ulceration is a major cause of mortality and morbidity in diabetic
patients. Management of every diabetic-related foot ulcer (DRFU) starts with debridment as an
essential component of wound bed preparation by removing non-viable tissue. This allows for
comprehensive examination of the wound bed and assessment of actual wound size.
Additionally, debridment may convert a chronic wound to an acute one with accelerated
healing. Reducing the local pressure on the wound is another benefit. Debridment can be
surgical or non-surgical. Surgical sharp debridment is indicated when there is deep necrotic
tissue, gangrene, drainage of a collection, or deep infections. Non-surgical debridment is
generally required every 1-2 weeks to remove non-viable tissue, including the surrounding
callous. Non-sharp debridment with enzymatic, autolyitic or biological preparations can be
used as an adjunct to sharp debridment and can be helpful for adherent slough, or when pain
limits surgical sharp debridment. NOTE – In the presence of peripheral vascular disease,
debridment should be avoided and the patient referred for specialist vasculature assessment.
The only exception to this rule is the presence of an abscess, in which case debridment and
drainage should be performed regardless of vascular status. The rest of management relies upon
whether the wound is infected or inflamed, and if so, the wound characteristics. The following
table outlines further steps for wound management based on the presence of
infection/inflammation and its severity: Wound Characteristics Antibiotic therapy Uninfected
Wound does not have purulence or any manifestations of inflammation Not required unless the
ulcer is odorous. Treat as mild infection. Without odor, only daily wet dressing. Mild infection
Presence of 2≥ manifestations of inflammation e.g. purulence, erythema, pain, tenderness,
warmth or induration BUT no cellulitis/erythema greater than 2cm AND no extension beyond
superficial subcutaneous tissue AND no systemic illness Oral Amoxicillin + clavulanate or
Cephalexin PLUS metronidazole or (if penicillin allergic) Ciprofloxacin +clindamycin
Moderate infection Infection as above PLUS cellulitis extending >2cm OR lymphangitic
streaking OR extension beyond superficial fascia, deep-tissue abscess, gangrene OR extension
to muscle, tendon, joint or bone; NO systemic involvement Dicloxacillin/ flucloxacillin
Metronidazole must be added if the wounds discharges are odorous (anaerobe activity) Severe
infection Infection as above AND systemic toxicity or metabolic instability e.g. fever, chills,
tachycardia, hypotension, confusion, vomiting, leukocytosis, sever hyperglycemia, or azotemia
Piperacillin + tazobactam or Ticarcillin + clavulanate or (if penicillin allergic) Ciprofloxacin
PLUS either clindamycin lincomycin This patient has a 1cm ulcer. There is no comment
regarding surrounding inflammation or cellulitis to suggest infection. Moreover, the wound
secretions are clear and non-infective. Management of such wounds, after initial debridment,
includes deep wound swabs (if not already taken) and wound dressing and daily check to assess
the healing process. Although numerous topical medications and gels are promoted for ulcer
care, relatively few have proved to be more efficacious than warm wet dressing. Check page
2819 NOTE – If there is peripheral vascular disease, dry wound dressing should be applied and
the patient referred for specialist vascular assessment and care. (Options A and B) Antibiotics
are used when the wound is associated with infection/inflammation. Oral amoxicillin-
clavulanate is indicated for ulcers with minimal infection or those without signs of infection but
with odorous secretions. Parenteral antibiotics are indicated when there is marked infection.
For deep or widespread infections or those associated with osteomyelitis, antibiotics such as
piperacillin- tazobactam, ticarcillin-clavulanate, meropenem, ertapenem, carbapenem,
moxifloxacin, or ciprofloxacin PLUS metronidazole, or a third-generation cephalosporin PLUS
metronidazole are used. (Option C) MRI is indicated where osteomyelitis is suspected based
on clinical or laboratory studies. (Option E) Topical antiseptics, such as povidone iodine, are
toxic to the healing wound.
133.A 62-year-old man is referred to your clinic for a health checkup by his insurance
company. An ultrasound of the kidney, ureter and bladder reveals a 3-cm heterogenous
mass in the upper pole of his right kidney. A contrast-enhanced CT scan is ordered that
confirms the presence of a contrast-enhaced cystic mass. Which one of the following
would be the most appropriate management for this patient?
A Total nephrectomy.
B Review in 12 months.
C Percutaneous biopsy.
D Partial nephrectomy.
E Thermal ablation.
Option D is correct Increasing use of imaging to diagnose other conditions has increased the
rate at which small renal masses are detected. Incidentally found small renal masses
(incidentalomas) are now a common clinical scenario. Recent data have revealed that over 50%
of renal cell carcinomas (RCCs) are incidentally found, and the classic presentation with a triad
of gross hematuria, flank pain, and abdominal mass is not commonly encountered. The
majority of incidentally-detected renal masses are benign simple cysts that require no further
work-up. However, completely solid or mixed solid and cystic renal lesions and cystic lesions
that enhance with contrast are likely to be malignant. This patient has also hematuria that
makes malignancy even more likely. By definition, a renal lesion <4cm in its largest diameter
that shows contrast enhancement on abdominal imaging is a small renal mass. These lesions
can be solid or complex cystic. Simple cysts not enhancing with contrast are not considered
renal small masses. For small renal masses, surgical resection is the most appropriate option if
life expectancy is >5 years and the patient is a good candidate for surgery. This man has a 3-cm
complex tumor and needs surgical resection of the tumor as the most appropriate management
option. For renal masses smaller than 7-cm in size, nephron-sparing surgery, or partial
nephrectomy rather than a total nephrectomy is performed to allow for preservation of renal
function. This patient with a 1cm mass is most likely to benefit from partial nephrectomy as the
most appropriate management option. (Option A) Total nephrectomy is the treatment of choice
in the following situations: Tumor size ≥7 cm Those tumors that have a more central location
Suspected lymph node involvement Tumor with associated renal vein or inferior vena cava
(IVC) thrombus Direct extension into the ipsilateral adrenal gland NOTE – Even in the
presence of the above, patients with any of the following conditions must have partial rather
than total nephrectomy: A solitary kidney Multiple, small, and/or bilateral tumors Patients with
or at risk for chronic renal disease (Option B) Active surveillance with CT scan or MRI in 6-
to 12-monht intervals is recommended for patients with a small renal mass who are not good
candidates for surgical resection (e.g. have a comorbid condition), or have a life expectancy of
≤5 years. Renal impairment may also be an indication for active surveillance in some patients.
Active surveillance means that the patient will either have delayed treatment or no treatment at
all. This patient has no comorbid condition precluding him from tumor resection.
nload/documents/AFP/2011/October/201110rao.pdf) (Option C) The role of renal biopsy is
controversial in the setting of a small renal mass. In particular, there is disagreement as to
whether it is necessary to biopsy these lesions before planning management and in what
settings. Circumstances under which a renal biopsy might be considered are where the patient
is not a surgical candidate (controversial), life expectancy is ≤5 years (controversial), or the
patient requests a definite diagnosis before proceeding with the surgery. Biopsy is not
recommended for patients who are candidates for active surveillance. (Option E) Ablation of
renal mass can be performed with either freezing (cryoablation) or heat (radiofrequency or
microwave ablation). Using a percutaneous approach, a needle is used to ablate the tumor with
heat or freezing. After ablation, all patients should undergo surveillance to assess for residual
or recurrent tumor that would require additional therapy. Although initial studies have shown
favorable shortterm results with thermal ablation, long-term outcomes are yet to be studied.
Thermal ablation is reserved for patients with a small RCC who are not candidates for surgery.
In addition, ablation is a treatment option for RCC after partial nephrectomy, those with a
unilateral or transplanted kidney, or for patients at risk for multiple RCCs over their lifetime.

134. A 77-year-old man presents to your clinic with vague abdominal pain. Among initial
laboratory tests, hematuria is notable. A repeated urinalysis confirms hematuria. You
order a four-phase contrasted abdominal CT scan that shows a 1cm complex mass in the
lower pole of the right kidney. He is otherwise asymptomatic, enjoys a healthy life, and do
not have any comorbidities. Which one of the following is the most appropriate inital
management option for him?
A Repeat the CT scan in 12 months.
B Nephrectomy.
C CT-guided percutaneous biopsy.
D Reassurance.
E Thermal ablation.
Option B is correct Abdominal pain and hematuria in the presence of a renal mass is
renal cell carcinoma (RCC) until proven otherwise. Small renal masses (<4cm) can be
found incidentally, or during evaluation for urinary symptoms, often by ultrasound as the initial
imaging modality. Once a mass is found in the kidney on ultrasound, a four-phase contrasted
CT scan should follow for further evaluation if contrast media is not contraindicated. Solid or
complex masses (a lesion demonstrating both cystic and solid structures), as well as cystic
masses enhanced with contrast are suspected to be malignant and treatment is warranted.
NOTE - By definition, a renal lesion <4cm in its largest diameter that shows contrast
enhancement on abdominal imaging is a small renal mass. These lesions can be solid or
complex cystic. Simple cysts not enhancing with contrast are excluded. If the patient is a good
candidate for surgical resection and has a life expectancy >5 years, surgical resection of the a
small renal mass would be the most appropriate initial management. This patient is otherwise
fit and healthy with no comorbid condition precluding him from surgical tumor resection.
Partial nephrectomy is the procedure of choice for masses less than 7cm that are not centrally
located. This is to preserve as much renal function as possible. (Option A) Active surveillance
with CT scan or MRI in 6- to 12-monht intervals is recommended for patients with small renal
masses who are not good candidates for surgical resection (e.g. have a comorbid condition), or
have a life expectancy of ≤5 years. Renal impairment may also be an indication for active
surveillance in some patients. Active surveillance means that the patient will either have
delayed treatment or no treatment at all. This patient has no comorbid condition precluding him
from tumor resection. (Option C) The role of renal biopsy is controversial in the setting of a
small renal mass. In particular there is disagreement as to whether it is necessary to biopsy
these lesions before planning management and in what settings. Circumstances under which a
renal biopsy might be considered are where the patient is not a surgical candidate
(controversial), life expectancy is ≤5 years (controversial), or the patient requests a definite
diagnosis before proceeding with the surgery. Biopsy is not recommended for patients who are
candidates for active surveillance. (Option D) With this clinical picture, the chances are that
the tumor is malignant; therefore, reassurance cannot be given unless investigations exclude
malignancy. (Option E) Ablation of renal mass can be performed with either freezing
(cryoablation) or heat (radiofrequency or microwave ablation). Using a percutaneous approach,
a needle is used to ablate the tumor with heat of freezing. After ablation, all patients should
undergo surveillance to assess for residual or recurrent tumor that would require additional
therapy. Although initial studies have shown favorable shortterm results with thermal ablation,
long-term outcomes are yet to be studied. Thermal ablation is reserved for patients with a small
RCC who are not candidates for surgery. In addition, ablation is a treatment option for RCC
after partial nephrectomy, those with a unilateral or transplanted kidney, or for patients at risk
for multiple RCCs over their lifetime

135. A 75-year-old man presents to your practice after he noticed blood in his urine. A
four-phase contrast CT scan is obtained that shows a 1cm mass in the left kidney, highly
suggestive of renal cell carcinoma. His medical history is significant for acute myocardial
infarction 1 month ago for which he underwent angioplasty and drug-eluting stent
placement. Currently, he is on dual antiplatelet therapy (DAPT) with daily clopidogrel
75mg and aspirin 100mg. Which one of the following would be the most appropriate next
step in management ?
A Nephrectomy.
B Heminephrectomy.
C Repeat the CT scan in 12 months.
D CT-guided percutaneous biopsy.
E Repeat the CT scan in 3 months.
Option C is correct Small renal masses can be found incidentally, or during evaluation for
urinary symptoms, often by ultrasound. Once a mass is found in the kidney on ultrasound, a
four-phase contrasted CT scan should follow for further evaluation, provided renal function
allows use of contrast media. These four phases include: (1) arterial, (2) corticomedullary, (3)
nephrographic and (4) excretory phases. This study allows a detailed examination of each
aspect of the functional anatomy of the kidney. The majority of renal lesions are benign simple
cysts, which would require no further work-up. However, completely solid, mixed (solid and
cystic), and cystic renal lesions that enhance with contrast are likely to be malignant. A small
renal mass is defined as a mass <4cm that has enhancement on abdominal contrast
imaging. For such lesions, surgical resection of the mass is the most appropriate option if
life expectancy is >5 years and the patient is a good candidate for surgery. However, this
patient has had an MI one month ago. Major surgery within the first 6 months after MI is
associated with high risk of mortality; therefore, this patient is not a candidate for
surgical resection of the mass. In such cases active surveillance of a small renal mass with
CT or MRI in 6- to 12-month intervals is recommended. Even if a small renal mass has
imaging characteristics highly suspicious for RCC, active surveillance may be appropriate,
particularly in patients with medical comorbidities that can increase the risk of active
intervention such as surgery, in elderly patients, and in those with decreased life expectancy
(≤5 years). Renal impairment may also be an indication for active surveillance in some patients.
Active surveillance means that the patient will either have delayed treatment or no treatment at
all. NOTE - Active surveillance is not generally recommended for healthy younger patients
because while there is some evidence to suggest such approach if the lesion is less than 1 cm;
more data is needed before adopting this as a standard protocol in this patient group. Another
issue of concern when considering active surveillance in young patients, is the number of scans
they will require over a lifetime, which is associated with considerable exposure to radiation.
(Options A and B) Tumor resection is the management of choice for small renal masses that
are suspected of malignancy. Such masses are solid, mixed solid, or cystic renal lesions that
enhance with contrast. If possible, nephron-sparing surgery, or partial nephrectomy rather than
a total nephrectomy is performed to allow for preservation of renal function. However, in the
following situation total nephrectomy is the treatment of choice: Tumor size ≥7 cm Those
tumors that have a more central location Suspected lymph node involvement Tumor with
associated renal vein or inferior vena cava (IVC) thrombus Direct extension into the ipsilateral
adrenal gland NOTE - even in the presence of the above, patients with any of the following
conditions must have partial rather than total nephrectomy: A solitary kidney Multiple, small,
and/or bilateral tumors Patients with or at risk for chronic renal disease (Options D) The role
of renal biopsy is controversial in the setting of a small renal mass. In particular there is
disagreement as to whether it is necessary to biopsy these lesions before planning management
and in what settings. Circumstances under which a renal biopsy might be considered are where
the patient is not a surgical candidate (controversial), life expectancy is ≤5 years (controversial),
or the patient requests a definite diagnosis before proceeding with the surgery. Biopsy is not
recommended for patients who are candidates for active surveillance. NOTE - With available
evidence, it may be safe to assume that routine discontinuation of the antiplatelet therapy is not
necessary in low risk patients planned to undergo percutaneous renal biopsies. (Option E)
Recommended interval for active surveillance with CT scan or MRI is 6-12 months, not 3
months.

136. A 24-year-old woman presents concerning about the risk of breast cancer after her
mother was recently diagnosed with breast cancer and was found to have BRCA1 gene
mutation. She is keen to know if she should undergo genetic testing for BRCA1 and the
expenses. Which one of the following is correct in this regard?
A She can be screened at her own expense.
B She can be screened at her employer’s expense.
C Screening for her is funded by the government.
D She does not require screening.
E She can be screened at the expense of Medicare.
Option C is correct Despite the fact that a family history of breast and/or ovarian cancer is
common in women with breast or ovarian cancer, only less than 10% of all breast cancers and
less than 15% of ovarian cancers are associated with inherited genetic mutations. Mutations in
BRAC1 and BRCA2 account for the majority of hereditary breast/ovarian cancers. Less
common mutations are TP53 (Li-Fraumeni syndrome) and PTEN (Cowden syndrome).
Hereditary syndromes associated with BRCA1 and BRCA2 are inherited in an autosomal
dominant fashion, meaning that 50% of offspring of a person with such mutations will have the
faulty gene. There are national guidelines that are used when offering publically funded BRCA
gene testing in Australia. It is generally accepted that when offering BRCA testing in a family
for the first time, the person who is offered testing must be either affected by a relevant cancer
(usually breast or ovarian cancer) and have a more than 10% likelihood of a gene mutation, or
if unaffected, have more than a 20% chance of a gene mutation. This woman is unaffected, and
has 50% of chance of having a BRCA1 gene mutation as mentioned above. In other words
offspring of a person with documented BRCA mutation are eligible for BRCA gene testing at
the expense of government. BRCA1 and BRCA2 testing is not subsidized by Medicare. and
these tests are done through the public hospital system, funded by state health department. The
following conditions are also eligibility criteria for BRCA gene testing funded by public health
system: Breast cancer <30yr; triple negative breast cancer <40yrs*, ovarian cancer (high grade
serous, endometrioid or clear cell) <70yrs Males with breast cancer < 60yr would usually be
eligible for testing while older men require additional family history Other (usually eligible):
Ashkenazi Jewish heritage and either a personal or family history of breast or ovarian cancer
An affected woman with a family history of breast and ovarian cancer Three first or second
degree relatives with breast cancer, especially if <50yrs Bilateral breast cancer with the first
cancer occurring in the 30’s or 40’s Affected patients whose family history scores >16 on the
Manchester risk score TOPIC REVIEW The following screening strategy is recommended by
expert groups for women with hereditary breast/ovarian cancer (HBOC) syndrome who have
not undergone risk-reducing surgery (i.e. bilateral mastectomy, oophorectomy): Monthly breast
self-examination beginning at age 18 Clinical breast examination 2 to 4 times annually
beginning at age 25 Annual mammography and breast magnetic resonance imaging (MRI)
screening (commonly alternated every 6 months) beginning at age 25 or individualized based
on the earliest age of onset in the family Twice yearly ovarian cancer screening with
transvaginal ultrasound and serum CA-125 levels (preferably drawn on day 1 to 10 of
menstrual cycle for premenopausal women) beginning at age 35, or 5 to 10 years earlier than
the earliest age of first diagnosis of ovarian cancer in the family NOTE - The lack of efficacy
of ovarian cancer screening has prompted many clinicians to recommend riskreducing bilateral
salpingo-oophorectomy at the completion of childbearing rather than intensified screening for
ovarian cancer. *Any breast cancer than does not express the genes for estrogen receptor,
progesterone receptor (PR) or Her2/neu

137. Which one of the following tests is most likely to differentiate hypercalcemia caused
by hyperparathyroidism and cancer?
A PTH.
B Calcium level.
C Phosphate.
D Alkaline phosphatase.
E Vitamin D level.
Option A is correct Primary hyperthyroidism and malignancy are the most common causes of
hypercalcemia, accounting for more than 90% of cases. Primary hyperthyroidism is the more
common of these two. It is usually not difficult to differentiate between them. Malignancy is
often evident clinically by the time it causes hypercalcemia, and patients with hypercalcemia of
malignancy have higher calcium concentrations and are more symptomatic from hypercalcemia
in individuals with primary hyperparathyroidism. However, it may be difficult to differentiate
the two problems clinically when the presentation is less typical. As an example, some patients
with occult malignancy may present with mild hypercalcemia. Alternatively, patients with
hyperparathyroidism can occasionally have acute onset of severe, symptomatic hypercalcemia
(parathyroid crisis). In such cases, measurement of intact PTH usually distinguishes these two
conditions. Intact PTH concentrations are generally undetectable or very low in hypercalcemia
of malignancy and are elevated or high-normal in primary hyperparathyroidism. It is
uncommon for patients with hypercalcemia of malignancy to have elevated PTH levels, but this
finding may occur rarely in patients with hypercalcemia of malignancy and concomitant
primary hyperparathyroidism or in individuals with PTH secreting tumors, which are also rare.

138.A 55-year-old man has been on warfarin for AF for the past 3 months. He presented
with an incarcerated inguinal hernia and was booked for emergent surgery. Warfarin
was stopped and fresh frozen plasma was given. Which one of the following is the time to
resume warfarin therapy?
A 12 hours post-op.
B 48 hours post-op.
C Immediately after recovery from anesthesia.
D 5 days post-op.
E When INR is less than 1.8 again.
Option A is correct For those, whose warfarin therapy has been stopped before major surgical
procedures, it is recommended that the previous maintenance dose of warfarin be resumed on
the night of surgery (12-24 hours). In addition to warfarin, low molecular weight heparin
(LMWH) in prophylactic dose or unfractionated heparin (UFH) with slow infusion is started at
the same time. The target APTT is 1.5 times the normal. LMWH or UFH is continued for at
least 5 days and is ceased 48 after the target INR is reached (≥1.8).

139. An 83-year-old man fell off a slippery toilet bowl in the bathroom while trying to
reach toilet paper, and had his left femoral neck fractured. He was treated with open
reduction and internal fixation. Today, he is being discharged. Which one of the following
pieces of advice is most important to give?
A He should start alendronate.
B He should have a bone scan.
C He should take low molecular weight heparin (LMWH) for 6 weeks.
D He should take warfarin for 6 months.
E He should take supplemental calcium and vitamin D.
Option A is correct Femoral neck fracture following falling off a short height is in fact a
fracture following a minor trauma. Minor (low) trauma fractures are always pathologic.
Osteoporosis is the most common cause of pathologic fractures. Unlike in women, osteoporosis
in man is often secondary and a manifestation of an underlying cause. In men with clinical
manifestations of osteoporosis e.g. low trauma fracture or with low bone mass (T-score less
than -2.0), an initial evaluation for secondary causes of osteoporosis should be performed.
Conditions to consider are renal or liver disease, hyperparathyroidism, Cushing's syndrome,
celiac disease and other forms of malabsorption, or hypercalciuria. The initial evaluation
should include routine biochemical tests to for renal or hepatic disease, a full blood count
(FBC), serum testosterone, calcium, alkaline phosphatase, 25-hydroxyvitamin D , and 24-hour
urine calcium. I addition to assessment for the underlying cause and treating them if possible,
patients with pathological fractures due to osteoporosis, should be started on osteoporosis-
specific treatment for prevention and treatment of osteoporosis. Bisphosphonates are (e.g.
alendronate, residronate, zolendronic acid) are first-line treatment options for this purpose and
the best advice to give for prevention of further fractures and bone density enhancement.
(Option B) Although it is recommended that bone mineral density scan be measured, treatment
can be started without a bone scan. (Option C) Patients undergoing hip fracture surgery are in
the highest risk category for development of venous thromboembolism (VTE), on the basis of
the procedure itself. Without thromboprophylaxis, reported rates of VTE are high following hip
surgery. This necessiates thromboprophylaxis following surgery for all patients admitted to
hospital with hip fracture. for up to 35 days starting 12 hours after the surgery. Although,
thromboprophylaxis is more vital than alendronate, it has already been started post-operatively
and continued during hospital stay, which is often 10-14 days. Six weeks of anticoagulation
with LMWH is beyond the maximum duration recommended and not a correct option. (Option
D) Warfarin may be less effective than LMWH, and it has the drawback of the need for INR
monitoring; however, patients who wish to avoid the discomfort of a twice daily injection may
be more compliant with warfarin. 6 months, however, is way beyond the recommeded duration
of prophylaxis. (Option E) Calcium and vitamin D supplementation should be advised if
dietary calcium and vitamin D are inadequate. Supplementation with calcium and vitamin D
has not shown to reduce the risk of further fractures in the future.

140.Daniel, a 78-year-old patient of yours, presents for consultation. He has a fall last
month and had his femoral neck fractured. He underwent surgery and intramedullary
nail placement. The fracture occurred when the chair leg broke and he fell off onto the
ground from a height of approximately 70cm. In laboratory assessment, significant
findings are a testosterone level of 8ng/dL (normal:8- 50ng/dL) and vitamin D of
12mmol/L (≥75mmol/L). Serum calcium is normal, as are other laboratory parameters
including liver and renal function tests. Which one of the following is the most important
management to consider as the next step?
A Alendronate 70mg, once weekly.
B Calcium 1000mg, daily plus vitamin D 600IU, daily.
C Calcitriol 0.25mg, daily.
D Testosterone replacement therapy.
E Adequate sun exposure.
Option C is correct The femoral neck fracture caused by falling off a short chair, as described,
is in fact a fracture following a minor (low) trauma. This is always pathologic. Osteoporosis is
the most common cause of pathologic fractures. Unlike in women, osteoporosis in men is
almost always secondary and a manifestation of an underlying disease. Therefore, in men with
clinical manifestations of osteoporosis e.g. low trauma fracture or with low bone mass by
DEXA (T-score less than -2.0), an initial evaluation for secondary causes of osteoporosis
should be performed. Conditions to consider are renal or liver disease, hyperparathyroidism,
Cushing's syndrome, celiac disease and other forms of malabsorption, or hypercalciuria. The
initial evaluation should include routine biochemical tests for renal or hepatic disease, a full
blood count (FBC), serum testosterone, calcium, alkaline phosphatase, 25-hydroxyvitamin D ,
and 24-hour urine cortisol. Based on the test results, Daniel is suffering from severe vitamin D
deficiency and vitamin D should be replenished as the most appropriate next step in
management. The treatment option of choice is calcitriol 0.25mg, daily. (Option A)
Bisphosphonates are medications of choice for treatment of osteoporosis and prevention of
further fractures in patients with osteoporosis, whose osteoporosis is not due to hypogonadism
(low testosterone levels), or those with hypogonadism in whom testosterone replacement
therapy is contraindicated. Indications are a T-score or less than -2.5 or history of pathologic
fracture regardless of the T-score. The two most widely used drugs are alendronate and
residronate. Alendronate should be given 70 mg, weekly (or 10mg, daily). It is, however,
necessary that vitamin D be corrected to a level of above 50nmol/L before therapy with
bisphosphonates is started. (Option B)This patient has normal serum calcium levels and does
not need calcium supplementation now. Moreover, calcium should not be added to calcitriol
therapy due to the risk of hypercalcemia. Dietary calcium intake should be maintained at 800 to
1000 mg, daily. (Option D) Testosterone replacement therapy is the treatment of choice in
osteoporotic men with hypogonadism. Daniel’s testosterone level is in the lower limit normal
range. Judged by his testosterone level, he is unlikely to have hypogonadism. (Option E)
Adequate sun exposure should be recommended to all patients as a means of preventing
vitamin D deficiency. Daniel has severe vitamin D deficiency and needs to be treated more
aggressively rather than just with sun exposure.

141. A 55-year-old woman presents to your practice after noticing a clear nipple
discharge from her right breast. On physical examination, no palpable mass is found in
her breasts. Both nipples appear normal, however, pressure over the right areola causes
discharge of clear fluid from the nipple. Which one of the following could be the most
likely diagnosis?
A Benign ductal papilloma.
B Paget disease of the breast.
C Ductal ectasia.
D Intraductal breast carcinoma.
E Breast fibroadenoma.
Option A is correct Nipple discharge is always a abnormal finding except in late pregnancy or
the postpartum period. Based on characteristics, there are 7 types of nipple discharge: 1. Milky:
white discharge; sometimes fat globules are seen under microscopy 2. Multicolored gummous:
sticky discharge 3. Purulent: pus with white cells seen under microscopy 4. Watery: colorless
discharge 5. Serous: faintly yellow, thin discharge 6. Serosanguineous: thin, clear discharge
with pink tint, RBCs seen under microscopy 7. Bloody (sanguinous): pure blood Conditions
associated with each type of discharge are as follows: Milky Discharge Galactorrhea, or
nonpuerperal lactation, usually results from multiple duct discharge from both breasts. The
most common cause of nonpuerperal lactation is hyperprolactinemia associated with pituitary
adenomas, medications or other causes resulting in increased production of prolactin (e.g.
primary hypothyroidism). In many women, galactorrhea can be idiopathic. Multicolored and
Sticky Discharge Duct ectasia or comedomastitis is the most common cause of a multicolored,
sticky discharge that is commonly bilateral in perimenopausal woman. It begins as a dilation of
the terminal ducts with an irritating lipid fluid collection and producing an inflammatory
reaction resulting in discharge from the nipple. Duct ectasia is most frequently associated with
pain, itching, and swelling in the nipple. Palpation of the areola can often reveal a tubular mass,
reflecting the dilated ducts. Often a history of nipple manipulation is elicited. If the disease
progresses, a mass can develop (plasma cell mastitis) that can mimic cancer. Surgery is
indicated only if a mass forms or the discharge changes to serosanguinous or bloody. Purulent
Discharge In patients with acute puerperal mastitis, chronic lactation mastitis, central breast
abscesses, or plasma cell mastitis, nipple discharge is purulent and usually unilateral. Breast
cultures and smears may reveal an organism responsible. Abscess formation usually requires
incision and drainage if appropriate antibiotics and warm compreses are not effective. It is
important to remove a portion of an abscess wall for histologic study to exclude an underlying
cancer associated with secondary necrosis and infection. Watery, serous, serosanguinous, and
bloody Discharges Intraductal papillomas are the most common cause of these discharges, but
fibrocystic disease, advanced duct ectasia, breast cancer, and vascular engorgement in near-
term pregnancy can also be the causes. In patients over 50, malignancy becomes increasingly
common, especially if the discharge is unilateral and associated with a mass. Surgical
exploration is mandatory in this group of patients with this type of discharge, even if cytologic
and mammographic findings are negative. NOTE - surgical referral is required if the nipple
discharge is spontaneous and unilateral, or the patient is older than 60 years. (Option B) Paget
disease of the breast most commonly presents with a scaly, raw, vesicular, or ulcerated lesion
that begins on the nipples and then spreads to the areola. Nipple discharge, if present at all,
tends to be bloody rather than clear. (Option C) Ductal ectasia presents with multicolored and
sticky discharge. Toothpaste like discharge is a classic description of discharge associated with
duct ectasia. (Option D) With unilateral clear discharge in this woman, the most likely cause is
a benign Intraductal papilloma. Although advanced duct ectasia and intraductal carcinoma are
among other etiologies, they are less likely to be the cause compared with Intraductal papilloma.
(Option E) Fibroadenoma does not cause nipple discharge.

142 . 48 A 50-year-old man presents with complaints of several episodes of painless gross
hematuria and a mass he felt in his left loin recently. The significant finding on
examination is a non-tender loin mass. Urine analysis shows blood in the urine. Urine
culture is negative. Which one of the following is the most appropriate next step in
management?
A CT scan.
B Cystoscopy.
C MRI. pyelography (IVP).
Option A is correct Gross hematuria is always concerning and warrants thorough
investigations, because the prevalence of urinary tract malignancies among patients with
macroscopic hematuria has been reported to be as high as 19%, but usually ranges from 3-6%.
Risk factors can help in determining which patients are at higher risk of urinary tract and
bladder malignancies. Risk factors include: Age >40 years A history of smoking History of
gross hematuria History of chronic cystitis or irritative lower urinary tract symptoms (e.g.
frequency, urgency, dysuria, nocturia, hesitancy, sensation of incomplete emptying) History of
pelvic irradiation Exposure to occupational chemical and dyes (e.g. heavy phenacetin use,
treatment with high doses of cyclophosphamide, aristolochic acid) One of the most common
causes of hematuria is urinary tract infection, which can be investigated with urine microscopy,
culture and sensitivity (MC&S). Ureteric and renal stones are another common cause, but these
typically present with pain and microscopic or gross hematuria. In cases where macroscopic
hematuria or risk factors are present, or if another cause cannot be determined, more extensive
investigations are recommended to exlcude an underlying malignancy. Despite extensive
investigation, studies have shown that in up to 50% of patients with macroscopic hematuria and
70% with microscopic hematuria have no identifiable cause is found. This could be attributed
to transient benign physiological conditions, including vigorous physical exercise, sexual
intercourse or menstrual contamination. In this patient, urinalysis is negative but for blood.
There is no increase in WBC to indicated infection; furthermore, a urine culture is negative,
excluding infections as a cause. There are also no red cell casts or other pathologic findings
indicating conditions such as glomerulonephritis as the underlying pathophysiology. An
enlarged or otherwise abnormal prostate would have come to attention on physical exam.
Given the presence of gross hematuria, the age (>40) and the loin mass, the most important
concern would be a renal tumor until proven otherwise. Such patients should undergo non-
contrast CT scan of kidney, ureter and bladder (KUB) (firstline) or ultrasound (second-line).
CT-KUB is a non-contrast study – the current gold standard in identifying ureteric and renal
stones, with 94-98% sensitivity; however, the sensitivity is much lower for indentifying
genitourinary malignancies. CT scan findings associated with increased risk of urinary tract
malignancies justify CT-KUP IVP. CT-KUB IVP (multidetector CT scanning of kidneys,
ureter and bladder after intravenous contrast media is administered), also known as CT
urography (CTU) is as accurate and diagnostic as a combination of ultrasonography, IVP and
CT-KUB. This, however, should be performed by,or at least under consultation with an
urologist. Sensitivity for detecting pathology in patients with hematuria varies from 94-100%,
with a 97.4% specificity. With the loin mass, the cause of hematuria is more likely to originate
from upper parts of urinary tract that are beyond the reach of cystoscopy. Cystoscopy, however,
can be considered somewhere along the diagnostic pathway, for patients with voiding
symptoms or where there is suspicion of bladder cancer, but not as the next best step in this
scenario. IVP (intravenous pyelogram), also called IVU (intravenous urogram) is inferior to CT
scan.

143.A 56-year-old woman is brought to the emergency department with sudden-onset


severe epigastric pain. On examination, she has a blood pressure of 90/55mmHg, heart
rate of 110bpm, and respiratory rate of 22 breaths per minute. There is abdominal
guarding and rigidity, as well as marked tenderness and rebound tenderness over the
epigastric area. A chest X-ray reveals free air under the right hemi-diaphragm. Based on
clinical findings, a perforated peptic ulcer is diagnosed and the patient is planned for
emergency laparotomy. She is on warfarin due to deep vein thrombosis (DVT) that
developed 2 weeks ago. Which one of the following is the most appropriate next step in
management?
A Stop warfarin, give vitamin K and do the surgery.
B Proceed with the surgery.
C Give fresh frozen plasma (FFP) and proceed with the surgery.
D Add intravenous heparin and perform the surgery.
E Stop warfarin, give her heparin and proceed with the surgery.
Option C is correct This patient is in need of emergency life-saving surgery in the setting of
perforated peptic ulcer disease and clinical manifestation of peritonitis (abdominal rigidity and
guarding). In the event of need for life-saving or emergency surgeries where deferring the
surgery is not possible, warfarin effect should be reversed immediately using prothrombin
complex concentrate (CCP) (Prothrombinex-VF®), or fresh frozen plasma (FFP) if CCP
unavailable, regardless of the risk of potential VTE if warfarin is temporarily stopped. Any
option offering addition of vitamin K to cessation of warfarin and administration of CCP (or
FFP) woudl be the most appropraite management (not an option). (Option A) Cessation of
warfarin and administration of vitamin K was the method of choice if the patient had an INR of
2-3 within the past 2-4 weeks and there was a time window for deferral of the surgery for at
least 24 hours. In such cases, vitamin K is given intravenously the evening before the surgery
and INR is checked on the day of the surgery. An INR of ≤1.5 is safe to proceed with the
surgery. If INR is >1.5 Prothrombinex-VF® should be given (FFP is used if Prothrombinex-VF
is not available) (Option B) Proceeding with the surgery without warfarin reversal is associated
with significant risk of intra- or post-operative bleeding and not recommended. (Option D)
Addition of heparin adds to the risk of bleeding. (Option E) Cessation of warfarin and bridging
with heparin is indicated in patients with high risk of VTE, who are planned for elective
surgery. The protocol is not used for emergency procedures. TOPIC REVIEW Management
of patients on warfarin therapy who are undergoing an invasive procedure is according to the
following table:check table on page 2884
144. A 66-year-old man presents with altered bowel habits, decreased stool caliber and
rectal bleeding in the form of blood covering the stool. Investigations show that he has a
colorectal cancer. He is planned for surgical tumor resection. Currently, he is on warfarin
due to atrial fibrillation (AF) and has an INR of 2.5. Which one of the following is the
most appropriate option to consider for warfarin reversal prior to the surgery?
A Proceed with the surgery.
B Give vitamin K before the surgery.
C Give fresh frozen plasma (FFP) and proceed with the surgery.
D Wait for 3 months. E Switch to clopidogrel and perform the surgery in one week. Option B
is correct This patient, as a candidate for a surgical procedure, is at increased risk of intra- or
postoperative bleeding due to warfarin therapy. Resection of a colonic tumor is an elective
procedure that allows for planned action for warfarin reversal. In those surgical candidates with
an INR of 2-3, cessation of warfarin 4-5 days before the surgery and administration of vitamin
K (3mg, intravenously) the evening before the surgery is the preferred plan. (Option A)
Proceeding with a major surgery without warfarin reversal is associated with a significantly
increased risk of bleeding and not appropriate. (Option C) FFP was the correct answer if this
patient required emergency surgery. Prothrombinex-VF (or if unavailable FFP) is used for
rapid reversal of warfarin effect (in minutes) for urgent surgical procedures associated with
increased risk of bleeding. Using FFP for warfarin reversal in elective procedures is not
appropriate. (Option D) Deferring the surgery for 3 months is unnecessary and inappropriate
because warfarin reversal can be achieved in 5 days, maximum. There is also no other medical
condition to preclude surgery in this patient. (Option E) Clopidogrel is an antiplatelet
medication. Switching to clopidogrel not only does not counteract the effect of warfarin, but it
also adds to the risk of bleeding by inhibiting platelet aggregation. TOPIC REVIEW
Management of patients on warfarin therapy who are undergoing an invasive procedure is
according to the following table: page 2896

145. A 73-year-old man presents to the emergency department with fracture of the right
femoral neck after he sustained a fall at home. He underwent coronary artery drug
eluting stent placement 2 months ago, and has been on clopidogrel since then. A full blood
count (FBC) is normal; however, he has several bruises over his body. He requires
surgery for fixation of the fracture. Which one of the following is the most appropriate
management of this patient?
A Stop clopidogrel and proceed with surgery in one week.
B Do the surgery now.
C Administer fresh frozen (FFP) plasma and proceed with the surgery.
D Give platelets and proceed with the surgery.
E Switch to heparin and perform the surgery in 7 days.
Option B is correct Femoral neck fracture requires emergency surgical fixation, because other
measures such as traction or rest pose a more significant risk to the patient than does the risk of
bleeding associated with antiplatelet therapy. Patients for whom surgery cannot be deferred
should be operated on immediately. Any option suggesting deferral of the surgery is
inappropriate. On the other hand, this patient has undergone coronary stenting with drug-
eluting stent and requires at least 12 months of antiplatelet therapy. Cessation of antiplatelet
therapy is associated with significantly increased risk of stent thromobosis and grave
complications; hence antiplatelet therapy should be continued. He should undergo emergecny
surgery while he is on clopidogrel. Of note, an Australian retrospective review of 181 patients
with proximal femoral fracture demonstrated no significant difference in the amount of
bleeding, transfusion requirement, complications rate, or length of stay in 16 patients taking
clopidogrel and in 48 taking aspirin compared to others. (Options A and E) Deferral of an
emergency surgery is incorrect. (Option C) FFP reverses the effect of heparin and warfarin. It
has no effect on inhibited action of platelets. (Option D) Platelet transfusion might be
considered in selected patients in whom the risk of major bleeding clearly outweighs the
benefits of counteracting the antiplatelet therapy. However, transfusion of platelets in this
patient may result in stent thrombosis and is not advisable. The only exception is when the risk
of bleeding is so high that remarkably outweighs the risk of stent thrombosis. TOPIC
REVIEW Coronary stent thrombosis is an uncommon but clinically devastating complication
of coronary artery stenting that usually results in significant risk of myocardial infarction or
death. Approximately 40% of reported cases have occurred in the context of non-cardiac
surgery (NCS) performed in patients with coronary artery stents, in whom dual antiplatelet
therapy or clopidogrel alone has been ceased. In patients with coronary disease, cessation of
aspirin or clopidogrel is associated with an approximate 2-3 fold increase in subsequent death
or myocardial infarction. This risk is further elevated in patients with intracoronary stent and is
of added concern because the dramatic consequences of stent occlusion. There is uncertainty
regarding the risks of stent thrombosis in individual patients, and in particular how to balance
this risk against that of surgical complications if antiplatelet therapy is continued throughout
the perioperative period. The following are current recommendations regarding antiplatelet
agents and non-cardiac surgery: Elective non-cardiac surgery should be deferred for at least 6
weeks and ideally 3 months following PCI with bare metal stenting. Elective surgery should be
deferred for 12 months following drug eluting stent because of an increased risk of
death/myocardial infarction or stent thrombosis. Despite the observation that dual antiplatelet
therapy increases the likelihood of bleeding for most surgical procedures, the consequences of
bleeding are less significant than those of stent thrombosis. The risk benefit ratio would favor
continuation of aspirin in most patients and dual antiplatelet therapy in many patients with prior
coronary artery stenting who are undergoing non-cardiac surgery. Exceptions to this include
patients undergoing spinal, intracranial, extraocular TURP or major plastic reconstructive
procedures. For these surgeries, patients at low risk of stent thrombosis should have their
antiplatelet therapy routinely ceased perioperatively. In patients with coronary artery disease in
whom no stent has been placed, antiplatelets can be stopped 7 days before the surgery. In the
event of emergency, surgery should be proceeded to without cessation of antiplatelet therapy.
platelet transfusion might be considered in selected patients in anticipation or occurrence of
major bleeding. In other situations THAN bare metal stents and drug-eluting stents placed
within the past 6 weeks and 12 months respectively, antiplatelet medications should be stopped
1 week (7 days) before the procedure.

146. A 67-year-old man in brought to the emergency department with sudden-onset


severe headache and confusion. He was diagnosed with atrial fibrillation (AF) 4 months
ago, for which he has been on warfarin ever since. Blood studies show an INR of 3.5. A
CT scan of the head reveals intracerebral hemorrhage. Which one of the following is the
most appropriate next step in management?
A Stop warfarin.
B Vitamin K.
C Fresh frozen plasma (FFP).
D Increase the dose of warfarin.
E Reduce the dose of warfarin.
Option C is correct Bleeding is the most common complication of warfarin therapy and is
related to the INR value. Although incremental rises in INR increase the risk of bleeding, most
intracranial bleed occurs in patients with an INR in the therapeutic range. Such events occur in
0.5-1.0% of patients with AF per year. In the event of major or life-threatening bleeding
associated with warfarin use, prothrombin complex concentrate (PCC) with or without FFP is
the most appropriate next step in management. FFP in large volumes should be used if PCC is
not available. (Option A) Cessation of warfarin should be considered in all patients with
bleeding; however, cessation of warfarin alone is not going to change the immediate manage
plan that is reversing its effects immediately. (Option B) Vitamin K is an effective antidote for
the anticoagulation effect of warfarin. Despite this, data are lacking to show that its use
improves outcome in life-threatening bleedings. The usual dose is 5-10mg administered orally
or intravenously. Intravenous route achieves a more rapid response compared with oral
administration, with an onset of action between 6-8 hours. However, both routes achieve a
similar correction of INR by 24 hours. In major or lifethreatening bleeds, 5-10mg of vitamin K
should be given in conjunction with PCC or FFP to sustain the reversal effect. Vitamin K is the
treatment of choice for patients on warfarin therapy with bleeding in whom the aim is to
normalize the INR (not immediate counteraction against major organ bleeding or
lifethreatening bleeding), vitamin K given intravenously is the preferred treatment. (Option D)
Increasing the dose of warfarin worsens the condition and not a correct option. Reducing the
dose of warfarin can lead to decreased INR in long-term. It does not counteract the bleeding.
(Option E) While the patient is bleeding, dose reduction of the warfarin is not an appropriate
option. Dose reduction is an option for patients with INRs beyond the target but no active
bleeding. TOPIC REVIEW Prothrombin complex concentrate (PCC) comes in two forms -
one formulated with 3 factors (II, IX and X) and the other with 4 factors (II, VII, IX and X).
Advantages of PCC over FFP include rapid reconstitution into a small volume for infusion over
20–30 minutes, fast onset of action (10-15 minutes), no requirement to check the patient’s
blood group, minimal risk of viral transmission due to pathogen reduction and inactivation
steps during manufacturing, and reduced risk of other clinical adverse reactions such as
transfusion-associated circulatory overload or transfusion-associated acute lung injury.
Prothrombinex-VF®, a 3-factor PCC, is the only product currently in routine use in Australia
and New Zealand for warfarin reversal. However, since the efficacy of Prothrombinex-VF®
alone has not been extensively evaluated for patients with major or life-threatening bleeding, or
INR > 10, it is currently recommended that Prothrombinex-VF® be supplemented with FFP for
addition of adequate amounts of factor VII to ensure optimal reversal thrapy when major or
life-threatening bleeding is a concern. For life-threatening (critical organ) and clinically
significant bleedings, the consensus is to use the maximum dose of Prothrombinex-F® (with
vitamin K1 and FFP) and the maximum amount of FFP when Prothrombinex-VF is unavailable.
Recommendations for managment of patients on warfarin therapy with increased INR and
bleeding are summarized in the following table: check page 2905
147. An 80-year-old man is planned for emergency surgery due to a strangulated direct
hernia. He has been on clopidogrel after he sustained an acute myocardial infarction 10
months ago. Which one of the following is the next best in management prior to the
surgery?
A Give platelets and proceed to the surgery.
B Give DDAVP and proceed to the surgery.
C Give cryoprecipitate and proceed to the surgery.
D Stop clopidogrel and proceed to the surgery.
E Give vitamin K intravenously.
Option A is correct Strangulated hernias are genuine emergencies requiring immediate
surgery. Failing to do so can lead to bowel perforation, peritonitis and death. This patient,
however, is on clopidogrel and has an increased risk of intra- or post-operative bleeding due to
effect of clopidogrel on platelet function. Since abdominal surgery is associated with marked
risk of bleeding in this patient, platelets should be given prior to the surgery to counteract the
effect of clopidogrel. If this patient had a coronary stent, antiplatelet therapy should have not
been ceased. Exceptions to this include patients with coronary stents who are undergoing spinal,
intracranial, extraocular TURP or major plastic reconstructive procedures. For these operations,
patients at low risk of stent thrombosis should have their antiplatelet therapy routinely ceased
perioperatively. (Option B) Desmopressin (DDAVP), often in conjunction with fresh frozen
plasma, is used for preoperative management of patients with Von-Willebrand disease in whom
there is deficiency of factor VIII. (Option C) Cryoprecipitate replenish coagulation factors
with no effect on platelet activity inhibited by clopidogrel. (Option D) It takes approximately 7
days for effects of antiplatelet drugs to suitably diminish. Stopping clopidogrel now will not
decrease the chance of bleeding immediately and is not useful. (Option E) Vitamin K is used
for warfarin reversal and has no effect on platelet function. TOPIC REVIEW Coronary stent
thrombosis is an uncommon but clinically devastating complication of coronary artery stenting
that usually results in significant myocardial infarction or death. Approximately 40% of
reported cases have occurred in the context of non-cardiac surgery (NCS) performed in patients
with coronary artery stents, in whom dual antiplatelet therapy or clopidogrel alone has been
ceased. In patients with coronary disease, cessation of aspirin or clopidogrel is associated with
an approximate 2-3 fold increase in subsequent death or myocardial infarction. This risk is
further elevated in patients with intracoronary stent and is of added concern because the
dramatic consequences of stent occlusion. There is uncertainty regarding the risks of stent
thrombosis in individual patients, and in particular how to balance this risk against that of
surgical complications if antiplatelet therapy is continued throughout the perioperative period.
The following are current recommendations regarding antiplatelet agents and non-cardiac
surgery: Elective non-cardiac surgery should be deferred for at least 6 weeks and ideally 3
months following PCI with bare metal stenting. Elective surgery should be deferred for 12
months following drug eluting stents because of a likely increased risk of death/myocardial
infarction/stent thrombosis. Despite the observation that dual antiplatelet therapy increases the
likelihood of bleeding for most surgical procedures, the consequences of bleeding are less
significant than those of stent thrombosis. The risk benefit ratio would favor continuation of
aspirin in most patients and dual antiplatelet therapy in many patients with prior coronary
artery stenting who are undergoing non-cardiac surgery. Exceptions to this include patients
undergoing spinal, intracranial, extraocular TURP or major plastic reconstructive procedures.
For these operations, patients at low risk of stent thrombosis should have their antiplatelet
therapy routinely ceased perioperatively. In patients with coronary artery disease in whom NO
stent has been placed, antiplatelets can be stopped 7 days before the surgery. In the event of
emergency, surgery should be proceeded to without cessation of antiplatelet therapy. platelet
transfusion might be considered in selected patients in anticipation or occurrence of major
bleeding. In other situations than bare metal stents and drug eluting stents placed within the
past 6 weeks and 12 months respectively, antiplatelet medications should be stopped 1 week (7
days) before the procedure.

148. A 72-year-old woman has been on routine dialysis sessions for the past 6 months due
to end-stage renal disease. At the beginning of each session, she is found to have a high
blood pressure (BP). During the session the BP normalizes, but goes up again after the
session. Which one of the following would be the most appropriate management option
for this patient?
A Addition of hypertensive medications.
B Addition of furosemide, daily.
C Increasing the dialysis time.
D Sedation before dialysis.
E Decreasing the dialysis time.
Option C is correct Although hypotension during hemodialysis is a frequent complication,
some patients (5-15%) develop paradoxical hypertension in the later stages of dialysis or when
the patients comes off the dialysis machine, a time at which most of the excess fluid has
already been removed. The pathogenesis is unclear; however, the following mechanisms have
been hypothesized as the cause: 1. Renin-angiotensin system activation because of
ultrafiltration (UF) induced hypovolemia 2. Sympathetic overactivity 3. Intradialytic Ca /k
variations 4. Blood viscosity/hemoconcentration-induced vasoconstriction caused by
erythropoietin treatment 5. Fluid overload 6. Increased cardiac output 7. Endothelin-driven
vasoconstriction 8. Antihypertensive drug removal by dialysis treatment The optimal therapy
for this problem is not known. While antihypertensive medications such as angiotensin
converting enzyme inhibitors (ACE inhibitors) and alpha-blockers have been used before (or
during) dialysis, they have not been predictably effective. Carvedilol, which blocks endothelin-
1 release, appears to be effective. Although there are no validated universal guidelines
regarding management of such patients, fluid removal has been accepted as the first-line
treatment for intradialytic hypertension (IDH). Theoretically, increasing the time of the dialysis
session and ultrafiltration (UF) rate would be efficient; however, this decision faces many
difficulties such as patient refusal or the unit limitations. This treatment should be done with
caution to avoid hazardous blood pressure drop that may occur in the elderly or patients with
severe comorbidity. The dry weight of patients should be gradually reduced by increasing the
dialysis time and the UF rate. In addition, patients should be advised to decrease their daily salt
and water intake in between their dialysis sessions. (Options A and B) Addition of anti-
hypertensive or other medications ACE inhibitors or angiotensin receptor blockers (ARBs),
beta blockers, endothelin-1 receptor blockers, furosemide, etc has been associated with
conflicting results. In some patients, hypertensive crises may occur. These hypertensive crises
are not persistent and usually the blood pressure level quickly decreases spontaneously;
however, addition of antihypertensive medications might be indicated. There is no comment
regarding a hypertensive crisis in this patient to necessitate addition of antihypertensives.
Furosemide is not the first-line option for management of hypertension crisis if it occurs.
(Option D) Sedation before dialysis has no role in management of IDH or post- dialytic
hypertension. (Option E) Decreasing the dialysis time results in insufficient excess fluid
removal and hypertension due to volume overload.

149. A 65-year-old man presents to your practice with complaint of blood in the urine.
These episodes of hematuria have all been painless. Which one of the following is the least
likely cause of painless hematuria?
A Cancer within the kidney.
B Anticoagulantion therapy.
C Glomerulonephritis.
D Benign prostatic hyperplasia (BPH).
E Use of cyclophosphamide.
Option E is correct All the given options can present with painless hematuria except
cyclophosphamide. Cyclophosphamide use can result in hemorrhagic cystitis. With cystitis, the
hematuria is more likely to be painful rather than painless. The following can result in
hematuria (painless or painful): Urothelial cancers (kidney cancers, bladder cancers, etc)
Hydronephrosis/distention Renal vein thrombus / renal artery embolism Arteriorvenous
malformation Papillary necrosis (sickle cell disease) Hypertension Glomerulonephritis
Structural abnormalities (polycystic kidney disease, medullary sponge kidney, etc)
Nephrolithiasis Urinary tract infections (pyelonephritis, cystitis, parasitic infections, etc)
Ureteral strictures BPH Prostate cancer Prostatic procedures Trauma (including traumatic
catheterization) Exercise-induced hematuria Bleeding diathesis / anticoagulation Urethritis
Urethral diverticulum Hypercalciuria/hyperuricosuria Urinary tract fistulas Mimics of
hematuria: Menstruation Drugs (phenazopyridine, pyriduim, rifampin, nitrofurantoin, etc)
Pigmenturia Beeturia

150.A 70-year-old man from a low-level-of-care nursing home is brought to the hospital
after he had a fall 3 hours ago. He is on warfarin for atrial fibrillation (AF). Laboratory
studies show that he has an INR of 4.9. A CT scan of the head is ordered which is normal.
Other investigations are unremarkable. Which one of the following is the next best step in
management?
A Stop warfarin.
B Skip one dose of warfarin.
C Give fresh frozen plasma (FFP).
D Give vitamin K.
E Repeat CT scan of the head in 2 days.
Option B is correct The therapeutic range of warfarin for most patients varies between 2 to 3.5
(2-3, or occasionally 2.5-3.5). Increased INR beyond therapeutic levels are associated with
higher risk of bleeding. For patients with an INR above the therapeutic range but less than 5,
who are not bleeding, skipping the next dose of warfarin and resuming lower doses once the
INR is within the therapeutic range is the recommended management. Ceasing warfarin, FFP,
vitamin K (intravenously) and Prothrombinex are used in situations where there is active
bleeding or the risk of bleeding is high.

151. A 30-year-old woman is concerned about breast cancer because her mother was
diagnosed with breast cancer at the age of 48 years. Clinical examination of the breast is
normal. Which one of the following would be the most appropriate advice for her?
A Mammography every year from the age of 40.
B Mammography every 2 years from 50-69 years of age.
C Refer to a surgeon.
D Refer for genetic studies.
E Reassure her that she is not at high risk of breast cancer.
Option A is correct With a first-degree relative with breast cancer diagnosed before 50 years,
this woman is considered as having moderate risk for breast cancer. For such patients 1-yearly
screening mammography starting from the age of 40 is the best advice to give. (Option B)
Mammography every 2 years is the most appropriate recommendation for general population
and those with just slightly increased risk above average for breast cancer. (Option C) Referral
to a surgeon would have been indicated if a diagnosis of breast cancer was established. (Option
D) Referral for genetic testing is an acceptable step for those women who are likely to have a
predisposition for familial forms of breast/ovarian cancer. Referral for genetic testing should be
considered once screening tools raise suspicion against strong risks of such cancers. (Option E)
Although this woman is not in high-risk category and is categorized as having moderate risk,
reassuring will not be an appropriate action because she needs a more extensive surveillance
program for breast cancer than general population.

152. A 42-year-old man presents to your clinic with history of a slowly-enlarging painless
scrotal mass over the past months. He has previous history of unilateral cryptorchidism
which required orchiopexy. On examination, a solid scrotal lump is palpated. Which one
of the following would be the next best step in management?
A Measuring serum alpha fetoprotein.
B Measuring serum human beta chorionic gonadotropin (βHCG).
C CT scan of the abdomen and pelvis.
D Ultrasonography of the scrotum.
E Scrotal needle biopsy.
Option D is correct All solid scrotal lumps should be considered malignant until proven
otherwise. In approach to scrotal masses the most appropriate next step is to request an
ultrasound to further evaluate the scrotal mass. Ultrasound of the testes can reliably diagnose
the testicular tumor with considerable precision and can also detect any invasion of the tunica
albuginea. This man also has undergone orchipexy that is a risk factor for development of
testicular cancer. Risk factors of development of testicular cancer include personal or family
history of any of the following: Cryptorchidism (undescended testes) Orchiopexy Testicular
atrophy Previous testicular cancer (Options A and B) Tumor markers (alpha fetoprotein and
beta HCG) are not requested unless ultrasound suggests cancer. (Option C) CT scan of the
abdomen, pelvis and chest is done for staging purposes and only considered after confirmed
diagnosis of testicular malignancy. (Option E) Needle scrotal biopsy should be avoided
because of potential risk of tumor implanting malignant cells in the scrotal wall.

153. A 20-year-old man patient presents to your clinic with painless scrotal swelling.
Ultrasound shows a solid mass in left testis. Which one of the following is the next best
step in management?
A Fine needle aspiration cytology (FANC).
B Surgical exploration.
C CT scan of the chest.
D Serum lactate dehydrogenase (LDH).
E Serum beta-hCG and alpha fetoprotein (AFP).
Option E is correct All scrotal lumps in a young male is malignancy until proven otherwise,
especially if painless. Ultrasound is the initial investigation of choice for assessment of a
scrotal lump. If a lesion within the testis is found on ultrasound, malignancy should come at the
top of the differentials list, and blood sample be obtained for serum tumor markers. Tumor
markrs used for testicular cancer are alpha feto-protein (AFP) , beta hCG and lactate
dehydrogenase (LDH). LDH is usually raised in metastatiic tumors. In seminoma, AFP is not
elevated beta-hCG is elevated in less than 20% of patients. In nonseminoma testicular cancer,
however, AFP and beta hCG are elevated in 85% of patients. These serum markers, especially
AFP and beta hCG can provide evidence for the initial diagnosis of a testicular cancer and are
useful for assessment of the prognosis and risk stratification. (Option A) Fine needle aspiration
is not recommended for testicular cancers due to potential risk of tumor implantation in the
scrotal wall. (Option B) Surgical exploration is contraindicated due to the risk of tumor spread.
Any scrotal incision should be avoided. Once the diagnosis of testicular cancer is established,
orchiectomy through inguinal approach is the best initial treatment. (Option C) CT scan of the
chest is helpful for tumor staging once the diagnosis is established. (Option D) LDH is tumor
marker for testicular cancers used to see if the tumor has metastasized. LDH alone is not often
diagnostic..

154. A 17-year-old boy presents to the emergency department with a 2-hour history of a
painful and tender right testis. On examination, the scrotum is tender and swollen.
Spermatic cord is thickened and can be moved above the lump. Which one of the
following is the next best step in management?
A Surgery.
B Ultrasound.
C Urine microscopy and culture.
D Full blood count.
E Technetium scan.
Option A is correct A painful scrotum should be taken seriously because the outcome of an
untreated testicular torsion can be catastrophic. The testis can be infarcted in 4 to 6 hours. With
physical findings of a swollen, tender and firm hemiscrotum with a short and thickened
spermatic cord the patient should immediately be taken to the operating room. No male
younger than 18 years should be considered to have epididymo-orchitis until the testis has been
exposed at operating room and torsion is excluded. (Option B) The role of ultrasound in
distinguishing between testicular torsion and epididymoorchitis is controversial, as it cannot
reliably detect changes that are diagnostic for an early torsion. (Options C and D) Once
torsion of testis has been ruled out, full blood count, urine microscopy and Chlamydia testing
are performed to clinch other possible diagnoses. (Option E) A technetium-99 m scan can
differentiate between the two conditions. In testicular torsion the testis is avascular, while it is
hyperemic in epididymo-orchitis. However, with acute scrotal pain in a young male, the
diagnosis is testicular torsion until proven otherwise by surgival exploration. No time should be
wasted on diagnostic procedures.

155. A 47-year-old man presents to your practice with a scrotal swelling. The swelling is
not painful and has been there for the past 4 months. He does not feel bothered with the
swelling and has just come to see you because her wife has forced him to. On examination,
there is a non-tender swelling in the left testis. Ultrasonography shows the mass to be cyst
within the right left testis. Which one of the following would be the next best step in
management?
A Surgery.
B Biopsy through inguinal route.
C Biopsy through scrotum.
D Measuring serum tumour makers (alpha-phetoprotein and beta-hCG).
E Reassurance.
Option D is correct Testicular cystic lesions are relatively common findings on testicular
ultrasound. The underlying pathology ranges from benign to malignant and may include:
Simple testicular cyst Tunica albuginea cyst Cystic transformation of rete testis (intratesticular
tubular ectasia) Intratesticular spermatocele Intratesticular varicocele Testicular epidermoid
cyst Cystadenoma of the rete testis Testicular abscess Testicular tumors Over 23% of testicular
cysts are malignant. For this reason, malignancy should be excluded by serum tumor markers
as the most appropriate next step in management once a testicular cyst is found on ultrasound.
Three serum tumour markers have established role in testicular cancer: alpha fetoprotein (AFP),
beta subunit of human chorionic gonadotropin (beta-hCG), and lactate dehydrogenase (LDH).
In seminomas, AFP is never elevated and beta-hCG is elevated in fewer than 20% of patients.
In non-seminomas, AFP and beta-hCG are elevated in 85% of patients. (Option A) Surgery for
orchiectomy is considered after investigations establish the diagnosis of testicualr cancer.
Without a definite diagnosis, surgery is not indicated for now. (Option B) Biopsy through
inguinal approach is inappropriate at this stage and before tumor marker assay. (Option C)
Testicular tumors are never resected through scrotal excisions. (Option E) This patient
cannot be reassured unless full assessment rules out the possibility of malignancy.

156. A 25-year-old patient is being staged for further management of a seminoma which
presented as a lump in the left testis. Involvement of which one of the following lymph
node groups by lymphatic spread would indicate the worst prognosis?
A Cervical.
B Mediastinal.
C Retroperitoneal.
D Inguinal. E Femoral canal.
Option A is correct Testicular neoplasms spread via draining lymphatics along the gonadal
veins in the spermatic cord, and along the inguinal canal to retroperitoneal nodes and from
retroperitoneal and paraaortic nodes, progressive spread can occur via the cistern chyli and
thoracic duct to Mediastinal and left-sided cervical nodes at the base of the neck. The latter
group drain into the systemic venous circulation; therefore, involvement of cervical nodes is
associated with the worst prognosis. Inguinal and femoral vessels and nodes (Cloquet) drain the
leg and the skin of lower abdomen, back, buttock, penis, scrotum and anal canal. They do not
drain testes.

157. A 25-year-old man presents with a painless right scrotal lump which he noticedwhile
bathing 3 days ago. There is no tenderness or other abnormal findings expect
asymptomatic enlargement of the right testis. The epididymis and spermatic cord can be
felt in their normal positions. An ultrasound scan is performed which is shown in the
accompanying photograph. Which one of the following is the most appropriate next step
in management?
A Reassurance.
B Measurement of serum alpha phetoprotein (AFP).
C Measurement of serum carcinoembryonic antigen (CEA).
D Trans-inguinal excision of the right testis.
E Trans-scrotal excision of the right testis.
Option B is correct The photograph pictures the ultrasonographic view of the right testis, in
which a lesion is seen. The lesion is combination of both cystic and solid structures. A
testicular lump in a young man is testicular cancer until proven otherwise. When testicular
cancer is suspected on ultrasound, the next best step is always obtaining blood sample for
serum tumor markers. Three serum tumor markers have established roles in patients with
suspected testicular cancer: alpha fetoprotein (AFP), beta subunit of human chorionic
gonadotropin (beta-hCG), and lactate dehydrogenase (LDH). Tumor markers differ by
histological subtype of testicular malignancies. In pure seminoma, AFP is never elevated and
beta-hCG is elevated in fewer than 20% of patients.
In other histological subtypes, AFP and beta-hCG are elevated in 85% of patients. Of the
options only measurement of serum AFP can be correct as the next best step in management.
Other measures to conisder prior to surgery are: High resolution CT scan of the abdomen,
pelvis and chest Chest X-ray Chemistry profile and full blood count (FBC) (Option A) No
man with a testicular lump can be reassured unless thorough investigations exclude testicular
malignancies. (Option C) CEA is not a tumor marker for testicular cancer. (Options D and E)
Tumor marker studies and imaging studies should be performed prior to surgical management.
If surgery is planned a trans-inguinal approach should be considered, as there is a risk of tumor
spread and scrotal seeding when the testis is reached through scrotum.

158. A 62-year-old woman presents to your clinic for discussing treatment options after
she sustained a wrist fracture while playing tennis a few weeks ago. On DEXA scan she is
found to have a T-score of -1.7. Which one of the following would be the most appropriate
management?
A Zoledronic acid.
B Vitamin D supplements.
C Advise that she should have more exercise.
D Reassure that she does not need treatment at this stage.
E Raloxifene.
Option A is correct Different guidelines and authors suggest various indications for
commencement of medical therapy for osteoporosis. This sometimes leads to confusion as to
whether which patients are candidates for medical therapy. Literally osteoporosis is defined as
a T score equal to or less than -2.5 on BDM. Established osteoporosis is the preferred term for
those with osteoporosis (a T-score less than -2.5 at the hip or spine), and one or more fragility
fractures. Recognized fractures for this definition include Colles’, hip, vertebral and low-
trauma fractures at the ankle. (While various other terms, such as ‘severe’, ‘advanced’ or ‘frank’
osteoporosis appears in reports, they have no specific meaning and should not be used). A first
fragility fracture doubles the risk of another fracture. With two previous fractures, there is 12-
fold increase in the risk of consequent fractures. Because of such risk, current guidelines
recommend treatment of those who have had a fracture and a T-score is below -1.5. In other
words, the diagnosis of osteoporosis is not required to treat these individuals (a T-score≤-2.5 is
not needed to start treatment) All the following patients need medical treatment: Those with a
minimal trauma fracture (even if the T-score is not <-2.5 ; a T-score of <-1.5 and a fracture
necessitate medical treatment) Those 70 years old or older Those currently on prolonged (at
least 3 months) high dose corticosteroid treatment (at least 7.5 mg/day prednisolone or
equivalent) and with a T-score of –1.5 or lower. For others, ensuring adequate dietary calciu
and vitamin D intake (or supplementation if there is adequate dietary intake) and lifestyle
modification such as exercise, reducing alcohol consumption and stopping smoking. All other
patients need to be treated medically. Unless contraindicated, the first-line option for treatment
of osteoprorosis is always bisphosphonates. Although alendronate was considered the drug of
choice, recent recommendation favor the use of intravenous zoledronic acid due to more
compliance. Calcium and vitamin D should be given to all patients undergoing medical
treatment regardless of the specific agent. If the patient is vitamin D deficient, correction of
vitamin D level (25-hudroxy vitamin D>50 nmol/L) is the most important step in management
prior to commencement of treatment with bisphosphonates. The patient should receive vitamin
D first if: There is documented vitamin D deficiency based on serum 25-hydroxy vitamin levels
Housebound women Nursing home residents Women who are shrouded due to cultural reasons

159. A 69-year-old woman has been in the Intensive Care Unit (ICU) for the past 7 days
following a complicated hip replacement surgery. She is currently on lowmolecular
weight heparin (LMWH) as well as intermittent pneumatic compression device. Today,
she has developed new-onset right calf pain. On examination, the leg is edematous and
tender to touch. A positive Homan’s sign is elicited. A Doppler ultrasound confirms the
presence of a clot in the right lower leg and the diagnosis of deep vein thrombosis. Blood
tests show a platelet count of 78,000 /m m, but there is no sign of hemorrhage. Which one
of the following is the next best step in management?
A Discontinue application of pneumatic compression device
B Start her on warfarin.
C Discontinue heparin.
D Perform venography.
E Transfuse platelets.
Option C is correct Development of DVT in the presence of thrombocytopenia is suggestive
of heparin-induced thrombocytopenia (HIT) as the most likely diagnosis. HIT is a complication
of heparin therapy. There are two types of HIT: Type 1 HIT - presents within the first 2 days
after exposure to heparin, and the platelet count normalizes with continued heparin therapy.
Type 1 HIT has a non-immune pathophysiology. Type 2 HIT - an immune-mediated disorder
that typically occurs 4-10 days after exposure to heparin and has life- and limb-threatening
thrombotic complications. In general medical practice, the term HIT refers to type 2 HIT. HIT
is caused by antibodies that bind to complexes of heparin and platelet factor 4 (PF4), activation
of the platelets, and promoting a prothrombotic state. HIT is more frequently encountered with
unfractionated heparin (UFH) than with low molecular weight heparin (LMWH). The risk of
HIT is highest with prolonged use of heparin for postoperative thrombophylaxis. However,
case studies have also shown that HIT can occur even with minimal heparin exposure via
intravascular flushes to maintain an arterial or venous catheter open. HIT must be suspected
when a patient, who is receiving heparin, has a decrease in the platelet count, particularly if the
fall is over 50% of the baseline count, even if the platelet count remains above 150,000/mm .
Clinically, HIT may manifest as skin lesions at heparin injection sites or by acute systemic
reactions (e.g. chills, fever, dyspnea, chest pain) after administration of an intravenous bolus of
heparin. Unlike other forms of thrombocytopenia, HIT is associated with thromboembolism
(e.g. DVT, PE) as the most common complication, not with bleeding. Sometimes, arterial
thrombosis (e.g. myocardial infarction) may occur. For this reason, the disorder is sometimes
termed heparininduced thrombocytopenia and thrombosis (HITT). Factors that increase the risk
for heparin-induced thrombocytopenia (HIT) are as follows: Use of unfractionated heparin
(UFH) rather than low molecular weight heparin (LMWH) Intravenous rather than
subcutaneous administration of heparin Longer duration of heparin use Surgical (especially
cardiac or orthopedic surgery) rather than medical patient Female sex HIT usually occurs 4-10
days after the start of heparin therapy. However, in patients with recent prior heparin exposure
(especially within the previous month, but even within the past 100 days), persistence of
circulating HIT antibodies may result in rapid-onset HIT, in which the platelet count falls
within 24 hours of starting heparin. Onset of HIT may also occur after heparin cessation.
Delayed-onset HIT should be considered when a patient presents with thrombosis and
unexplained thrombocytopenia up to 3 weeks after heparin therapy has ended. Patients with
heparin-induced thrombocytopenia (HIT) may present with unusual characteristic sequelae,
including the following: Venous limb gangrene Bilateral adrenal hemorrhagic infarction Skin
lesions at injection sites Acute systemic reactions following an intravenous heparin bolus If
heparin-induced HIT is suspected, the first step is to discontinue and avoid all heparin products
immediately. Even small doses for catheter flushing should be avoided. (Option A) HIT is not
caused by pneumatic compression; therefore, its discontinuation has no benefit in management
of this patient. (Option B) Warfarin should be avoided in such patients because it can lead to
microthrombosis. Interestingly, patients with HIT have an INR of above 4. Warfarin can be
started once the platelet count is back to normal. If warfarin has already been started, vitamin K
should be given. (Option D) Once a doppler ultrasound has confirms the diagnosis of DVT,
venography is not required as an extra measure unless some surgical intervention is planned
and an accurate venous map is required. (Option E) Platelet transfusion should be avoided in
HIT, as it may increase the thrombogenic effect. Guidelines from the American College of
Chest Physicians suggest limiting platelet transfusions to patients with severe
thrombocytopenia, who are experiencing bleeding or undergoing an invasive procedure with a
high risk of bleeding. Once HIT occurs, alternatives to heparin are direct thrombin inhibitors
(DTIs) such as argatroban and the indirect factor Xa inhibitor fondaparinux (Arixtra®).

160. A 62- year-old woman presents with bloody discharge from her left nipple. The
family history is significant for a cousin recently diagnosed with breast cancer. On
examination, no mass is palpated. Which one of the following is the most likely diagnosis?
A Breast duct papilloma.
B Breast duct ectasia.
C Paget disease of the breast.
D Mastitis.
E Breast duct carcinoma.
Option A is correct Nipple discharge, as a sign, may have a variety of underlying causes.
Some of these causes are as follows: Bloody nipple discharge – bloody discharge from nipple is
usually caused by benign duct papilloma, but breast neoplasm including breast duct carcinoma
and Paget disease of the breast should always be excluded. Serous, green, yellow-brown
discharge – it is usually due to benign fibrocystic changes Toothpaste (worm)-like discharge –
it is characteristic of mammary duct ectasia. The bloody nipple discharge of this woman is
most likely to have been caused by benign ductal papilloma; however, breast duct carcinoma
(option E) and Paget disease of the breast (option C), as less common possibilities, should be
considered as well. Breast duct ectasia (option B) presents with worm-like or toothpaste-like
but not bloody nipple discharge. Mastitis (option D) presents with local tenderness and redness
of the breast. The clinical picture is completely different from this case scenario. Moreover, it
is a very uncommon condition in a woman of this age.

161.During a routine health assessment, a 43-year-old man is found to have established


asymptomatic hematuria. Which one of the following is the most appropriate next step in
management?
A Ultrasonography of the renal system.
B CT scan.
C Cystoscopy.
D Urine culture.
E Intravenous pyelogram (IVP).
Option D is correct Hematuria may be seen in a variety of situations including, but not limited
to: Infections Renal stones Glomerulonephritis Urinary tract malignancies Trauma Infections
are the most common cause of hematuria in both symptomatic and asymptomatic patients. For
this reason, the next best step in management of every patient with hematuria would be a urine
analysis and culture. Urologic malignancies (mostly cancer of the bladder) should be
considered and ruled out once urine culture is negative and renal stones are excluded using
ultrasound (the best initial test) or spiral CT (the most accurate test).

162. A 72-year-old man presents with complaint of one episode or blood in the urine. Five
years ago he underwent colectomy after he was diagnosed with colon cancer. A while
back, he developed back pain for which he was assessed and diagnosed with metastatic
bone disease. Three months ago he was started on tramadol for management of the back
pain. He has no urinary symptoms. Which one of the following is the most appropriate
next best step in management?
A Perform a pelvic and abdominal CT scan.
B Perform a renal ultrasound.
C Urine culture.
D Stop tramadol.
E Perform an intravenous pyelogram (IVP).
Option C is correct Hematuria should be always considered as a sign; therefore, for
established cases of hematuria an underlying cause should be investigated through history,
physical examinations and laboratory/imaging studies. For established hamaturia, the most
common causes such as vigorous exercise, menstruation, trauma, viral illnesses, and infections
should be excluded first. One of the most common causes of hematuria even in the absence of
symptoms is urinary tract infection (UTI), for which a urine analysis and culture should be
performed. (Option A) Genitourinary malignancies (either primary or metastatic) can cause
hematuria. Malignancies should always be excluded and CT scan is one of the mostl accurate
and commonly used means. However, other common causes of hematuria should be excluded
first. (Option B) Renal ultrasound may be later needed as further work-up but not as the first
priority because the hematuria has been painless making renal stones less likely. Small stones
may cause painless hematuria, but since they almost always pass spontaneously, their detection
on sonography does not change the management plan. (Option D) Although hematuria has
been reported as a rather rare adverse effect of tramadol, initial assessment should be focused
on more common possibilities such as UTI. Even so, cessation of tramadol in an almost end-
stage patient would not be recommended. (Option E) If imaging studies are required during the
evaluation process, CT scan is the preferred over IVP. IVP is rarely done owing to the fact that
more modern and convenient imaging modalities are available.

163.A 37-year-old woman, who underwent thyroidectomy 4 days ago, presents with
complaints of tingling of her hands and feet and around her mouth. Her total calcium
level was 2.02 (normal 2.1- 2.6 mmol/L) on discharge. Which one of the following is the
best immediate management of this patient?
A Intravenous calcium.
B Calcium gluconate, orally.
C Calcium gluconate and vitamin D, orally.
D Diazepam.
E Calcitriol.
Option A is correct Every patient with symptomatic hypocalcemia (paresthesia of hands, feet
and perioral area are sensory equivalents of tetany) with any calcium concentration below the
reference and those with asymptomatic hypocalcemia, whose total calcium levels have acutely
fallen below 1.9 mmol/L are recommended to urgently have calcium replaced intravenously.
Ten to 20ml of 10% calcium gluconate in 50-100 ml of Dextrose 5% should be given
intravenously over 10-20 minutes with ECG monitoring. This can be repeated until the patient
is asymptomatic. It should be followed with a calcium gluconate infusion. Calcium chloride
can be used as an alternative to calcium gluconate, but it is more irritating to veins and should
only be given via a central line. If the case was asymptomatic hypocalcemia (with calcium
above 1.9 mmol/L) or latent tetany (represented by positive Trousseau and/or Chvostek sign)
oral calcium with or without vitamin D supplementation was the choice.

164. A 25-year-old man presents to the emergency department of a tertiary hospital with
complaint of back pain. The pain is felt over the L4/L5 region. He denies any history of
preceding back trauma or sprain, but admits to injecting drug use previously. He also
mentions positivity for hepatitis C virus antibody. Remarkable finding on physical
examination is erythema and tenderness over the L4/L5 area. Which one of the following
would be the most appropriate diagnostic approach to reach a diagnosis in this patient?
A CT scan of the lumbar area.
B MRI of the lumbar area.
C HIV serology.
D Bone scan.
E X-ray of the lumbar area.
Option B is correct Local tenderness and erythema over a bony area in a patient with history
of injection drug abuse should raise the suspicion of osteomyelitis as a very likely diagnosis.
This possibility is increased given the positive hepatitis C antibody titer that indicates hepatitis
C infection and the likelihood of other infections associate with shared needle use. Needle
sharing is associated with a significant risk of hematogenous spread of infections to bones,
joints and soft tissue. NOTE - Osteomyelitis can occur as a result of contiguous spread of
infection from adjacent soft tissue and joints, hematogenous seeding, or direct inoculation into
the bone as a result of trauma or surgery. Hematogenous osteomyelitis is more common in
children than in adults. In children, long bones are most often affected, while the vertebral
column is the most common site of involvement with hematogenous spread. Typically, acute
osteomyelitis presents with gradual onset of symptoms over several days. There is often a dull
pain in the affected area that can be related or unrelated to movement. There might be local
findings such as erythema, swelling, tenderness or warmth, or systemic manifestations such as
fever and rigors. However, patients with osteomyelitis of the hip, vertebrae or pelvis may have
few signs and symptoms other than pain (such as in this patient). MRI is the investigation of
choice in diabetic patients with suspected osteomyelitis of the foot and all patients with
suspected osteomyelitis of the vertebral column. (Option A) CT scan is use for evaluation if
MRI is contraindicated or not available. (Option D) A bone scan is considered for patients with
suspected osteomyelitis and contraindications to MRI and CT scan (e.g. metal hardware).
(Option C) While HIV and decreased immunity is a risk factor for osteomyelitis as well as
other infections, HIV positivity is not diagnostic for osteomyelitis. It is necessary though to
check, as well as hepatitis B status, in a patient with history of intravenous drug use. (Option
E) ESR and full blood exam are tests routinely consider but they are not specific. Findings
associated with osteomyelitis are leukocytosis and elevated ESR (and often CRP) which are
common findings in many other conditions than osteomyelitis. NOTE - An X-ray is the initial
diagnostic modality to consider in patients presenting with signs and symptoms of
osteomyelitis. X-rays however are usually negative and not sensitive in the first 2-3 weeks of
infection. X-ray becomes positive when at least 50% of the mineral bone is lost due to infection.

165. A 56-year-old Victorian farmer presents to your GP clinic with chest tightness and
coughing for the past 2 months. He also mentions vague right upper quadrant abdominal
pain. On examination, no remarkable findings are noted except for mildly jaundiced
sclera. Which one of the following could be the most likely diagnosis?
A Hydatid cyst.
B Q fever.
C Ross River fever.
D Brucellosis.
E Dengue fever.
Option A is correct The clinical picture, suggestive of lung and hepatic involvement, and the
occupation of the patient make echinococcus infection (hydatid cyst) the most likely diagnosis.
In Australia, most hydatid infections are passed between sheep ad dogs, although other animals
including horses, kangaroos, dingoes and foxes may be involved. The hydatid parasite is
carried by dogs in their bowels, without any symptoms of infection. Sheep become infected
while grazing in areas contaminated with dog feces. Dogs become infected by eating the
uncooked organs of the infected sheep. People become infected by ingesting eggs of the
parasite, usually while there is a hand-tomouth transfer of eggs in dog feces. This can occur
when handling dogs or objects (including food and water) soiled with dog feces. Person-to-
person or sheep-to-person transmission does not occur. The pressure effect of the cyst on the
liver can produce symptoms of obstructive jaundice and abdominal pain. With biliary rupture,
the classic triad of biliary colic, jaundice and urticaria is observed. Passage of hydatid
membranes in vomit (hydatid emesia) and passage of membrane in the stool (hydatid enterica)
may rarely occur. Involvement of the lungs produces chronic cough, dyspnea, pleuretic chest
pain and hemoptysis. Expectoration of cyst membrane and fluid is observed with intrabronchial
rupture.

166. A 20-year-old man presents to the emergency department with acutely painful
scrotal swelling for the past 12 hours. Evaluation establishes the diagnosis of epidiymo
orchitis. Which one of the following is the most likely causative organism? A E.cloi.
B Staphylococcus aureus.
C Chlamydia.
D Neisseria gonorrhoea.
E Treponema pallidum.
Option C is correct Causative organism in epididymo-orchitis varies based on the age of the
patient: Children and men older than 35 years: Urinary coliforms (e.g. E coli, Pseudomonas
species, Proteus species, Klebsiella species) are the most common cause. Less frequent
pathogens are Ureaplasma urealyticum, Corynebacterium species, Mycoplasma species, and
Mima polymorpha have also been isolated. Systemic Hemophilus influenzae and Neisseria
meningitides infections are rare yet possible cause. Sexually active men younger than 35 years
Chlamydia is the most common cause in sexually active men younger than 35 years
(accounting for up to 50% of cases, although laboratory evidence of Chlamydia may be absent
in up to 90% of cases). Infections with Neisseria gonorrhea, Treponema pallidum, Trichomonas
species, and Gardnerella vaginalis also occur in this population. In this sexually active man,
who is younger than 35 years, Chlamydia would be the most likely causative organism.

167. A 67-year-old man presents to your office complaining of a painful swollen right
thigh. He has diabetes well controlled on metformin 500mg 12-hourly and hypertension
for which he is taking losartan 25mg 12-hourly. He is also on simvastatin 20 mg/daily
hypercholesterolemia. His recent medical history is significant for atrial fibrillation (AF)
under treatment with warfarin. One week ago, he was started on amiodarone after he was
diagnosed with ventricular tachycardia (VT). On physical examination, he has a blood
pressure of 140/95mmHg, pulse rate of 98bpm and a temperature of 37.3°C. The right
thigh is painful, swollen and slightly tender but not warm or red, and has a circumference
4 cm greater than that of the left thigh. Which one of the following could be the most
likely cause of this presentation?
A Deep vein thrombosis (DVT).
B Drug reaction.
C Cellulitis.
D A thigh hematoma.
E Rhabdomyolysis.
Option D is correct A painful swollen thigh can be caused by a number of different conditions
such as DVT, cellulitis and hematoma. This patient is on warfarin for treatment of AF.
Although not impossible, it is less likely that he develops DVT (which is treated with warfarin
as well). This makes DVT (option A) a less likely yet possible diagnosis. Interaction between
warfarin and amiodarone is well-known. Amiodarone results in decreased metabolization of
warfarin through hepatic pathways and leads to increased bleeding tendency. Considering that
the patient has been recently started on amiodarone while on warfarin, a hematoma can also be
possibility and in fact the most likely one. Hematomas are a frequent result of increased
bleeding tendency occurring in over-anticoagulated patients. Drug reaction (option B) is
different from drug interaction. Drug reaction means an adverse reaction caused by a single
drug while in drug interaction the effect of one drug leads to unwanted or exaggerated response
to another. While this scenario is very likely to have been caused by the interaction between
amiodarone and warfarin, resulting in increased untoward effects of warfarin, none of the drugs
in the medications he is taking are not likely to result in such presentation. Cellulitis (option C)
presents with a warm and red areas of swollen skin. Fever is often present. This patient has no
fever and the thigh swelling is not warm and red. This makes cellulitis a less likely diagnosis
compared to hematoma. Rhabdomyolysis (option E) is a serious condition caused by muscle
fibers breakdown and release of muscle cell contents such as myoglobin and potassium.
Myoglobinuria results in deposition of myoglobin in kidneys and renal failure. Release of
excess potassium from damaged muscle cells leads to hyperkalemia and serious complications
such as cardiac arrhythmias. Statins such as atorvastatin and simvastatin in particular are
metabolized by cytochrome P450-3A4 (CYP3A4), and amiodarone is a potent inhibitor of this
cytochrome. Concomitant use of amiodarone and statins can result in increased activity of
statins and rises the likelihood of statin-related adverse effects such as muscular pain,
myopathy and rarely rhabdomyolysis. However, in the event of statin-induced rhabdomyolysis,
a systemic presentation is expected. Rhabdomyolysis never causes focal signs mentioned in the
scenario.
168. A nurse from the surgery ward calls you to visit a 65-year-old inward patient for
agitation. When you arrive at the ward you are informed that she underwent
cholecystectomy 48 hours ago. Her file shows that she received prophylactic amoxicillin
prior to the surgery. On examination, she is confused and delirious, has a blood pressure
of 135/87mmHg, heart rate of 110 and temperature of 38.4°C. She is agitated and difficult
to deal with. Which one of the following is the most appropriate next step in management?
A Start her on ceftriaxone. 4
B Give her haloperidol.
C Intravenous fluids.
D Chest X-ray.
E ABG.
Option B is correct Delirium following a major surgery in the elderly is a common
postoperative complication. Delirium and confusional state after surgery are most commonly
caused by hypoxia. Hypoxia can be caused by the effect of hypoventilation due to anesthetics
or analgesics, or simply by shallow breathing due to pain. Fluid and electrolyte disturbances,
hypoglycemia, or infections can also cause post-operative delirium. Alcohol withdrawal or
delirium tremens are other important etiologies not to miss. Urinary retention or fecal
impaction should be thought of as well. Management of delirium is by identification of the
underlying etiology and treating it. This woman is febrile, making infection a likely cause of
her delirious state. Atthough prophylactic antibiotics decrease the risk of post-operative
infections, they do not eliminated such risk. A confused and delirious patient can be difficult to
deal with at times, and pharmacological treatment should be considered for sedation before
other diagnostic or therapeutic measures are carried out. An ABG is often the very initial step
to exclude hypoxemia as the most likely cause of delirium, but attempts to obtain an arterial
sample in an agitated delirious patient is not easy and may result in arterial injuries in a
combative patient. An agitated patient may not let an oxygen face mask to be fixed in place.
Under such conditions, medications are used to sedate and calm the patient. Haloperidol is a
convenient drug, which is most frequently used for this purpose and is the most appropriate
next step in management. Atypical antipsychotics such as risperidone, olanzapine or
ziprasidone are second-line choices that can be used as alternatives; however, in the presence of
extrapyramidal symptoms, they should be use in preference to haloperidole. Intravenous
diazepam (not an option) is the medication of choice if alcohol withdrawal or delirium tremens
are suspected from the history and clinical findings. NOTE - Pharmacological therapy should
only be considered in a delirious person with severe behavioral disturbance and/or severe
emotional disturbance if their behavior threatens their own safety or the safety of others, is
likely to interfere with essential medical or nursing care, or if the disturbance is causing
significant distress (Option A) Although the patient is febrile, antibiotics should not be
administered unless bacterial infections are suspected after initial investigations such as full
blood count (FBC), chest X-ray, etc. (Option B) This patient has a normal blood pressure and
does not seem to be in urgent need for fluid resuscitation. However, it may be considered if
further studies indicate otherwise. (Options D and E) ABG is often the first-line investigation
to exclude hypoxia as the most common cause of post-operative delirium/confusion. A chest X-
ray is indicated for evaluation and workup of conditions such as atelectasis, pneumonia or
pulmonary embolism if the patient is found to be hypoxic/hypoxemic. Again, obtaining an
arterial sample for ABG can be difficult in this patient and should be attempted once the patient
is easier to deal with.

169. An otherwise fit 57-year-old woman spikes a fever of 39°C five days after an
appendicectomy for acute appendicitis. On examination, there is a tender, erythematous
and fluctuant swelling at the medial end of the wound. Which one of the following options
would be the most appropriate next step in management?
A Administer high dose, broad spectrum antibiotics, intravenously.
B Return the patient to the OR for resuturing the wound.
C Start the patient on oral antibiotics.
D Make an incision over the swelling to allow free draining.
E Remove the underlying suture material to prevent sinus formation.
Option D is correct The tender, erythematous, fluctuant swelling at the surgical site indicates
abscess formation as a post-operative complication of the surgical wound. Once abscess is
formed, incision and drainage is the most appropriate management to considered first.
Intravenous antibiotics are indicated as an adjunctive therapy. Wound infection and abscesses
are common post-operative complications of surgical procedures. Often, wound infections are
expected on the seventh day post-op or thereabouts. Abscess formation usually takes longer;
nonetheless these timings are inaccurate, and surgical complications can potentially occur at
any time during the post-operative period. Simple wound infections present with surrounding
cellulitis (erythema and local tenderness). Abscesses, on the other hand, are more indurated and
fluctuating and characteristically contain puss; they tend to cause higher fevers. (Option A)
Intravenous antibiotics are used after incision is made to let free drainage of the abscess.
Intravenous antibiotics are necessary, particularly in this patient with systemic infection and
high fever, but are not sufficient if used alone. Wound infections with cellulitis but no
fluctuance can be treated with a course of antibiotics without drainage. However, when
clinical findings suggest an abscess, incision and drainage is the mainstay of therapy.
(Option B) Opening the wound, irrigation and resuturing is not an effective management for
abscesses. The sutures are removed, wound debridment and irrigation is performed, but the
wound is left open and packed. (Option C) Oral antibiotics are definitely insufficient for
management of this wound infection. Such infections often require intravenous antibiotics after
incision and drainage has been carried out. (Option E) Removal of the underlying suture
material to prevent sinus formation is important but does not treat the patient’s current
condition.

170. On the sixth day after an uneventful appendectomy on a 23-year-old man due to
gangrenous appendicitis , he develops a fever. On examination, he has a blood pressure of
135/90mmHg, pulse rate of 94bpm, and a temperature of 39°C. A small tender fluctuating
swelling is noted on the lateral side of the wound. Which one of the following would be the
most appropriate next step in management?
A Oral antibiotics for 14 days.
B Ultrasonography of the wound area.
C CT scan of the abdomen.
D Incision of the wound and drainage.
E Intravenous antibiotics for 3 days, followed by oral antibiotics.
Option D is correct Wound infection and abscess formation are common post-operative
complications of surgical procedures. Usually wound infections are expected on day 7 post-
operation or thereabouts. Abscess formation usually takes longer; nonetheless these timings are
inaccurate, and surgical complications can potentially occur at any time during post-operative
period. Simple wound infections present with fever, erythema and local tenderness. Abscesses,
on the other hand, are more indurated and fluctuating, and characteristically contain puss.
Abscesses tend to be associated with higher fevers. Superficial wounds infections can be
managed by suture removal and no antibiotics. Wound infections with cellulitis alone but no
fluctuance to indicate underlying abscess can be treated with a course of antibiotics without
drainage. However, when the clinical findings suggest an abscess, incision and drainage is the
mainstay of therapy. The tender fluctuating swelling in this patient is very likely to be an
abscess. Moreover, he has systemic symptoms of infection. In such circumstance, the most
appropriate management is incision of the abscess and drainage. He also needs intravenous
antibiotics. The wound should be left open and daily sterile packing performed. (Option A)
Oral antibiotics are definitely insufficient for management of this wound infection. Such
infections often require intravenous antibiotics after incision and drainage has been carried out.
(Options B and C) Wound infection can often be differentiated from abscesses on clinical
basis. If they cannot be differentiated, ultrasonography is the investigation of choice to
distinguish them. CT scan is not the favored diagnostic tool for this purpose. Here, the clinical
findings are sufficiently diagnostic for an abscess without need for further evaluation.
(Option E) Antibiotics alone are not sufficient for management of wound abscesses, but are
used in conjunction with incision and drainage.

171. A 45-year-old man presents to your clinic complaining of weak right wrist flexion.
On examination, small muscles of his right hand are wastes. He is not able to flex his wrist
against resistance. A lesion of which of the following nerves could have led to this
presentation?
A Right ulnar nerve at the wrist.
B Right median nerve at the wrist.
C Right median nerve at the elbow.
D Right ulnar nerve at the elbow.
E Right anterior interosseous nerve.
Option D is correct Most small muscles of the hand including dorsal and palmar interossei,
lumbrical muscles of digits 4 and 5, and hypothenar eminence muscles are innervated by the
ulnar nerve, the paralysis of which results in wasting of most small muscles of the hand
(lumbrical muscles of digits 2 and 3 and muscles of thenar eminence are innervated by the
median nerve). Ulnar nerve also supplies flexor carpi ulnaris which is a wrist flexor. Ulnar
nerve injuries or neuropathies at the level of elbow can cause weakness and wasting of most of
the small muscles of hand and weakness of wrist flexion. Another important muscle involved in
wrist flexion is flexor carpi radialis that is innervated by the median nerve. (Option A)
Although ulnar nerve injury at wrist can affect the function of small muscles of the hand, flexor
carpi ulnaris is innervated proximal to the wrist and ulnar nerve injuries at wrist does not cause
weak wrist flexion. (Option B) Median nerve injury at wrist results in weakness of thenar
muscles and the first two lumbricals but does not affect the action of flexor carpi radialis as it is
innervated proximal to wrist. (Option C) With median nerve injury at the elbow, all of the
muscles innervated by the median nerve and its branches including anterior interosseous nerve
are affected. However, with the exception of the first two lumbrical muscles and muscles of the
thenar eminence, all other small muscles of the hand remain intact as they are supplied by the
ulnar nerve. (Option E) Anterior interosseous nerve does not innervate any of the small
muscles of the hand or main flexors of the wrist. Muscles supplies by the anterior interosseous
nerve include flexor pollicis longus, the radial half of flexor digitorum profundus and pronator
quadratus.

172.Which one of the following nerves supplies the interosseous muscles of the hand?
A Radial nerve.
B Median nerve.
C Ulnar nerve.
D Anterior interosseous nerve.
E Posterior interosseous nerve.
Option C is correct Interosseous muscles are intrinsic hand muscles that abduct (dorsal
interosseous muscles) and adduct (palmar interosseous muscles) the fingers. These muscles are
innervated by deep branch of the ulnar nerve.

173.Which one of the following can cause weakness in small muscles of the hand while
sparing the thenar eminence?
A Ulnar nerve injury.
B Median nerve injury.
C Radial nerve injury.
D Anterior interosseous nerve injury.
E Posterior interosseous nerve injury.
Option A is correct Of the options, ulnar nerve injury can cause weakness of most intrinsic
hand muscles while sparing the thenar eminence. In hand, ulnar nerve innervates all
interosseous muscles, the 3 and 4 lumbrical muscles, and muscles in the hypothenar eminence.
While ulnar nerve injuries affect the function of these muscles, thenar eminence muscles are
spared because they are supplied by the median nerve. (Option B) Low (distal) median nerve
injuries affect the thenar muscles flexor pollicis brevis, abductor pollicis brevis and opponens
pollicis. (Option C) Radial nerve supplies the muscles in the posterior compartment of the
forearm, but no small muscle in the hand, thenar, or hypothenar eminences. (Option D)
Anterior interosseous nerve is a purely motor branch of the median nerve supplies the muscles
flexor pollicis longus, flexor digitorum profundus (2 and 3 fingers), and pronator quadratus, all
of which originating from the forearm, not the hand. (Option E) Posterior interosseous nerve
(PIN) is a branch of the radial nerve that supplies the muscles originating from the posterior
compartment of forearm. PIN injuries do not affect the function of any of the small hand
muscles, thenar, or hypothenar eminence.

174.Lesions of which of the following nerves can lead to wrist drop?


A Median nerve at the wrist.
B Median nerve at the elbow.
C Radial nerve at the wrist.
D Radial nerve at the elbow.
E Ulnar nerve at elbow.
Option D is correct Wrist extension is mainly the action of extensor carpi radialis longus,
extensor carpi radialis brevis, extensor carpi ulnaris, and extensor digitorum, all of which
originating from around the lateral epicondyle of the humerus and are innervated in the vicinity
of the elbow. Therefore, radial nerve injury at the level of the elbow can be associated with
wrist drop. (Option A) Median nerve injury at wrist causes the weakness of thenar eminence
muscles that are flexor pollicis brevis, abductor pollicis brevis and opponens pollicis. These
muscles are not involved in wrist extension. (Option B) Median nerve injury at elbow results
in dysfunction of all the muscles innervated by the median nerve and its motor branch the
anterior interosseous nerve. These muscles include all muscles in the anterior compartment of
the forearm (except flexor carpi ulnaris that is innervated by the ulnar nerve) as well as muscles
of thenar eminence and the 1 and 2 lumbrical muscles. Wrist extension is not affected by
median nerve injuries at elbow. (Option C) Radial nerve injury at wrist is not associated with
any motor function impairment and only causes sensory deficit in the territory or radial nerve
that includes the dorsal aspect of the thumb, index and middle fingers, the radial half of the ring
finger and the dorsal aspect of the first web space. (Option E) Ulnar nerve injury at elbow
affects the flexion of fingers 4 and 5 , wrist flexion (paralysis of the flexor carpi ulnaris muscle
and radial deviation when flexion is attempted), the 3 and 4 lumbrical muscles, interosseous
muscles of the hand and muscles in the hypothenar eminence. Sensory deficits in the territory
of the ulnar nerve can be another physical finding. None of the wrist extensor muscles are
innervated by the ulnar nerve; hence unaffected by its injuries.

175.A 27-year-old woman, typist by profession, presents to your practice with complaint
of right wrist pain and numbness the index, middle fingers and radial half of the ring
fingers. On examination, she is unable to appropriately do the opposition of the thumb
and little finger on the right side. Which one of the following is the most likely diagnosis?
A Ulnar nerve lesion.
B Radial nerve lesion.
C Carpal tunnel syndrome.
D Anterior interosseous nerve syndrome.
E Tendonitis.
Option C is correct The clinical presentation is highly suggestive of carpal tunnel syndrome
(CTS) as the most likely diagnosis. CTS refers to a constellation of signs and symptoms caused
by compression of the median nerve as it travels through the carpal tunnel. CTS presents with
pain and paresthesia, and less commonly weakness in the median nerve distribution in hand.
CTS is the most frequent compressive focal mononeuropathy seen in clinical practice. Classic
CTS is associated with pain or paresthesia (numbness and tingling) in the distribution of the
median nerve territory, with involvement of the first three digits and the radial half of the fourth
digit. Symptoms are typically worse at night and often awaken patients. The patients often
shake or wring their hands or place them under warm running water to alleviate the symptoms.
Symptoms of CTS are usually limited to fingers innervated by the median nerve; however, the
pain and paresthesia may be localized to the wrist or involve the entire hand. It even possible
that sensory symptoms radiate proximally into the forearm, or less frequently to above the
elbow or even the shoulder. CTS symptoms often worsen with activities involving wrist
flexion/extension or arm raising. Some of such activities include typing, reading, driving, and
holding a telephone. In more severe cases, motor involvement follows and results in weakness
or clumsiness when using the hands, such as difficulty holding objects, turning keys or
doorknobs, buttoning clothing, or opening jar lids. Clinical signs may include weakness of
thumb abduction and opposition, and atrophy of the thenar eminence. Fixed sensory loss is
usually a late finding characterized by a distinctive clinical pattern characterized by
involvement of the fingers that are supplied by the median nerve, while the thenar eminence is
spared. This pattern occurs because the palmar sensory cutaneous nerve comes of the median
nerve proximal to the wrist and passes over the carpal tunnel, not through it. (Option A) Ulnar
nerve lesions result in sensory disturbances in the sensory territory of the ulnar nerve. In hand,
the ulnar nerve provides sensation of the little finger and ? of the ring finger, as well as ulnar
part of the palm and dorsum of the hand. Motor involvement results in decreased ability to
abduct/adduct finger (an action of interosseous muscles) as well as weak pincer grip. (Option
B) Radial nerve lesions, presents with disturbed sensation over the dorsal aspect of the thumb,
index finger, middle finger and radial ? half of the ring finger. Thumb opposition is provided
by the action of opponens pollicis that is innervated by median nerve and is not affected in
radial nerve injury. (Option D) Anterior interosseous nerve (AIN) syndrome is a rare condition
associated with entrapment of anterior interosseous nerve. It presents with forearm pain,
inability to make an ‘O.K. sign’ with the thumb and index finger, and positive pinch test in
which the patient pinches in between the thumb and index finger in a fashion similar to tongs
rather than clamps. AIN is a pure motor nerve branching off the median nerve; hence, its
injuries do not cause sensory symptoms. (Option E) Tendonitis is not associated with sensory
disturbances.

176. A 35-year old woman presents to your practice with complaint of decreased ability to
pinch between the thumb and index finger of the right hand. Which one of the following
conditions could have led to this presentation?
Option D is correct Four types of pinching have been described: Tip to tip pinch (pulp of the
thumb against that of the index/middle finger) – this type of pinch is used for holding and
positioning of small objects Pad to pad pinch (pad of the thumb against index / middle finger) –
this type of pinch is used for larger objects or objects that need more stabilization Three jaw
chuck pinch (thumb against tips of index and middle fingers) – it is used for larger or heavier
objects e.g., holding a piece of chuck while writing on the blackboard or holding a pencil
Lateral / key pinch (pulp of thumb against radial/lateral aspect of the index finger) – object is
aligned with forearm and with supination/pronation the object is moved through the space
(using a key to open a door) Different muscles work together to provide precise alignment and
power of different types of pincer grip; however, the two key muscles providing the strength of
a grip are adductor pollicis and dorsal interosseous muscle of the index finger, both of which
are supplied by the ulnar nerve. With decreased power of pinching, an ulnar nerve injury is
more likely to be the underlying cause of this presentation. If the patient with ulnar nerve injury
is asked to grasp a piece of paper between the thumb and lateral aspect of the index finger, loss
of power to adductor pollicis muscle will cause flexion of the distal thumb as the flexor pollicis
longus compensates. Patient makes a “clamp” instead of the naturally-occurring


tongs” to hold the paper (Froment’s sign).
(Option A) Radial nerve and its branch the posterior interosseous supply the extensors of the
forearm, wrist and fingers. Radial nerve lesions do not cause weakness of pinching between the
thumb and index finger. (Option C) For tip-to-tip approximation during pinching between the
thumb and index finger (thumb-index finger pincer grip) the two key muscles involved are
flexor pollicis longus (flexing the interphalangeal joint of the thumb) and flexor digitorum
profundus (flexing the distal interphalangeal joint of the index finger). Both flexor pollicis
longus and the flexor digitorum profundus muscles of fingers II and III are innervated by
anterior interosseous nerve (AIN), a motor branch of the median nerve. Patients with AIN
lesions typically fail to make an “O.K.” sign, as flexion of the interphalangeal joint of the
thumb and the distal interphalangeal joint of the index finger are impaired. Another sensitive
test is the pinch test: a patient with AIN lesions will also not be able to pinch a sheet of paper
between his thumb and index finger, instead they hold the sheet between his extended thumb
and index fingers, giving an appearance similar to a “tong” rather than a “clamp”. The anterior
interosseous nerve, however, does not contribute as significantly to the power of the pincer grip
as do adductor pollicis and dorsal interosseous muscle.

NOTE - Anterior interosseous nerve innervates the following muscles: Flexor pollicis longus –
main flexor of the thumb Flexor digitorum profundus of digits II and III (index and middle
fingers) – flexor of the distal interphalangeal joints of forefinger and middle finger Pronator
quadratus – along with the pronator teres, and while the elbow is flexed to a right angle, turns
the hand so that the palm of the hand faces the floor. (Options B and E) Carpal tunnel
syndrome is caused by compression of the median nerve in the carpal tunnel and presents with
sensory disturbances in palmar aspect of thumb, index finger, middle finger and half of the ring
(4 ) finger. It also causes the weakness of the thenar muscles the abductor pollicis brevis, flexor
pollicis brevis and opponens pollicis. While thumb opposition can be significantly affected in
patients with carpal tunnel syndrome, pinching between thumb and index fingers are NOT
markedly impaired because the key muscles to the action, which are flexor pollicis longus and
flexor digitorum profundus muscle of the index finger are supplied by AIN that does not run in
the carpal tunnel so un affected in carpal tunnel syndrome.

177.A 71-year-old man, who has been diagnosed with bilateral osteoarthritis of his knees
few years ago, presents to the GP clinic you work at with complaint of paresthesia and
numbness of the lateral side of his right leg. The condition is worse at night and improves
after walking for about 10 minutes. Which one of the following is the most important
physical exam finding to consider in this patient?
A Positive straight leg raise test.
B Any tenderness over the right lateral condyle.
C Any loss of sensation over the lateral side of the right leg.
D Weakened dorsiflexion of the right foot.
E Weak distal pulses of the right leg.
Option D is correct The chief complaint of this man is paresthesia and numbness below the
knee on the lateral side of the right leg. This presentation can be caused by common peroneal
nerve injury (CPN). CPN is one of the two main nerves comprising the sciatic nerve. Along
with the tibial nerve, CPN runs down from the buttock down the posterior part of the thigh to
the apex of the popliteal fossa where it changes its path lateral to reach the knee. After
wrapping around the fibular head CPN enters the lateral compartment of the leg. There, this
nerve divides in deep peroneal and superficial peroneal nerves. The proximal lateral aspect of
the leg is innervated by CPN, while the distal part is innervated by superficial peroneal nerve.

CPN through its branches supplies the muscles of ankle


dorsiflexion, eversion and inversion in the anterior and lateral compartments of the leg. CPN
nerve injury can cause weak ankle dorsiflexion (extension) resulting in foot drop and ankle and
foot eversion, as well as toes extension. This patient has presented with sensory deficits
without any obvious motor weakness. He has osteoarthritis and bony processes (osteophytes)
can compress the CPN in the lateral aspect of the knee, resulting in nerve irritation and damage.
It should be noted that, motor weakness, if subtle, might not be always appreciated by the
patient. Therefore, a thorough physical examination is required to reveal any motor deficits that
have remained unnoticed by the patient. In this case, weakened dorsiflexion of the right foot
makes the diagnosis almost certain (D is correct). Weakened dorsiflexion is the characteristic
feature of CPN nerve injury as well as L5 radiculopathy. (Option A) This manifestation can
also be caused by L5 radiculopathy. Lesions of L5 nerve root and CPN nerve presentation in
the leg are similar. However, these two can be differentiated by other historical and clinical
findings. L5 radiculopathy is often associated with low back pain as a prominent feature. The
pain radiates down from the lateral aspect of the thigh, is aggravated by walking (especially
uphill) and bending, and relived by recumbency and lying down. Pain relieved by walking and
aggravated during sleep in unusual for L5 nerve root lesions. If in doubt, tests such as SLR or
slump test can reproduce the symptoms in L5 radiculopathy, but not in CPN lesions. (Option B)
While tenderness over the lateral condyle may be caused by compressing an injured CPN, it
could also be just due to osteoarthritis itself and is not a distinguishing feature. (Option C)
With this presentation, loss of sensation over the lateral aspect of the leg is very likely as the
main complaint is paresthesia and numbness of this area. Such finding adds no significant
diagnostic benefit. (Option E) Weak distal pulses of the leg can be an important finding
pointing towards vascular problems as a concomitant condition. It does not justify or adds any
diagnostic clue to the presentation.

178. A 48-year-old man presents to your practice, complaining of the soreness of his right
elbow. He is an office worker who uses the computer keyboard and mouse a lot at work,
to which he thinks his pain is related. On examination, the right elbow has both normal
appearance and range of motion. There is, however, tenderness over the lateral
epicondyle. Wrist extension while the forearm is extended reproduces the pain. Which
one of the following is the most appropriate treatment option for him?
A Finger immobilization splint.
B Rest and paracetamol.
C Immobilization of the elbow.
D Using encircling support orthotic on forearm.
E Topical diclofenac.
Option D is correct This man has presented with a clinical presentation that is very typical for
lateral epicondylitis. Lateral epicondylitis is caused by degenerative changes of the wrist
extensor muscles at their origin in the humeral lateral epicondyle. The extensor carpi radialis
brevis muscle (ECRB) is the most commonly affected wrist extensor muscle. For patients with
lateral epicondylitis, conservative management with activity modification to prevent further
tendon overuse, counterforce bracing, and home-based exercises to strengthen the wrist
extensor muscles is almost always the best initial management option. Persons with lateral
epicondylitis should be instructed to modify their wrist and elbow movements in a way and
direction that makes the palm facing up rather than down to prevent or limit forearm pronation
in which wrist extension can exert excess force on the wrist extensor tendons. This, however, is
not applicable for all patients because their daily activity, including their job, entails activities
such as typing or computer mouse clicking. Another important step in management is
counterforce bracing of the forearm by placing an encircling band around it. Counterforce
braces are wrapped around the forearm below the elbow and reduce the tension forces on the
wrist extensor tendons and provide rest, which is a very important portion of initial
conservative management. Although evidence is conflicting and controversial, counterforce
braces are still widely used and deemed useful for treatment because they are cheap and easy to
use and may provide benefit during the first 6 weeks of treatment. An encircling support strap
below the elbow acts as a counterforce brace for this patient and the best choice among other
options. (Option A) Fingers extension does not exert force on wrist extensor tendons; therefore,
finger immobilization is not effective in providing rest for the wrist extensor tendons at the
elbow. (Option B) Use of paracetamol as a non-prescribed analgesic has been used for
alleviation of pain in short term. Advice for rest, however, is not practical in this scenario as the
nature of his job entails use of computer keyboard and mouse, during which the causative wrist
and forearm movements are continued. For him, counterforce strapping decreased tension on
wrist extensor tendons while he can continue his daily activities. (Option C) Immobilization of
the elbow has been shown to be inferior to counterforce bracing below the elbow in pain
reduction and increasing strength of grip. (Option E) Although controversial, short term use of
topical (or sometimes oral) non-steroidal anti-inflammatory drugs (NSAIDs) may be
considered for acute and very painful cases. However, rest from the implicated activity by
means of counterforce bracing remains the most appropriate initial option.

179. Janet is a 43-year-old IT worker, who has presented to you with complaint of pain in
the right elbow. She describes that the pain increases through the day as she does her job
which includes lots of typing with computer keyboard and use of computer mouse. On
examinations, the elbow looks normal with preserved range of motion, but slight pain at
the extreme of pronation. Tenderness is elicited over the lateral epicondyle of the right
humerus when palpated deeply. The pain is reproduced on resisted extension of the wrist
while her forearm is fully extended. The arm is otherwise normal. Which one of the
following would be the best treatment option for Janet at this stage?
A Immobilization of the elbow.
B Corticosteroid injection.
C Encircling support strap for the right forearm.
D Oral NSAIDs.
E Topical NSAIDs.
Option C is correct The scenario is a typical description of lateral epicondylitis, which is
caused by degenerative changes of the wrist extensor muscles at their origin in the humeral
lateral epicondyle. Of these muscles, the extensor carpi radialis brevis muscle (ECRB) is the
most commonly affected one. The diagnosis is almost always clinical, but x-rays, ultrasound, or
MRI can be used when the diagnosis is in doubt or for further management planning in
refractory cases. A myriad of treatment options has been investigated and applied for treatment
of lateral epicondylitis; however, conservative management including activity modification to
prevent further tendon overuse, counterforce bracing, and home-based exercises to increase the
strength of the extensor muscles is almost always the best initial option to consider. Although
controversial, short term use of topical (or sometimes oral) non-steroidal antiinflammatory
drugs (NSAIDs) may be prescribed for acute and very painful cases. Application of ice can also
help with pain reduction in an acute phase. The outcome is then reviewed, often in 4 to 6 or 8
weeks for further management. Patients with lateral epicondylitis should be advised and taught
to modify their arm movements in a way that most of their activities are done with the palm
facing up rather down in an attempt to prevent or limit forearm pronation in which wrist
extension can exert excess force on the wrist extensor tendons. However, for some patients,
whose daily activity, including their job, entails tasks such as typing or computer mouse
clicking, such instruction may be impractical to follow. Another important step in management
is counterforce bracing of the forearm by placing an encircling band around it. Counterforce
braces are strapped around the forearm below the elbow to reduce the tension forces on the
wrist extensor tendons and provide rest, an important part of the initial management, for the
extensor tendons. Despite conflicting evidence, counterforce braces are still widely used.
Counterforce braces are costeffective and easy to use, and may provide benefit during the first
6 weeks of treatment by pain relief and grip strengthening. An encircling support strap below
the elbow acts as a counterforce brace for Janet and is the best choice among the given options.
(Option A) Immobilization of the elbow has been shown to be inferior to counterforce bracing
below the elbow in pain reduction and increasing grip strength. (Option B) Intralesional
corticosteroid injection is reserved for patients with refractory severe pain and disability despite
conservative management. Corticosteroid injection is associated with a rapid response in short
term but poorer outcome in long term. (Options D and E) Lateral epicondylitis is a tendinosis
rather than tendinitis with minimum or absent inflammatory changes on histopathology.
Despite the absence of apparent inflammation, NSAIDs either topical or oral (if not
contraindicated) have shown benefit in short term (up to 14 days) in some studies. Although
controversial, they are often used for patients with lateral epicondylitis, specially during the
acute initial phase. Diclofenac is the preferred NSAIDs as it has showed efficacy in some
studies (to date naproxen has proved not more effective than placebo).

180.A 45-year-old man presents to your GP clinic with complaint of a painful right elbow.
The pain is felt laterally radiating to the back of the forearm, aggravates with activity and
relieves with rest. He is a carpenter and makes cabinets. His job includes lots of sawing,
drilling, and using screw drivers. On examination, you notice a tender point 1-2 cm distal
to the right lateral epicondyle. Extension of the wrist with the elbow in full extension
brings up the pain. Which one of the following is the most appropriate next step in
management?
A Physiotherapy.
B Advising himr to quit his job.
C Intralesional corticosteroids.
D Surgery.
E Band support below the elbow.
Option E is correct The scenario describes a typical case of lateral epicondylitis, also known
as ‘lateral tennis elbow’, which is the most common overuse syndrome of the upper limb and is
related to excessive wrist extension. The condition is typically seen in persons of 40-50 years
of age. Affected individuals often report an insidious onset, with a history of overuse without
specific trauma almost always present in the history. Patients with the condition complain of
pain over the lateral elbow that worsens with activity and alleviates with rest. The pain often
can be pinpointed 1-2 cm distal to attachment of the extensor carpi radialis brevis muscle
(ECRB) tendon to the lateral epicondyle. Lateral epicondylitis is benign self-limiting condition
that often resolves within 6 to 12 months in most cases. Activity modification, counterforce
bracing, and home-based exercise programs constitute the initial conservative management in
almost all cases. This patient should be advised to modify all his daily activities in a way that
limits forearm pronation, the position in which wrist extension exerts great force on wrist flexor
tendons. This often includes using a palm-up position to avoid or limit pronation. Of the given
options band support below the elbow (counterforce bracing) is the most appropriate option to
consider for him. Counterforce braces encircle the forearm 6 to 10 cm to reduce the tension
forces on the wrist extensor tendons and provide rest for the extensor tendons. Despite
conflicting evidence, they are still widely used in management of lateral epicondylitis.
Counterforce braces are inexpensive, convenient, and have shown to decrease pain and increase
grip strength in most cases within 3-6 weeks.
Application of ice and short-term use of topical or oral non-steroidal anti-inflammatory drugs
(NSAIDs) are other measures that might be considered for acute management of lateral
epicondylitis. (Option A) Some studies have shown that physiotherapy can be more effective
than a wait-and-watch approach or topical corticosteroid injection in both the short and long-
term outcomes; however, it is often reserved for those who fail to respond to the above
measures such as home-based exercise programs. (Option B) Although this patient has
developed lateral epicondylitis most likely due to the overuse of his elbow in his carrier,
advising him to quit the job he is doing for a living is not appropriate and practical. (Options C)
Intralesional corticosteroid injection is reserved for patients with refractory severe pain and
disability despite conservative management. Corticosteroid injection is associated with a rapid
response in short term (usually within 4 weeks) but poorer outcome in long term. (Option D)
Surgery is rarely used for treatment of patients who fail to response to all available measure for
treatment of lateral epicondylitis

181.Overuse of which one the following is most likely the underlying mechanism of lateral
epicondylitis?
A Wrist extension.
B Wrist flexion.
C Elbow flexion.
D Elbow extension.
E Wrist pronation.
Option A is correct Lateral epicondylitis, also referred to as ‘lateral tennis elbow’ is the most
common overuse syndrome related to excessive wrist extension. The lateral epicondyle of the
humerus serves as the bony common origin of the wrist extensors. Overuse injury to the
extensor carpi radialis brevis muscle (ECRB), which is felt at the tip of lateral epicondyle and
occasionally, the extensor digitorum communis muscle (EDC) (felt just posterior and lateral to
the tip of lateral epicondyle) constitutes lateral epicondylitis. ‘Tennis elbow’ and ‘lateral
epicondylitis’ are, however, misnomers because the condition is seen more commonly in non-
tennis players than tennis players and the underlying pathophysiology is degenerative rather
than inflammatory. Microscopic evaluation of the tendons does not show signs of inflammation,
but rather angiofibroblastic degeneration and collagen disarray. Light microscopy reveals both
an excess of fibroblasts and blood vessels that are consistent with neovessels or angiogenesis.
The injury to the tendon often occurs 1 to 2 cm below the attachment point of ECRB to lateral
epicondyle.
182. Which one of the following wrist movements is most likely to reproduce the pain in a
patient with lateral epicondylitis?
A Radial deviation.
B Resisted extension.
C Supination.
D Ulnar deviation.
E Resisted flexion.
Option B is correct Lateral epicondylitis, also referred to as ‘lateral tennis elbow’ is the most
common overuse syndrome related to excessive wrist extension. Patients with lateral
epicondylitis typically present with pain in the lateral aspect of the forearm that may radiate
down into the forearm. There is often maximal tenderness 1-2 cm distal to the origin of the
extensor carpi radialis brevis muscle (ECRB) at the lateral epicondyle. The pain in lateral
epicondylitis is reproduced with resisted wrist extension with the elbow fully extended, or
passive terminal wrist extension while the elbow is fully extended. Resisted extension of the
middle finger is also painful secondary to stress placed on the ECRB tendon, as it is
preferentially stressed in this position when it must contract synergistically to anchor the third
metacarpal, such that extension can take place at the digits. Resisted pronation of the wrist and
gripping while the elbow is extended can also bring up the pain. Of the given options, only
resisted extension of the wrist can reproduce the pain in a patient with lateral epicondylitis.
(Options A and D) Radial and ulnar deviation of the wrist do not exert a significant force on
extensor muscles and are unlikely to reproduce the pain. (Option C) supination of the wrist,
especially if resisted, can reproduce the pain in medial epicondylitis (golf elbow), as does the
resisted flexion of the wrist (option E).

183. Which one of the following is most suggestive of sciatic nerve injury?
A Absent ankle reflex.
B Foot drop.
C Inability to flex the hip.
D Decreased sensation over the anterior thigh and medial leg.
E Intervertebral disc prolapse at L2/L3 level.
Option A is correct The sciatic nerve is derived from the lumbosacral plexus. Once formed, it
leaves the pelvis and enters the gluteal region via greater sciatic foramen. It emerges inferiorly
to the piriformis muscle, and enters the posterior thigh by passing deep to the long head of the
biceps femoris, and descends in an inferolateral direction. Within the posterior thigh, the nerve
gives off branches to the hamstring muscles and adductor magnus. When the sciatic nerve
reaches the apex of the popliteal fossa, it terminates by bifurcating into the tibial and common
fibular nerves. The sciatic nerve can be described as tibial and common peroneal nerves
bundled together in at sheath of connective tissue. Common peroneal nerve and tibial nerve
usually separate at the apex of the popliteal fossa, however in a minority of individuals, they
separate as they leave the pelvis. Sciatic nerve Injury manifests as paralysis of the hamstring
muscles (knee flexion weakness) and all the muscles below the knee. All sensation of the leg
except the medial aspect is impaired. Absent or weak ankle reflex is most specific to sciatic
nerve injury among other options.. (Option B) Foot drop is seen in sciatic as well as common
peroneal nerve injuries and is not specific to sciatic nerve injury. (Option C) Hip flexion is the
action of L1-L2 nerve roots. Hip flexion is not an action of the sciatic nerve (L4, L5, S1, S2,
S3). (Option D) Decreased sensation over the anterior thigh and medial leg is caused by
femoral nerve injuries. (Option E) Intervertebral disc lesion at L2/L3 compromises the action
of L3 nerve root, which does not contribute to the formation of the sciatic nerve.

184. Which one of the following can be a presentation associated with L5/S1 disc prolapse?
A Absent knee reflex.
B Absent ankle reflex.
C Impaired ankle dorsiflexion.
D Impaired ankle eversion.
E Impaired sensation over the lateral aspect of the leg.
Option B is correct In L5/S1 disc prolapse, which often occurs laterally, the herniated disc
compresses the S1 nerve root causing S1 radiculopathy. S1 radiculopathy presents with pain
radiating down the posterior aspect of the leg into the foot from the back. On examination,
weakness of plantar flexion is specific.Weakness of leg extension (gluteus maximus) and toe
flexion are other possible features. Sensation is generally reduced on the posterior aspect of the
leg and the lateral edge of the foot. Loss of the ankle reflex is typical. (Option A) Knee reflex
is an action of L3 and L4 nerve roots. (Options C, D and E) Ankle dorsiflexion, eversion, and
sensation over the lateral aspect of the leg are innervated by L5 that can be affected in L4/L5
disc prolapse.

185. A 26-year-old man develops weakness of foot dorsiflexion and eversion. Which nerve
is most likely to have been damaged?
A Common peroneal nerve.
B Tibial nerve.
C Sciatic nerve.
D Lumbosacral plexus.
E Lumbar nerve roots.
Option A is correct Ankle eversion is an action of 3 muscles in the lateral compartment of the
leg, including: Peroneous (fibularis) longus Peroneous (fibularis) brevis Peroneous tertius
These muscles are innervated by superficial peroneal nerve, one of the two main branches of
the common peroneal nerve. Ankle dorsiflexion is the action of the following: Tibialis anterior
Extensor hallucis longus Extensor digitorum longus Peroneous (fibularis) tertius Peroneous
tertius is innervated by the superficial peroneal nerve. The other three are supplied by deep
peroneal nerve, the other main branch of the common peroneal nerve. With both ankle
dorsiflexion and eversion affected, the common peroneal nerve is most likely to be damaged.
(Option B) Tibial nerve innervates the posterior compartment of the leg. Muscles in this
compartment are mostly ankle plantar flexors. With tibial nerve injury plantar flexion and ankle
jerk will be affected. (Option C) With sciatic nerve involvement, global weakness and
impairment of all ankle and foot movements is expected. Ankle reflex will be affected too.
Weakness of the knee flexion could also be problem. (Option D) Clinical findings associated
with lumbosacral plexus problems vary depending on the affected nerve root. S1 radiculopathy
presents similar to tibial nerve injury with impaired plantar flexion, ankle jerk and sensory
disturbances in the back of the leg and most parts of the sole of the foot. (Option E) Lumbar
nerve root problems cause deficits in the movements and sensation of the hip, thigh and knee
extension and reflex.
186. A 75-year-old woman underwent total left hip replacement surgery yesterday. Today,
she has developed weakness and numbness of her left foot. On examination, there is
weakness of all the left ankle movements including dorsiflexion, plantar flexion, eversion
and inversion, and numbness over the dorsum and sole of her foot and lateral aspect of
her leg. The left ankle jerk is absent. Which one of the following is the most likely site of
the lesion?
A Left common peroneal nerve.
B Left femoral nerve.
C Left obturator nerve.
D Left sciatic nerve.
E Left tibial nerve.
Option D is correct Ankle dorsiflexion and eversion are actions of the deep peroneal and
superficial peroneal nerves, respectively. Inversion is supplied by deep peroneal nerve (anterior
tibialis muscle) and tibialis nerve (posterior tibialis muscle). Plantar flexion and ankle reflex
are supplied by the tibial nerve. The efferent limb of the ankle jerk reflex is innervated by S1
and to lesser extend S2 fibers within this nerve. The pattern of the sensory impairment indicates
that both nerves are affected. As a matter of fact, this woman has a clinical presentation
consistent with both common peroneal and tibial nerves injuries. When such clinical
presentation is encountered, injury to the sciatic nerve is the most likely explanation. This can
be the most likely diagnosis given the history of hip surgery. The sciatic nerve is derived from
lumbosacral plexus. Once formed, it leaves the pelvis and enters the gluteal region via greater
sciatic foramen. It emerges inferiorly to the piriformis muscle, and enters the posterior thigh by
passing deep to the long head of the biceps femoris, and descends in an inferolateral direction.
Within the posterior thigh, the nerve gives off branches to the hamstring muscles and adductor
magnus. When the sciatic nerve reaches the apex of the popliteal fossa, it terminates by
bifurcating into the tibial and common fibular (peroneal) nerves. NOTE - The sciatic nerve can
be described as two individual nerves bundled together in a sheath of connective tissue. These
two nerves are the tibial and common peroneal nerves. These two nerves usually separate at the
apex of the popliteal fossa, however in a minority or persons, they separate as they leave the
pelvis. Sciatic nerve injury can occur during hip surgeries. Stretch, compression, ischemia, and
direct damage are primary mechanisms. The lithotomy, frog leg, and sitting positions have
been implicated in perioperative injury to this nerve. Regional anesthetic techniques and hip
arthroplasty may also cause injury. The common peroneal component is more common to be
affected because it is more superficial compared with the tibial component. Sciatic nerve Injury
manifests as paralysis of the hamstring muscles (knee flexion weakness) and all the muscles
below the knee. All sensation below the knee is affected with the exception of the sensation of
the medial aspect of the leg and lateral aspect of the foot, which remains intact because these
two areas are supplied by saphenous nerve (a sensory branch of the femoral nerve) and sural
nerve, respectively. (Option A) Left common peroneal nerve injury results in foot drop (loss of
ankle dorsiflexion), weak inversion and inability of the patient to evert the ankle. Sensory
impairment occurs over the lateral aspect of the leg and dorsum of the left foot (except the
lateral margin of left foot). (Option B) Left femoral nerve injury causes global weakness of all
muscles in the anterior compartment of the thigh, impaired knee jerk and sensory disturbances
of the lateral, and anteromedial surfaces of the thigh, as well as the medial aspect of the leg.
(Option C) Obturator nerve consists of L2, L3, and L4 nerve roots. It innervates the muscles
adductor longus, adductor brevis, adductor magnus, gracilis, and obturator externus. It provides,
along with the femoral nerve, the sensation of the medial aspect of the thigh. Obturator nerve
injury presents with medial thigh or groin pain, weakness of leg adduction and sensory loss in
the medial thigh. (Option E) Tibial nerve damage results in weak plantar flexion. Impaired
ankle jerk and sensory disturbances over the posterior aspect of the leg and sole of the foot are
other manifestations.

187. A 45-year-old man presents with loss of inversion, eversion, dorsiflexion and plantar
flexion of his left foot. His left knee jerk is preserved, but that of the ankle is weak. Which
one of the following can be the cause of this presentation?
A Sciatic nerve injury.
B Common peroneal nerve injury.
C Tibial nerve injury.
D L4/L5 disc herniation.
E Femoral nerve damage.
Option A is correct Loss of ankle inversion in this man can be either due to dysfunction of
deep peroneal nerve (a branch of common peroneal nerve), or tibial nerve that supply the two
main ankle invertors: tibialis anterior and tibialis posterior. Ankle eversion is the action of the
muscles in the lateral compartment of the leg: peroneous (fibularis) longus, peroneous (fibularis)
brevis and peroneous tertius. These muscles are supplied by the other main branch of the
common peroneal nerve, superficial peroneal nerve. Plantar flexion of the ankle is mediated
mostly by the muscles in the posterior compartment of the leg, all of which are innervated by
tibial nerve. On the other hand, ankle reflex in this patient is weak suggesting the involvement
of S1 (and to a lesser extend S2). In fact the clinical picture is only justified if both common
peroneal and tibial nerves are involved and affected. This can be explained by injury to sciatic
nerve. Tibial nerve and common peroneal (fibular) nerve exit from the pelvis together
contained in a sheath of connective tissue to form the sciatic trunk (nerve). As a matter of fact
sciatic nerve is common peroneal nerve and tibial nerve running down together in the buttock
and back of the thigh. In the distal thigh these two nerves separate and run different paths.
Injury to sciatic nerve (trunk) can present with motor and sensory deficits attributable to tibial
nerve and common peroneal nerve. The clinical findings include: Common peroneal nerve:
Weak or absent ankle eversion Weak or absent ankle dorsiflexion (drop foot) Weak or absent
toes extension Weak ankle inversion Paresthesia / impaired sensation over the lateral aspect of
the leg and dorsum of the foot (except the areas supplied by saphenous and sural nerves) Tibial
nerve: Weak or absent plantar flexion (the patient cannot stand on the ball of his foot on the
affected side) Weak ankle inversion Weak or absent toes flexion Weak or absent ankle jerk
reflex Paresthesia/impaired sensation over the posterior part of the leg and sole of the foot
(except those areas supplied by saphenous and sural nerves) In summary, injury to sciatic trunk
causes weakness of ankle movements in all directions, impaired or absent ankle reflex and
impaired sensation below the knee except the medial aspect of the leg and the outer part of the
foot. Knee reflex usually remains intact. (Option B) Common peroneal nerve damage (in
isolation) explains loss of sensation over the lateral leg, as well as weakened dorsiflexion,
eversion and inversion, but not the weak plantar flexion. (Option C) Tibial nerve injury (in
isloation) explains absence of the ankle plantar flexion and impaired ankle jerk, but not the
other clinical aspects. (Option D) Lateral L4/L5 disc herniation affects the L5 nerve root and
results in L5 radiculopathy. L5 and common peroneal nerve injuries present similarly. L5
radiculopathy does not cause weak plantar flexion and impaired ankle reflex. (Option E)
Femoral nerve supplies all the muscles in the anterior compartment of the thigh, as well as the
sensation of the thigh except the posterior part (supplied by sciatic trunk) and a small medial
part (supplied by obturator nerve), and through the saphenous branch, the medial aspect of the
leg and outer part of the foot.

188. One week after a knee surgery, a 24-year-old man presents with complaints of
numbness and paresthesia of the right leg. On examination, there is foot drop and
weakness of dorsiflexion and eversion of the right ankle. Sensation over the outer aspect
of the right leg is also lost. Right ankle jerk is intact. Which one of the following nerves is
most likely to have caused such presentation?
A L4 nerve root.
B L5 nerve root.
C Common peroneal nerve.
D Tibial nerve.
E Sciatic nerve.
Option C is correct The clinical picture suggests damage to common peroneal nerve as an
adverse outcome of the knee surgery. Foot drop is the result of weakness of the ankle
dorsiflexors (extensors) innervated by the deep peroneal nerve (with the exception of
peroneous tertius, a weak ankle dorsiflexor located in the lateral compartment of the leg, and
supplied by the superficial peroneal nerve). Sensation over the outer aspect of the leg is
supplied by the common peroneal nerve and its superficial branch. These together make the
common peroneal nerve injury the most likely explanation. After branching off the sciatic
nerve, the common peroneal nerve runs laterally, wraps around the fibular head and enters the
leg. The nerve is superficial in this region and susceptible to injuries during knee surgery, with
compression or in trauma. (Option A) L4 nerve root is in the lumbar area. L4 radiculopathy
can cause weakened knee reflex and partially impaired ankle inversion, but does not explain
other manifestations. (Option B) L5 nerve root damage has a very similar clinical presentation
to that of the common peroneal nerve injury; however, the history of knee surgery makes the
latter more likely. (Option D)Tibial nerve damage causes impaired ankle jerk, weak or absent
plantar flexion and weak ankle inversion. Sensory impairment due to tibial nerve injury affects
the back the leg and most parts of the sole of the foot. (Option E) Sciatic trunk (nerve) is above
the knee, and unlikely to have been affected by the knee surgery. Moreover, sciatic damage
affects the ankle and foot movements globally, and gives a clinical picture consistent with
injuries of both common peroneal and tibial nerves at the same time.

189. To perform a dilation and curettage on a woman, she is placed in lithotomy position,
after which she is found to have developed a nerve injury. Which one of the muscles is
most likely to be affected by this injury?
A Extensor hallucis longus.
B Flexor digitorum longus.
C Tibialis posterior.
D Soleus.
E Quadriceps.
Option A is correct The most commonly injured lower extremity nerve in patients undergoing
surgery in lithotomy position is the common peroneal nerve. The injury is thought to be
secondary to compression of the nerve between the lateral aspect the fibular head and the bar
holding the leg, especially when candy cane stirrups are used (see below). Low weight,
smoking and prolonged surgery are risk factors for this injury.

level
of the popliteal fossa, the sciatic nerve bifurcates into tibial and common peroneal (fibular)
nerves. Medial to the course of the biceps femoris muscle, common peroneal nerve runs
laterally and inferiorly, wraps around the fibular head and enters the lateral compartment of the
leg. It passes between the attachments of the fibularis longus muscle, where the nerve divides
into the superficial and deep fibular (peroneal) nerves. Injury to common peroneal nerve can
cause dysfunction of both deep and superficial peroneal nerves that can present with the
following: Loss of dorsiflexion and eversion of the foot (equinovarus deformity) Sensory
manifestations along the anterolateral border of the leg and dorsum of the toes except those
supplied by saphenous and sural nerves. The following muscles of the leg are innervated by
deep and superficial branches of the common peroneal nerve: Tibialis anterior Extensor
digitorum longus Extensor hallucis longus Peroneous tertius Peroneous longus Peroneous
brevis Of the given options, only extensor hallucis longus can be affected as a result of
common peroneal nerve injury in lithotomy position. In fact, inability to extend the greater toe
is a common finding in patients with injured common peroneal nerve. (Options B, C and D)
Soleus (plantar flexor), tibilalis posterior (plantar flexor and ankle inverter), and flexor muscles
of the foot, including flexor digitorum longus are innervated by tibial nerve; therefore,
unaffected with common peroneal nerve injury. (Option E) Quadriceps muscle is supplied by
femoral nerve and unaffected by common peroneal injury. NOTE - L5 radiculopathy can
present exactly similar to the common peroneal nerve injury.

190. A 37-year-old man is brought to the emergency department after he sustained a


motor vehicle accident. Upon arrival, a quick review excludes any life-threatening
internal injuries. He has a deformed right leg highly indicative of a fracture. There is a
bleeding laceration over the deformity. An X-ray shows fractures of the tibial and fibula.
As the initial management, intravenous fluid and antibiotics are started. Which one of the
following is the most appropriate next step in management?
A External fixation.
B Internal fixation.
C Tetanus prophylaxis.
D Wound debridment.
E Wound closure.
Option C is correct The vignette describes a typical case of open fracture of the right leg.
Open fractures are characterized by the fracture exposure to the environment trough a breach of
the skin and underlying soft tissue. Such fractures are particularly at increased risk of infection
that can compromise the fracture and healing process. A stepwise approach to any open
fracture is as follows: 1. Removal any gross contamination off the wound. Provisional
irrigation or debridment should be avoided at this stage. 2. Taking photographs of the wound
and covering it with wet sterile cover – (the photographs are taken, so that other treating
physicians do not need to uncover the wound to see it) 3. Analgesics, preferably intravenously
for pain control and preparation for reduction in the emergency department if there is
significant misalignment 4. Reduction of the fracture and correction of misalignments by gentle
traction as much as possible 5. Starting the patient on intravenous prophylactic antibiotics 6.
Tetanus prophylaxis if indicated 7. Obtaining x-rays 8. Urgent arrangement for transferring the
patient to the operating room for surgical wound debridment and definite treatment of the
fracture and dislocation There is no comment regarding significant misalignment. The patient’s
wound should be covered by wet dressing after photographs are taken (if possible). He also has
been started on intravenous antibiotics; therefore, the next best step for him would tetanus
prophylaxis if indicated. (Options A and B) Fixation of fractures is performed in the operating
room after surgical irrigation and wound debridment. (Option D) Wound debridment including
surgical removal of all devitalized soft tissues and bone is a crucial step in management of any
open fracture/dislocation because infections may ensue and result in poor outcomes. However,
debridment should be performed in the sterile environment of the operating room, not in the
non-sterile environment of the emergency department where more exploration of the wound
can lead to increased risk of infection.

191. A 35-year-old woman is brought to the emergency department after she sustained a
motor vehicle accident as a front seat passenger and had her right ankle injured. On
examination, her vital signs are stable. The left ankle joint is laterally displaced and there
is a 13-cm laceration over the joint. The dorsal pedis pulse of the left foot is barely felt
and the foot is cold and pale. The condition is diagnosed as open fracture/dislocation of
the ankle joint. Which one of the following is the most important step in preventing
wound infection and ensuing complications?
A Intravenous antibiotics.
B Wound debridment.
C Tetanus prophylaxis.
D Reduction of the displacement.
E X-ray of the joint.
Option B is correct This patient has an open fracture/dislocation of the ankle joint with
vascular compromise. Open fractures/dislocations are characterized by the bone and/or joint
exposure to external environment, or when the fracture or dislocation is caused by blunt or
penetrating forces sufficient to disrupt or penetrate skin, subcutaneous tissue, muscle fascia,
muscle, and/or the bone or joint. Open fractures are often contaminated by foreign material
(e.g., clothing, grass, dirt, gravel), dead or devitalized tissue and bacteria. The order in which
an open fracture/dislocation is managed is as follows: 1. Removal any gross contamination off
the wound. Provisional irrigation or debridment should be avoided at this stage. 2. Taking
photographs of the wound and covering it with wet sterile cover – (the photographs are taken,
so that other treating physicians do not need to uncover the wound to see it) 3. Analgesics,
preferably intravenously for pain control and preparation for reduction in the emergency
department if there is significant misalignment 4. Reduction of the fracture and correction of
misalignments by gentle traction as much as possible 5. Starting the patient on intravenous
prophylactic antibiotics 6. Tetanus prophylaxis if indicated 7. Obtaining x-rays 8. Urgent
arrangement for transferring the patient to the operating room for surgical wound debridment
and definite treatment of the fracture and dislocation The question, however, asks about the
most important step in preventing the infections and consequent complications. Debridment of
all devitalized soft tissues and bone all the way down to the bone is the most crucial step in
preventing the infection and the resultant adverse outcomes such as delayed healing,
amputation, etc. Prophylactic antibiotics are started prior to debridment and definite treatment,
and are of great importance; however, debridment remains the cardinal step in preventing
infections. Tetanus prophylaxis should be given to the patient if indicated but this does not
prevent other infections if adequate debridment is not carried out. Wound debridment is always
crucial but should be performed in the operating room, after the above measures have been
undertaken; therefore, it is not a priority but an extremely important step in preventing
infections.

192. A 28-year-old man is brought to the emergency department after he had an accident,
while he was driving a motorcycle, and had his right ankle injured. On examination, his
vital signs are stable. The right ankle joint is laterally displaced and there is a laceration
over the joint. Which one of the following is the most important initial step in
management?
A Wound debridment.
B Tetanus immunization.
C Intravenous antibiotics.
D Reduction of the displacement.
E X-ray of the joint.
Option D is correct Open fractures and/or dislocations are defined as the bone and/or joint
being exposed to the external environment, or when the fracture or dislocation is caused by
blunt or penetrating force sufficient to disrupt or penetrate skin, subcutaneous tissue, muscle
fascia, muscle, and/or the bone or joint. Open fractures are often contaminated by foreign
material (e.g., clothing, grass, dirt, gravel), dead or devitalized tissue and bacteria. Always
follow the following rules in open fractures / dislocations 1. Remove any gross contamination
off the wound. No provisional irrigation or debridment is performed at this stage. 2. Take
photographs of the wound and dress it with wet sterile cover – (the photographs are taken, so
that other treating physicians do not need to uncover the wound to see it) 3. Give patient
analgesic (preferably intravenously to both control the pain and prepare for a reduction in the
emergency department). Morphine is a good option. 4. By gentle traction, reduce the fracture
and correct misalignments as much as possible. 5. Start the patient on intravenous prophylactic
antibiotics. 6. Give the patient tetanus prophylaxis if indicated. 7. Obtain X-rays. 8. Urgently
arrange for transferring the patient to the operating room for surgical wound debridment and
definite treatment of the fracture and dislocation. NOTE - initial irrigation or debridment in the
emergency department is not recommended and should be avoided. In this scenario the most
appropriate initial management would be reduction of the dislocation by gentle traction after
initial non-surgical removal of any gross contamination, and sedation. The patient then should
be started on intravenous antibiotics and receive tetanus prophylaxis. X-ray of the joint is
required pre-operatively to visualize the anatomical disruption. Wound debridment is crucial
but should be performed in the operating room ideally with 1-2 hours of presentation. NOTE -
In open fractures, wound treatment may sometimes be more time-consuming and require more
work than the treatment of the fracture itself. Debridement may be carried out several times
before and after definitive fracture treatment is performed.

193.A 72-year-old man was brought to the emergency department after he sustained a
fracture of his left hip following a fall at home. Initial pain management was considered
and the operation carried out with intramedullary nail placement. Today, he is being
discharged. Which one of the following is the most important management option to
consider for him on discharge?
A Painkillers.
B Walker.
C Crutches.
D Low molecular weight heparin.
E Warfarin.
Option A is correct According to guidelines provided by the Agency for Clinical Innovation
(ACI), it is crucial to mobilize patients with hip fracture within 24 hours of the surgery. Pain
following hip fracture should be presumed severe and continuous. Effective pain management
is a primary goal for patients with a hip fracture, because immobility caused by pain has been
associated with the increased risk of the following: Pressure ulcers Pneumonia Venous
thromboembolism (VTE) With greater pain the likelihood of the following will increase:
Delirium Sleep disturbances Depression Pain control is of significant importance for early
mobilization of patients who have undergone hip surgery, because the pain brought on by
physical activity is the most important factor preventing from being mobile; therefore pain
control with painkillers play a crucial role in early mobilization. Pain is often undertreated or
poorly controlled in orhtogeriatric patients due to: 1. Reluctance or inability of an older patient
to request analgesia 2. Reluctance of medical staff to prescribe analgesia in older frail patients
3. Cognitive impairment in patients, making assessment of pain level difficult Pain
management techniques for patients with hip fracture include pharmacological approaches such
as paracetamol and opioids and use of femoral nerve block. (Options B and C) Walking aids
such as crutches or walkers should be advised to provide more convenient mobilization, but
they are not superior to analgesics, because with pain the patient is unlikely to become
physically active and changes posture (e.g. from sitting to standing, walking). (Option D)
LMWH is also important in prevention of VTE in this patient, but this has already been started
immediately after the surgery and continued during the hospital stay. (Option E) Warfarin is
not routinely used for prophylaxis of VTE in patients with hip surgery.

16. Which one of the following is most important to look for on an ultrasound scan in a
patient with jaundice?
A Gallstones.
B Head of the pancreas.
C Dilation of bile ducts.
D Hepatic masses.
E Thickening of the gallbladder wall.
Option C is correct
Jaundice can be caused by a variety of etiologies that broadly are categorized into extrahepatic
or intrahepatic. A careful history and physical examination is essential. Laboratory tests follow
for further evaluation.. The most important laboratory values to consider are bilirubin (total and
unconjugated), hepatic transaminases, gamma glutamyl transpeptidase (GGT), alkaline
phosphatase and coagulation profile. If the history, physical exam, and initial laboratory studies
suggest obstruction of the biliary tree as the underlying pathophysiology of jaundice, an
imaging study is indicated to differentiate extrahepatic and intrahepatic causes of cholestatic
jaundice. The imaging tests that can be used are abdominal ultrasonography, endoscopic
ultrasonography, abdominal CT scan, ERCP, and MRI. Abdominal ultrasonography is the
initial imaging modality of choice, becasue it is inexpensive and readily available. In evaluation
of jaundice, the most important finding to look for is obstruction of the biliary tree, evident by
biliary duct dilation. Sensitivity of abdominal sonography ranges between 55% and 91%
according to different studies. The sensitivity increases with increased serum bilirubin and
duration of the jaundice. Ultrasound can also show stones; however, common bile duct (CBD)
stones may not be visualized enough due to the duodenal gas obscuring the distal CBD.
Gallstones, hepatic masses and pancreas may also be visualized on ultrasonography.
Gallbladder thickening suggests cholecystitis. Cholecystitis per se does not result in jaundice,
unless cholangitis or distal biliary tree obstruction occurs.
45. A 67-year-old man presents to your office with complaints of reduced dexterity
of his right hand and difficulty grasping objects. His hand is shown in the following
photograph. Which one of the following investigations will confirm the diagnosis?

A X-ray of the hand.


B MRI.
C Ultrasonography of the hand.
D CT scan of the hand.
E Blood sugar level.
Option C is correct The picture shows palmar skin puckering and a nodule at the base of
the fourth finger, indicative of Dupuytren contracture. The diagnosis is almost always
made clinically and no imaging is needed.No routine imaging is necessary, but
ultrasonography can demonstrate thickening of the palmar fascia and the presence of a
nodule. At cases, ultrasonography of the hand is considered if the patient is planned for
injection of corticosteroids into the cord or the nodule, in an attempt to avoid the flexor
tendon. Since there is an association between Dupuytren contracture and diabetes
mellitus, blood sugar level can be measured if the physician is suspicious of diabetes
based on history and/or clinical findings. X-ray, CT scan or MRI is almost never
indicated. Ultrasound of the palm in Dupuytren disease of the fourth digit; the Dupuytren
nodule and thickening of the palmar fascia with puckering of the skin is noted. green
arrow: flexor tendon; red arrow: dupuytren nodule; blue arrow: thickened palmar fascia;
yellow arrow: puckering of the skin.

TOPIC REVIEW Treatment options for Dupuytren contracture: Physiotherapy (for


mild cases): e.g. heat and ultrasonographic waves, physical therapy, custom splint or
brace for finger stretching Occupational therapy Intralesional corticosteroid injection
Collagenase injection Other measures: 5-fluorouracil, imiquimod, botulinum toxin, and
hyperbaric oxygen Surgery: surgery is indicated if MCP contracture is more than 30° or
there is contracture of PIP.
50. A 57-year-old construction worker man presents to your practice with the hand
deformity illustrated in the accompanying photograph. He works with a drilling
machine and has tingling sensation in his hand after work. He drinks 2 bottles of
beers every day and even more on weekends. Which one of the following could be

the most likey cause of his problem?


A Alcohol consumption.
B An autoimmune process.
C Vibration injury.
D Direct trauma.
E Injury to ulnar nerve.
Option A is correct The picture shows puckering of palmar skin and a nodule at the base
of fourth (ring) finger which are highly suggestive of Dupuytren contracture (DC). DC is
caused by fibrous hyperplasia of palmar fascia leading to nodular formation and
contracture over the fourth and fifth fingers in particular. This condition occurs in 10% of
males over the age of 65 years particularly in those of North European descent. There is a
genetic predisposition for the diseas,e and the condition is assumed to have an autosomal
dominant inheritance, but for clinical presentation to develop, other factors are
hypothesized to be involved. An association between DC and some conditions have been
well-established. These conditions are as follows: Male gender Alcohol Excess Smoking
Liver cirrhosis COPD Diabetes mellitus https://a-
medex.com/mcqs/show/2e4c22eb72f41a222beaccb4e8125dbf557f4c8b426c?page=139
10/31/18, 11C27 PM Page 2 of 3 Heavy manual labour Of the above factors, the most
common risk factors in a descending order are: Age Alcohol excess Sex (male) Previous
hand injuries With 2 bottles of beers a day and more on weekends, this man’s alcohol
consumption is beyond the standard 2 drinks per day for men; therefore, alcohol
consumption can be the most likely contributing factor to his condition. Each bottle of
beer contains 1.5 - 2 units of alcohol depending on the size and the alcohol concetraion.
This amounts to 3-4 units per day. The pathophysiology of DC is not well-understood.
Proliferation of fibrocytes and inflammatory processes are speculated to be the most
likely pathophysiology. The disease is not autoimmune. No nerve is involved in DC.

AMC Q BANK OF 1500 MCQs

You are one of the senior residents in surgery.You are called to see Mr Hull, a 65 years
old man who has just been diagnosed by emergency consultant to have acute cholangitis.

Two final year medical students are with you as well.They are keen to learn about
management of acute cholangitis.
Which of the following statement is not correct?

a. Plan for immediate decompression if a patient does not response after initial
measures

b. Plan for biliary decompression on semi-urgent basis(<72 hours) if patient


responding to initial resuscitation measures

c. Plan for urgent decompression (within 24-48hrs) if patient is more than 70 year old

d. The most appropriate method of biliary decompression is ERCP Sphincterectomy


and stenting

e. Initial aggressive resuscitation and antibiotics usually fail to get good response in
majority

Correct Answer & Detail

Correct Answer: e. Initial aggressive resuscitation and antibiotics usually fail to get
good response in majority

Answer Detail:

The principles of management of acute cholangitis involve initial aggressive fluid


resuscitation and intravenous antibiotics followed by biliary decompression.

The most appropriate method of biliary decompression is ERCP with sphincterectomy


and stenting.

About 10%-15% of patients will show no or minimal response after initial measures and
will continue to have hypotension,renal failure, pyrexia etc.So plan for immediate biliary
decompression in these patients.
Patients with poor prognostic parameter – (elderly patients; associated
comorbidities)needs urgent decompression (within 24-48hrs).

About 85%-90% of patients will respond to initial measures and plan for biliary
decompression on semi-urgent basis within 72 hours. So option e is correct answer.

A young-male is brought in hospital by an ambulance after motor vehicle accident. He is


clinically stable and sustained minor bruises at multiple areas with no major bleed.

He is now complaining of inability void and blood is noticed at urethral meatus.

What would be the next step in his management?

a. Retrograde urethrogram

b. Foleys catheter

c. Trial of void in 30 minutes

d. Bladder ultrasound

e. Adequate analgesia

Correct Answer & Detail

Correct Answer: a. Retrograde urethrogram

Answer Detail:
The correct answer is a.

The signs and symptoms of urethral injury include:

- Blood at the urethral meatus.

- Gross hematuria.

- Inability to void.

- An absent or abnormally positioned prostate on digital rectal examination.

- Ecchymosis or hematoma involving the penis, scrotum, or perineum.

In cases of suspected urethral injury, it is imperative to evaluate the integrity of the urethra by
retrograde urethrogram prior to attempting placement of a Foley (bladder or urinary) catheter
to avoid worsening a partial disruption.

A 45-year old man presented to the emergency department with 15% burns. His weight is 80 kg.

Which of the following fluids replacement will be required for the first 24 hours?

a. 2 litre Hartman 1.8 L blood

b. 2 litre normal saline 1 L blood

c. 3 litre Hartman 1.8 L 5 % dextrose

d. 3 litre Hartman 1.8 L blood

e. 5 litres of normal saline 2 Litres of dextrose


Correct Answer & Detail

Correct Answer: c. 3 litre Hartman 1.8 L 5 % dextrose

Answer Detail:

The Parkland formula:

(4 mL/kg/percentage of total body surface area burned) is used to estimate fluid volume needs
in the first 24 h after the burn and determines the rate of IV fluid administration.

Fluid is given as lactated Ringer solution (Hartman) because large amounts of normal saline
could result in hyperchloremic acidosis.

Parkland Formula for Burns:

In an 80-kg man with a 15% total body surface area burn, fluid volume by the Parkland formula
would be: (4) (80) (15) =4800ml.

According to Parkland formula patient will need 4.8 liters of fluid in first 24 hours and 2.4 liters
has to be given in first 8 hours.

A 73-year-old male presented in emergency department with upper abdominal pain which
radiates to back. Patient had a syncopal episode while in the hospital. Blood pressure 85/50,
heart rate 100/min.

What would be the best next investigation?

a. CT aortogram

b. Erect abdominal X-ray

c. Non-contrast abdominal CT scan


d. Bedside ultrasound

e. ECG

Correct Answer & Detail

Correct Answer: d. Bedside ultrasound

Answer Detail:

This patient has clinical diagnosis of abdominal aortic aneurysm rupture and need urgent
evaluation with bedside ultrasound also called fast ultrasound.

Any patient with the clinical triad of abdominal and/or back pain, a pulsatile abdominal mass,
and hypotension should be emergently evaluated by a vascular surgeon.

While doing the bedside ultrasound, contact the vascular surgeon simultaneously.

Standard resuscitative manoeuvres (insertion of two large-bore IV catheters, initiation of cardiac


monitoring, and administration of supplemental oxygen) are required in the emergency
department.

Imaging should not delay the patient transfer to the operation theatre.

Which one of these hernias is least likely to strangulate?

a. Femoral hernia

b. Incisional hernia

c. Umbilical hernia
d. Indirect inguinal hernia

e. Direct inguinal hernia

Correct Answer & Detail

Correct Answer: e. Direct inguinal hernia


Answer Detail:
The correct answer is e.

Direct inguinal hernia most of the times, does not cause


marked symptoms or become very large, nor does it obstruct
or strangulate as frequently.However there can be no strict
rules and direct hernias can complicate.

Femoral hernias are more common in slender, often older


females, and are more prone to strangulation. It can be
difficult to differentiate between an incarcerated femoral hernia
and lymphadenitis.

Generally indirect inguinal hernia should be surgically repaired


because they become larger, cause symptoms and may
obstruct and strangulate.

Incisional and umbilical hernias can also obstruct and


strangulate at any time and need an elective repair.
A-79-year-old lady with history of hip surgery develops pain in left limb after few days post-
surgery.Left leg is swollen more than right leg and feels hot.
What is the most accurate investigation to establish the diagnosis?

a. Doppler ultrasound

b. X-ray of the limb

c. Contrast venography

d. Thermography

e. D-dimers

Correct Answer & Detail

Correct Answer: a. Doppler ultrasound


Answer Detail:
This patient has suspected diagnosis of deep vein thrombosis
and requires urgent Doppler ultrasound to establish the
diagnosis.

Doppler ultrasound is more than 90% sensitive and more than


95% specific for femoral and popliteal vein thrombosis but is
less accurate for iliac or calf vein thrombosis.

D-dimers if negative, rule out deep vein thrombosis or


pulmonary embolism.
If D-dimers are positive, further investigations like Doppler
ultrasound of legs or CT pulmonary angiography is needed to
diagnose deep vein thrombosis or pulmonary embolism
respectively.

As D-dimers level is positive in many clinical conditions,so it is


never considered as first line choice of investigation to rule out
PE or DVT.

Contrast venography was the definitive test for the diagnosis


of DVT but has been largely replaced by ultrasonography,
which is noninvasive, more readily available, and almost
equally accurate for detecting DVT.
During abdominal examination of a 75-year-old man,you found a pulsatile mass.Which of the
following is non-invasive investigation to rule out aortic aneurysm with 100% sensitivity and
specificity?

a. Abdominal ultrasound

b. CT angiogram

c. Angiography

d. MRI

e. Non contrast CT scan

Correct Answer & Detail

Correct Answer: a. Abdominal ultrasound


Answer Detail:
The correct answer is a.

Because clinical diagnosis of abdominal aortic aneurysm is


unreliable,clinician should have low threshold of arranging an
abdominal ultrasound for patients at risk of having it.

Ultrasound is reliable,non-invasive,readily available choice


100% sensitivity and specificity.

CT angiogram is also extremely useful investigation in this


situation however it is invasive and not readily available.

Angiography is another invasive way of knowing about


presence of aortic aneurysm.

MRI is preferably not used to assess blood vessels


diseases.Non-contrast CT scan is inadequate to give good view
of the aneurysm.
A 60-year old man presented with history of vomiting for 3 days. Vomit is clear in colour with
identifiable food colour. A few years ago he was treated with ranitidine.

Over the last 6 months he has been having intermittent epigastric pain, for which he has been
taking aspirin.

He also last 5 kg weight in recent few months.

What is the most likely diagnosis?

a. Chronic duodenal ulcer disease

b. Pancreatic cancer
c. Cancer of the stomach

d. Urinary tract infection

e. Drug-induced gastritis

Correct Answer & Detail

Correct Answer: c. Cancer of the stomach


Answer Detail:
This patient has clinical features of gastric carcinoma.

Factors in favour of diagnosis of gastric malignancy include:

- Age more than 55.


- Persistence of epigastric pain for 6 months and weight loss.
- Presence of food particles in the vomitus means early
dysphagia.
- Weight loss and persistent abdominal pain are the most
common symptoms at initial diagnosis of gastric carcinoma.

Weight loss usually results from insufficient caloric intake


rather than increased catabolism and may be attributable to
anorexia, nausea, abdominal pain, early satiety, and/or
dysphagia.
When present, abdominal pain tends to be epigastric, vague
and mild early in the disease but more severe and constant as
the disease progresses.

Dysphagia is a common presenting symptom in patients with


cancers arising in the proximal stomach (cardia) or at the
esophagogastric junction.
A 62-year old male is found to have painless hematuria and normal renal function.Which of the
following is the best investigation to reach the diagnosis?

a. Cystoscopy

b. Intravenous pyelography

c. Ultrasound

d. X-ray

e. CT scan

Correct Answer & Detail

Correct Answer: a. Cystoscopy


Answer Detail:
The correct answer is a.

Painless macroscopic hematuria is caused by following clinical


conditions:
1-Urinary tract malignancy.
2-Urinary calculi.
3-Urinary tract infection.
4-Benign prostatic hypertrophy.
5-Haemorrhagic cystitis.
6-Nephrological disease.
7-Bleeding diathesis/anticoagulation therapy.
8-Arteriovenous malformation/angiomyolipoma.

Macroscopic hematuria has a high diagnostic yield for


underlying malignancy, particularly in
appropriate age groups.In men aged more than 60 the positive
predictive value of macroscopic hematuria for urological
malignancy is 22.1 percent.

Given the high prevalence of malignancy, it is mandatory to


further evaluate patients with
suspected urological disease as the cause of macroscopic
hematuria.

If patient is less than 50 years of age,do CT scan of abdomen


and pelvis.All patients above 50 years require cystoscopy.

Cystureterocopy is the gold standard for lower urinary tract


malignancies.Ultrasound is effective in the detection of upper
tract tumours.

A 42 year-old lady presents with an incidental finding of calcified gallbladder on abdominal


ultrasound.She has mild dyspepsia and no other symptoms.What will be your best next step in
her management?
a. ERCP

b. No further action needed

c. Prophylactic cholecystectomy

d. MRI

e. CT angiogram

Correct Answer & Detail

Correct Answer: c. Prophylactic cholecystectomy


Answer Detail:
Prophylactic cholecystectomy is warranted in a patient with
calcified gallbladder.This calcified gallbladder is also called
porcelain gallbladder.Otherwise it increases the risk of cancer
of gallbladder.
A 55-year-old male presented with sudden onset of right sided leg pain and paresthesias.

On examination distal pulses are absent and the limb is cold.

Neurological examination is normal. CT angiogram shows femoral artery embolism. Patient is


given heparin and emergency embolectomy is done.

What will you do next?

a. Give aspirin life long

b. Give heparin for one week


c. Give warfarin life long

d. Give combination of aspirin and warfarin for 6 months

e. Refer patient to haematologist

Correct Answer & Detail

Correct Answer: c. Give warfarin life long


Answer Detail:
Any patient who develops acute limb ischemia and required
embolectomy or any other procedure would require lifelong
anti-coagulation with warfarin.

Aspirin is indicated in patients with peripheral arterial disease


causing chronic ischemic changes.

Heparin is only required while patient starts taking warfarin to


prevent skin necrosis caused by pro-coagulants and INR is in
therapeutic range 2-3. It usually takes 2-3 days. Heparin is
discontinued thereafter.

Combination of warfarin and aspirin do not give added


advantage however it increased the risk of bleeding
significantly.
Which one of the following is a key clinical feature of orbital floor fracture?
a. Diplopia

b. Epistaxis

c. Painful eye movement

d. Facial numbness

e. Decrease visual acuity

Correct Answer & Detail

Correct Answer: a. Diplopia


Answer Detail:
The correct answer is a.

The classic presentation of orbital fractures is the absence of


subconjunctival haemorrhage, with upgaze diplopia (double
vision) and general malaise. Pain on upward gaze is often an
accompanying symptom.

After facial trauma, patients may describe decreased visual


acuity however this is not a common feature of orbital floor
fracture.

Painful eye movement is a key feature of orbital cellulitis.


Facial numbness is a very non-specific symptom and may be
present in orbital floor fracture if infra-orbital nerve is
involved.
A-40-year-old woman presents with colicky pain in right upper quadrant and in epigastrium. The
pain is associated with diaphoresis, nausea, and vomiting. It is not exacerbated by movement
and is not relieved by squatting, bowel movements, or passage of flatus.

Which of the following is investigation of choice for diagnosis of this condition?

a. Abdominal ultrasound

b. CT Scan

c. MRI

d. Oral cholecystogram

e. Plain X-ray

Correct Answer & Detail

Correct Answer: a. Abdominal ultrasound


Answer Detail:
Gallstones are suspected in patients with biliary colic.

Abdominal ultrasonography is the investigation of choice for


detecting gallstones.It has sensitivity and specificity of 95
percent.

CT scan,MRI and oral cholecystography are alternate options


to investigate a patient with biliary colic.
Oral cholecystogram is rarely used now although it used to be
quite accurate.

Endoscopic ultrasonography accurately detects small


gallstones less than 3 mm and may be needed if other tests
are equivocal.

About 10 to 15 percent of gallstones are calcified and visible


on plain x-rays.
Achalasia is a neurogenic esophageal motility disorder
characterized by impaired esophageal peristalsis, a lack of
lower esophageal sphincter relaxation during swallowing, and
an elevation of lower esophageal sphincter resting pressure.
All of the following features favour the diagnosis of achalasia, except?

a. Dysphagia of liquids

b. Regurgitation of food

c. Painful swallowing

d. Nocturnal regurgitation

e. Dysphagia of solids

Correct Answer & Detail

Correct Answer: c. Painful swallowing


Achalasia occurs at any age but usually begins between ages
20 and 60. Onset is insidious, and progression is gradual over
months or years.
Dysphagia for both solids and liquids is the major
symptom.Nocturnal regurgitation of undigested food occurs in
about 35 percent of patients and may cause cough and
pulmonary aspiration.
When the diagnosis of achalasia is suspected, a barium
esophagram with fluoroscopy is the single best diagnostic
study.This test will reveal loss of primary peristalsis in the
distal two-thirds of the oesophagus with to-and-fro movement
in the supine position.
Odynophagia is a condition in which an individual experiences
pain each time he or she swallows.It is not a feature of
achalasia.
Which of the following is best test to diagnose sliding hiatus hernia?

a. Barium swallow

b. Upper GI endoscopy

c. Twenty four hours pH monitoring

d. Manometry

e. Esophageal biopsy

Correct Answer & Detail


Correct Answer: a. Barium swallow
Answer Detail:
Barium swallow is best diagnostic test for sliding hiatus hernia
and rolling hiatus hernia.

Upper GI endoscopy visualizes the mucosa however cannot


reliably exclude hiatus hernia.

24-hour pH monitoring is required to diagnose


gastroesophageal reflux disease with reliability.

Manometry is usually performed along with pH monitoring.

Esophageal biopsy does not help in diagnosing hiatus hernia.

A 34-year-old male presented for your advice regarding recurrent pilonidal sinus after surgical
treatment performed 4 months ago. What is the most appropriate advice to the patient to avoid
recurrence?

a. Keep plastic bandages on for another 2 months

b. Refer to another GP

c. Review the patient daily for 2 weeks

d. Take advice from hospital director

e. Shave the area and keep it clean


Correct Answer & Detail

Correct Answer: e. Shave the area and keep it clean


Answer Detail:
Pilonidal sinus is a skin condition in the midline of the natal
cleft. A primary pit forms in the midline, caused by a hair
follicle that has become infected, into which loose hairs enter
to create a track or abscess.

Prevention from recurrent pilonidal sinus disease:

1- Keep the area clean and dry.


2- Avoid sitting for a long time on hard surfaces.
3- Remove hair from the area
Which of the following is the most common cause of death after burn injury in Australia?

a. Respiratory failure

b. Cardiac failure

c. Renal failure

d. Delayed wound healing

e. Shock

Correct Answer & Detail


Correct Answer: e. Shock
Answer Detail:
The most common cause of death after burn injury is septic
shock leading to multiple organ failure.Infection leading to
septic shock is the most common complication of burns and is
the major cause of death in burn victims.
Septic shock by definition is about 43 to 54 percent of fluid
loss.Septic shock is best defined by a systolic blood pressure
less than 90 mmHg or a mean arterial pressure less than 65
mmHg after a crystalloid fluid challenge of 30 ml per kg body
weight in a patient with severe sepsis.
Respiratory failure,cardiac failure and renal failure can be the
sequel of septic shock.However non of them is the most
common cause of death in burn patients in isolation.
A 73-year old male presented with left leg swelling and was found to have deep vein thrombosis
in calf as well as in popliteal and femoral veins on Doppler ultrasound.He had not history of
deep vein thrombosis in the past.

What is the best initial treatment?

a. Angioplasty

b. Angiography

c. Warfarin

d. Heparin

e. Aspirin
Correct Answer & Detail

Correct Answer: d. Heparin


Answer Detail:
The correct answer is d.

All patients with deep vein thrombosis are given


anticoagulants. Initially an injectable heparin (unfractionated
or low molecular weight) is given followed by warfarin started
within 24 to 48 hours.

Inadequate anticoagulation in the first 24 to 48 hours may


increase risk of recurrence or pulmonary embolism.

After starting warfarin, daily INR is tested and heparin is


continued until INR is in therapeutic range (2-3 in this case).

Once INR is in therapeutic range, heparin can be discontinued and


warfarin should be continued for 6 months.Patient should see GP
regularly for INR monitoring

A 57-year-old male presents with fever, right upper quadrant pain, hypotension and tachycardia.
On examination, there is right upper quadrant pain.

Blood tests show leukocytosis and mildly elevated bilirubin. You suspect gallbladder sepsis.

Which of the following is the cause of the septic shock?


a. Staphylococcus aureus Should be streptococcus

b. Pseudomonas aeruginosa

c. Enterococcus faecalis

d. Escherichia coli

e. Treponema pallidum

Correct Answer & Detail

Correct Answer: d. Escherichia coli


Answer Detail:
The correct answer is d.

This patient has suspected clinical diagnosis of acute infective


cholecystitis. It is caused by bacteria in about 50-80% patients
with acute cholecystitis.

Acute inflammation of the gallbladder wall usually follows


obstruction of the cystic duct by a stone.

The organisms most frequently isolated by culture of


gallbladder bile in these patients include Escherichia coli,
Klebsiella spp, Streptococcus spp, and Clostridium spp.
A 17-year-old male presented with right testicular pain for 2 days. On examination,scrotum is
tender and swollen. Spermatic cord is thickened and one can go above the lump.
What is the most appropriate next step?

a. Surgery

b. Ultrasound

c. Urine microscopy and culture

d. Full blood count

e. CT scan

Correct Answer & Detail

Correct Answer: a. Surgery


Answer Detail:
The correct answer is a.

A painful scrotum should be taken seriously.This is because of


the possibility of testicular torsion, which can cause infarction
(death of the testicle by cutting off its blood supply) in a
matter of 4-6 hours.

Physical examination demonstrates a swollen, tender and firm


hemiscrotum with a short and thickened spermatic cord when
testicular torsion is present.

According to Australian guidelines,no person younger than 18-


years should be diagnosed as suffering from acute epididymo-
orchitis until the testis has been exposed at operation and
torsion excluded.So option A is correct here.

Role of ultrasound to distinguish between testicular torsion and


epididymo-orchitis is controversial as it can not reliably detect
changes that are diagnostic of an early torsion.

A technetium-99 m scan can differentiate between the two


conditions-in torsion,testis is avascular while it is hyperaemic
in epididymo-orchitis.

Once torsion of testis has been ruled out,full blood count,urine


microscopy and chlamydia antigen detection test are done to
find out the cause of epididymo-orchitis.It can be treated by
antibiotics.
A 45-year-old male presented with sharp,burning,stabbing pain in lateral aspect of the right
foot.You suspect L5-S1 radiculopathy.Which of the following is lost in L5-S1 disc prolapse?

a. Knee jerk

b. Dorsiflexion of big toe and foot

c. Plantar flexion

d. Eversion of foot

e. Ankle jerk
Correct Answer & Detail

Correct Answer: e. Ankle jerk


Answer Detail:
A herniation of the disc between the L5 and S1 vertebrae will
impinge on the S1 spinal nerve, which exits between the S1
and S2 vertebrae.In L5-S1 disc prolapse,ankle reflex is lost.A
brief description of signs and symptoms of nerve root
involvement at each level in lumbar region is given below.
1. T12-L1-Pain in inguinal region and medial thigh.
2. L1-2- Pain in anterior and medial aspect of upper thigh-
slight weakness in quadriceps
3. L2-3-Pain in anterolateral thigh and weakness of
quadriceps leading to weak of knee reflex.
4. L3-4-Pain in posterolateral thigh and anterior tibial area
with diminished knee reflex.
5. L4-5-Pain dorsum of foot with weak dorsiflexion of big toe
and foot.
6. L5-S1-Pain in lateral aspect of foot with absent ankle
jerk.
A 77-year-old woman presented with low back pain. She has history of colon cancer surgery 5
years ago. On examination she is tender at T7-T8 level. There is no history of trauma.
Investigations show elevated PTH and alkaline phosphatase. What is the most likely diagnosis?

a. Malignant metastasis

b. Multiple myeloma

c. Musculoskeletal back pain

d. Vertebral Fracture

e. Disc prolapse
Correct Answer & Detail

Correct Answer: d. Vertebral Fracture


Answer Detail:
Alkaline phosphatase levels are usually elevated in
hyperparathyroidism. So this patient have primary
hyperparathyroidism and is high risk for fracture biochemically.
Also she is postmenopausal and high risk of fracture due to
osteoporosis. So d is correct.

Intact PTH concentrations are generally undetectable or very


low in hypercalcaemia of malignancy and are elevated or high-
normal in primary hyperparathyroidism. So option a is
incorrect.

It is unlikely multiple myeloma as there is no information


regarding osteolytic bone lesions, hypercalcaemia, anemia and
renal failure. So option b is incorrect.

There are multiple causes of elevation of serum alkaline


phosphatase including bone disease, liver disease, and
pregnancy. Alkaline phosphatase is elevated in conditions
associated with high bone turnover. These include Paget’s
disease, hyperthyroidism, fractures and hyperparathyroidism.

PTH is secreted almost instantaneously in response to very


small reductions in serum ionized calcium, which are sensed by
the calcium-sensing receptor. The increase in PTH release
raises the serum calcium concentration toward normal via
three actions.
1- Increased bone resorption, which occurs within minutes
after PTH secretion increases.
2- Increased intestinal calcium absorption mediated by
increased production of calcitriol, the most active form of
vitamin D, which occurs days after PTH secretion increases.
3- Decreased urinary calcium excretion due to stimulation of
calcium reabsorption in the distal tubule, which occurs within
minutes after PTH secretion increases.

It is uncommon for patients with hypercalcaemia of


malignancy to have elevated PTH levels.
A 73-year-old male, type ll diabetic, presented with sudden onset of horizontal diplopia, better
at near and worse at distance. On examination, left eye is deviated towards medial side.

What is the most likely diagnosis?

a. 3rd nerve palsy

b. 6th nerve palsy

c. 4th nerve palsy

d. 2nd nerve palsy

e. Facial fracture

Correct Answer & Detail


Correct Answer: b. 6th nerve palsy
Answer Detail:
Sixth nerve palsy causes isolated weakness of abduction.
Patients with sixth cranial nerve palsies primarily complain of
horizontal diplopia.
On examination there is an esotropia (inward deviation) that is
worsened with gaze into the field of the weak lateral rectus
muscle. Abduction is commonly limited on the side of the
lesion.

Poorly controlled diabetes is a predisposing factor.

The 3rd cranial nerve supplies the levator muscle of the eyelid
and four extraocular muscles: the medial rectus, superior
rectus, inferior rectus, and inferior oblique. These muscles
adduct, depress, and elevate the eye.
Patients with acute acquired third nerve palsy usually complain
of the sudden onset of binocular horizontal, vertical, or oblique
diplopia and a droopy eyelid.

A person with 4th nerve palsies may complain of binocular


(present with both eyes open) vertical diplopia and/or
subjective tilting of objects (torsional diplopia).

The affected eye usually is extorted because the superior


oblique muscle is an intorter of the eye. Objects viewed in
primary position or especially in down-gaze may appear double
when going down a flight of stairs so that the child does not
know which step to select).

You might also like